Location via proxy:   [ UP ]  
[Report a bug]   [Manage cookies]                

Disjointly strictly singular inclusions between variable Lebesgue spaces

Francisco L. Hernández, César Ruiz and Mauro Sanchiz∗∗ IMI and Departamento de Análisis Matemático y Matemática Aplicada, Facultad de Matemáticas, Universidad Complutense, 28040 Madrid, Spain (F.L. Hernández) pacoh@ucm.es (C. Ruiz) cruizb@mat.ucm.es (M. Sanchiz) mauro.sanchizalonso@ceu.es
Abstract.

Disjointly strictly singular inclusions between variable Lebesgue spaces Lp()(μ)superscript𝐿𝑝𝜇{L^{p(\cdot)}(\mu)}italic_L start_POSTSUPERSCRIPT italic_p ( ⋅ ) end_POSTSUPERSCRIPT ( italic_μ ) on finite measure are characterized. Suitable criteria in terms of the (bounded or unbounded) exponents are given. It is proved the equivalence of L𝐿Litalic_L-weak compactness (also called almost compactness) and disjoint strict singularity for variable Lebesgue space inclusions. For infinite measure any inclusion  Lp()(μ)Lq()(μ)superscript𝐿𝑝𝜇superscript𝐿𝑞𝜇{L^{p(\cdot)}(\mu)}\hookrightarrow{L^{q(\cdot)}(\mu)}italic_L start_POSTSUPERSCRIPT italic_p ( ⋅ ) end_POSTSUPERSCRIPT ( italic_μ ) ↪ italic_L start_POSTSUPERSCRIPT italic_q ( ⋅ ) end_POSTSUPERSCRIPT ( italic_μ )  is not disjointly strictly singular. No restrictions on the exponent are imposed.

2000 Mathematics Subject Classification:
46E30, 47B60
Partially supported by grant PID2019-107701G-I00
∗∗ Partially supported by grant PID2019-107701G-I00 and scholarship CT42/18-CT43/18

1. Introduction

A linear operator T𝑇Titalic_T between two Banach spaces E𝐸Eitalic_E and F𝐹Fitalic_F is strictly singular (or Kato) if T𝑇Titalic_T fails to be an isomorphism on any infinite-dimensional (closed) subspace of E𝐸Eitalic_E, i.e. given ϵ>0italic-ϵ0\epsilon>0italic_ϵ > 0 and an infinite dimensional subspace E0subscript𝐸0E_{0}italic_E start_POSTSUBSCRIPT 0 end_POSTSUBSCRIPT of E𝐸Eitalic_E there exists an unitary vector xE0𝑥subscript𝐸0x\in E_{0}italic_x ∈ italic_E start_POSTSUBSCRIPT 0 end_POSTSUBSCRIPT such that Txϵnorm𝑇𝑥italic-ϵ\|Tx\|\leq\epsilon∥ italic_T italic_x ∥ ≤ italic_ϵ. In the context of Banach lattices E𝐸Eitalic_E a useful weaker notion is that an operator T𝑇Titalic_T from E𝐸Eitalic_E to F𝐹Fitalic_F is said to be disjointly strictly singular (DSS in short) if there is no disjoint sequence of non-null vectors (xn)subscript𝑥𝑛(x_{n})( italic_x start_POSTSUBSCRIPT italic_n end_POSTSUBSCRIPT ) in E𝐸Eitalic_E such that the restriction of T𝑇Titalic_T to the (closed) subspace   [xn]delimited-[]subscript𝑥𝑛[x_{n}][ italic_x start_POSTSUBSCRIPT italic_n end_POSTSUBSCRIPT ]  spanned by (xn)subscript𝑥𝑛(x_{n})( italic_x start_POSTSUBSCRIPT italic_n end_POSTSUBSCRIPT ) is an isomorphism.

The study of strictly and disjointly strictly singular inclusions have been quite extensive for symmetric (or rearrangement invariant) function spaces. Recall that for symmetric function spaces  E(μ)𝐸𝜇E(\mu)italic_E ( italic_μ )  on finite measures the left canonic inclusions of  L(μ)superscript𝐿𝜇L^{\infty}(\mu)italic_L start_POSTSUPERSCRIPT ∞ end_POSTSUPERSCRIPT ( italic_μ ) in  E(μ)𝐸𝜇E(\mu)italic_E ( italic_μ )  is always strictly singular, while the right inclusion of  E(μ)𝐸𝜇E(\mu)italic_E ( italic_μ ) in  L1(μ)superscript𝐿1𝜇L^{1}(\mu)italic_L start_POSTSUPERSCRIPT 1 end_POSTSUPERSCRIPT ( italic_μ )  is disjointly strictly singular. And this inclusion  E(μ)L1(μ)𝐸𝜇superscript𝐿1𝜇E(\mu)\hookrightarrow L^{1}(\mu)italic_E ( italic_μ ) ↪ italic_L start_POSTSUPERSCRIPT 1 end_POSTSUPERSCRIPT ( italic_μ )  is strictly singular if and only if the Orlicz space  L0expx2subscriptsuperscript𝐿superscript𝑥20L^{\exp x^{2}}_{0}italic_L start_POSTSUPERSCRIPT roman_exp italic_x start_POSTSUPERSCRIPT 2 end_POSTSUPERSCRIPT end_POSTSUPERSCRIPT start_POSTSUBSCRIPT 0 end_POSTSUBSCRIPT  cannot be included in  E(μ)𝐸𝜇E(\mu)italic_E ( italic_μ ). When considering two symmetric function spaces with  E(μ)F(μ)𝐸𝜇𝐹𝜇E(\mu)\hookrightarrow F(\mu)italic_E ( italic_μ ) ↪ italic_F ( italic_μ )  this inclusion i𝑖iitalic_i is strictly singular if and only if  i𝑖iitalic_i  is disjointly strictly singular and the norms of  E(μ)𝐸𝜇E(\mu)italic_E ( italic_μ )  and  F(μ)𝐹𝜇F(\mu)italic_F ( italic_μ )  are not equivalent on  [rn]E(μ)subscriptdelimited-[]subscript𝑟𝑛𝐸𝜇[r_{n}]_{E(\mu)}[ italic_r start_POSTSUBSCRIPT italic_n end_POSTSUBSCRIPT ] start_POSTSUBSCRIPT italic_E ( italic_μ ) end_POSTSUBSCRIPT  and [rn]F(μ)subscriptdelimited-[]subscript𝑟𝑛𝐹𝜇[r_{n}]_{F(\mu)}[ italic_r start_POSTSUBSCRIPT italic_n end_POSTSUBSCRIPT ] start_POSTSUBSCRIPT italic_F ( italic_μ ) end_POSTSUBSCRIPT, the subspaces spanned by the Rademacher functions (rn)subscript𝑟𝑛(r_{n})( italic_r start_POSTSUBSCRIPT italic_n end_POSTSUBSCRIPT ) ([4, 12, 18]). This strengthens the interest in knowing characterizations of disjointly strictly singular inclusions for distinguished classes of function spaces (see [5, 15, 19] and references within).

One of the goals of this paper is to study the disjoint strict singularity of inclusion operators between variable Lebesgue spaces (or Nakano spaces)  Lp()(μ)superscript𝐿𝑝𝜇{L^{p(\cdot)}(\mu)}italic_L start_POSTSUPERSCRIPT italic_p ( ⋅ ) end_POSTSUPERSCRIPT ( italic_μ )   for finite and infinite measures. These non-symmetric classical function spaces  Lp()(μ)superscript𝐿𝑝𝜇{L^{p(\cdot)}(\mu)}italic_L start_POSTSUPERSCRIPT italic_p ( ⋅ ) end_POSTSUPERSCRIPT ( italic_μ )  have seen a strong renewed relevance in the last decades due to their applications (cf. [8, 7]).

In this context of variable spaces the inclusion behavior is more diverse than in the symmetric case. Compact and weakly compact inclusions have been considered in [17, 11, 9, 22]. The study of L𝐿Litalic_L-weak compactness of variable space inclusions is motivated by its applications to the compactness of associated Sobolev embeddings (see [9, 11]). Recall that an operator T𝑇Titalic_T between two Banach function lattices E𝐸Eitalic_E and F𝐹Fitalic_F on a measure space (Ω,μ)Ω𝜇(\Omega,\mu)( roman_Ω , italic_μ ) is said to be L𝐿Litalic_L-weakly compact (or almost compact) whenever T(BE)𝑇subscript𝐵𝐸T(B_{E})italic_T ( italic_B start_POSTSUBSCRIPT italic_E end_POSTSUBSCRIPT ) is a equi-integrable subset in  F𝐹Fitalic_F  for BEsubscript𝐵𝐸B_{E}italic_B start_POSTSUBSCRIPT italic_E end_POSTSUBSCRIPT  denoting the unit ball of E𝐸Eitalic_E, i.e.

limnsupfBE{TfχAnF}=0,subscript𝑛subscriptsupremum𝑓subscript𝐵𝐸subscriptnorm𝑇𝑓subscript𝜒subscript𝐴𝑛𝐹0\lim_{n\rightarrow\infty}\sup_{f\in B_{E}}\{||Tf\chi_{A_{n}}||_{F}\}=0,roman_lim start_POSTSUBSCRIPT italic_n → ∞ end_POSTSUBSCRIPT roman_sup start_POSTSUBSCRIPT italic_f ∈ italic_B start_POSTSUBSCRIPT italic_E end_POSTSUBSCRIPT end_POSTSUBSCRIPT { | | italic_T italic_f italic_χ start_POSTSUBSCRIPT italic_A start_POSTSUBSCRIPT italic_n end_POSTSUBSCRIPT end_POSTSUBSCRIPT | | start_POSTSUBSCRIPT italic_F end_POSTSUBSCRIPT } = 0 ,

for every sequence  (An)subscript𝐴𝑛(A_{n})( italic_A start_POSTSUBSCRIPT italic_n end_POSTSUBSCRIPT )  of measurable sets in ΩΩ\Omegaroman_Ω with  χAn0subscript𝜒subscript𝐴𝑛0\chi_{A_{n}}\rightarrow 0italic_χ start_POSTSUBSCRIPT italic_A start_POSTSUBSCRIPT italic_n end_POSTSUBSCRIPT end_POSTSUBSCRIPT → 0 μ𝜇\muitalic_μ-a.e.. In [9] (Thm. 3.4) Edmunds, Gogathisvili and Nekvinda have given the following L𝐿Litalic_L-weak compactness criterion for bounded exponents defined on bounded open subsets  ΩΩ\Omegaroman_Ω  of  nsuperscript𝑛\mathbb{R}^{n}blackboard_R start_POSTSUPERSCRIPT italic_n end_POSTSUPERSCRIPT with Lebesgue measure |||\cdot|| ⋅ |: an inclusion Lp()(Ω)Lq()(Ω)superscript𝐿𝑝Ωsuperscript𝐿𝑞ΩL^{p(\cdot)}(\Omega)\hookrightarrow L^{q(\cdot)}(\Omega)italic_L start_POSTSUPERSCRIPT italic_p ( ⋅ ) end_POSTSUPERSCRIPT ( roman_Ω ) ↪ italic_L start_POSTSUPERSCRIPT italic_q ( ⋅ ) end_POSTSUPERSCRIPT ( roman_Ω )  is L𝐿Litalic_L-weakly compact if and only if for every  a>1𝑎1a>1italic_a > 1,

0|Ω|a(1pq)(x)𝑑x<,superscriptsubscript0Ωsuperscript𝑎superscript1𝑝𝑞𝑥differential-d𝑥\int_{0}^{|\Omega|}a^{\left(\frac{1}{p-q}\right)^{*}(x)}dx<\infty,∫ start_POSTSUBSCRIPT 0 end_POSTSUBSCRIPT start_POSTSUPERSCRIPT | roman_Ω | end_POSTSUPERSCRIPT italic_a start_POSTSUPERSCRIPT ( divide start_ARG 1 end_ARG start_ARG italic_p - italic_q end_ARG ) start_POSTSUPERSCRIPT ∗ end_POSTSUPERSCRIPT ( italic_x ) end_POSTSUPERSCRIPT italic_d italic_x < ∞ ,

where (1pq)(x)superscript1𝑝𝑞𝑥(\frac{1}{p-q})^{*}(x)( divide start_ARG 1 end_ARG start_ARG italic_p - italic_q end_ARG ) start_POSTSUPERSCRIPT ∗ end_POSTSUPERSCRIPT ( italic_x ) denotes the decreasing rearrangement of (1pq)(t)1𝑝𝑞𝑡(\frac{1}{p-q})(t)( divide start_ARG 1 end_ARG start_ARG italic_p - italic_q end_ARG ) ( italic_t ). Another L𝐿Litalic_L-weak compactness criterion in Lp()(μ)superscript𝐿𝑝𝜇{L^{p(\cdot)}(\mu)}italic_L start_POSTSUPERSCRIPT italic_p ( ⋅ ) end_POSTSUPERSCRIPT ( italic_μ ) (of De la Valleé-Pousin type) has been given in [22] (Prop. 3.3).

The study of disjointly strictly singular inclusions  Lp()(μ)Lq()(μ)superscript𝐿𝑝𝜇superscript𝐿𝑞𝜇{L^{p(\cdot)}(\mu)}\hookrightarrow{L^{q(\cdot)}(\mu)}italic_L start_POSTSUPERSCRIPT italic_p ( ⋅ ) end_POSTSUPERSCRIPT ( italic_μ ) ↪ italic_L start_POSTSUPERSCRIPT italic_q ( ⋅ ) end_POSTSUPERSCRIPT ( italic_μ )  was initiated in [13]. In the present paper we continue this research line obtaining now complete characterizations of disjointly strictly singular inclusions and L𝐿Litalic_L-weakly compact inclusions  Lp()(μ)Lq()(μ)superscript𝐿𝑝𝜇superscript𝐿𝑞𝜇{L^{p(\cdot)}(\mu)}\hookrightarrow{L^{q(\cdot)}(\mu)}italic_L start_POSTSUPERSCRIPT italic_p ( ⋅ ) end_POSTSUPERSCRIPT ( italic_μ ) ↪ italic_L start_POSTSUPERSCRIPT italic_q ( ⋅ ) end_POSTSUPERSCRIPT ( italic_μ ), giving suitable conditions on the exponents. It comes out the equivalence of these two concepts in this setting of variable Lebesgue space inclusions (a fact rather unexpected according with the Orlicz space behavior, see Section 2). The strict singularity of inclusions  L(μ)Lp()(μ)superscript𝐿𝜇superscript𝐿𝑝𝜇L^{\infty}(\mu)\hookrightarrow{L^{p(\cdot)}(\mu)}italic_L start_POSTSUPERSCRIPT ∞ end_POSTSUPERSCRIPT ( italic_μ ) ↪ italic_L start_POSTSUPERSCRIPT italic_p ( ⋅ ) end_POSTSUPERSCRIPT ( italic_μ )  is also studied giving suitable criteria for it.

The paper is divided in 6 sections. Section 2 recall some definitions and basic results. Section 3 contains some useful preliminary results on Lp()(μ)superscript𝐿𝑝𝜇{L^{p(\cdot)}(\mu)}italic_L start_POSTSUPERSCRIPT italic_p ( ⋅ ) end_POSTSUPERSCRIPT ( italic_μ ) spaces and decreasing rearrangement functions. Thus Proposition 3.3 states, by an analysis of disjoint function sequences  (χEnμ(En)1p(t))subscript𝜒subscript𝐸𝑛𝜇superscriptsubscript𝐸𝑛1𝑝𝑡(\frac{\chi_{E_{n}}}{\mu(E_{n})^{\frac{1}{p(t)}}})( divide start_ARG italic_χ start_POSTSUBSCRIPT italic_E start_POSTSUBSCRIPT italic_n end_POSTSUBSCRIPT end_POSTSUBSCRIPT end_ARG start_ARG italic_μ ( italic_E start_POSTSUBSCRIPT italic_n end_POSTSUBSCRIPT ) start_POSTSUPERSCRIPT divide start_ARG 1 end_ARG start_ARG italic_p ( italic_t ) end_ARG end_POSTSUPERSCRIPT end_ARG ), that if an inclusion  Lp()(μ)Lq()(μ)superscript𝐿𝑝𝜇superscript𝐿𝑞𝜇{L^{p(\cdot)}(\mu)}\hookrightarrow{L^{q(\cdot)}(\mu)}italic_L start_POSTSUPERSCRIPT italic_p ( ⋅ ) end_POSTSUPERSCRIPT ( italic_μ ) ↪ italic_L start_POSTSUPERSCRIPT italic_q ( ⋅ ) end_POSTSUPERSCRIPT ( italic_μ )  is disjointly strictly singular, then

limxμ(Ω)(μ(Ω)x)(pqpq)(x)=0.subscript𝑥𝜇superscriptΩsuperscript𝜇Ω𝑥superscript𝑝𝑞𝑝𝑞𝑥0\lim_{x\rightarrow\mu(\Omega)^{-}}\,(\mu(\Omega)-x)^{(\frac{p-q}{p\,q})^{*}(x)% }=0.roman_lim start_POSTSUBSCRIPT italic_x → italic_μ ( roman_Ω ) start_POSTSUPERSCRIPT - end_POSTSUPERSCRIPT end_POSTSUBSCRIPT ( italic_μ ( roman_Ω ) - italic_x ) start_POSTSUPERSCRIPT ( divide start_ARG italic_p - italic_q end_ARG start_ARG italic_p italic_q end_ARG ) start_POSTSUPERSCRIPT ∗ end_POSTSUPERSCRIPT ( italic_x ) end_POSTSUPERSCRIPT = 0 .

In Section 4, disjointly strictly singular inclusions Lp()(μ)Lq()(μ)superscript𝐿𝑝𝜇superscript𝐿𝑞𝜇{L^{p(\cdot)}(\mu)}\hookrightarrow{L^{q(\cdot)}(\mu)}italic_L start_POSTSUPERSCRIPT italic_p ( ⋅ ) end_POSTSUPERSCRIPT ( italic_μ ) ↪ italic_L start_POSTSUPERSCRIPT italic_q ( ⋅ ) end_POSTSUPERSCRIPT ( italic_μ ) for finite measures are studied, looking for suitable criteria on the exponents. First we do under the hypothesis of the exponent  q()𝑞q(\cdot)italic_q ( ⋅ )  be bounded (Theorem 4.1). After that we consider the general case, thus Theorem 4.8 claims the equivalence of the following statements for exponents   q()<p()𝑞𝑝q(\cdot)<p(\cdot)italic_q ( ⋅ ) < italic_p ( ⋅ ) μ𝜇\muitalic_μ-a.e. on a finite measure space:

  1. (1)

    The inclusion  Lp()(μ)Lq()(μ)superscript𝐿𝑝𝜇superscript𝐿𝑞𝜇{L^{p(\cdot)}(\mu)}\hookrightarrow{L^{q(\cdot)}(\mu)}italic_L start_POSTSUPERSCRIPT italic_p ( ⋅ ) end_POSTSUPERSCRIPT ( italic_μ ) ↪ italic_L start_POSTSUPERSCRIPT italic_q ( ⋅ ) end_POSTSUPERSCRIPT ( italic_μ )  is L𝐿Litalic_L-weakly compact.

  2. (2)

    The inclusion Lp()(μ)Lq()(μ)superscript𝐿𝑝𝜇superscript𝐿𝑞𝜇{L^{p(\cdot)}(\mu)}\hookrightarrow{L^{q(\cdot)}(\mu)}italic_L start_POSTSUPERSCRIPT italic_p ( ⋅ ) end_POSTSUPERSCRIPT ( italic_μ ) ↪ italic_L start_POSTSUPERSCRIPT italic_q ( ⋅ ) end_POSTSUPERSCRIPT ( italic_μ )  is disjointly strictly singular.

  3. (3)

    limxμ(Ω)(μ(Ω)x)(pqpq)(x)=0subscript𝑥𝜇superscriptΩsuperscript𝜇Ω𝑥superscript𝑝𝑞𝑝𝑞𝑥0\lim_{x\rightarrow\mu(\Omega)^{-}}\,\,(\mu(\Omega)-x)^{(\frac{p-q}{p\,q})^{*}(% x)}=0roman_lim start_POSTSUBSCRIPT italic_x → italic_μ ( roman_Ω ) start_POSTSUPERSCRIPT - end_POSTSUPERSCRIPT end_POSTSUBSCRIPT ( italic_μ ( roman_Ω ) - italic_x ) start_POSTSUPERSCRIPT ( divide start_ARG italic_p - italic_q end_ARG start_ARG italic_p italic_q end_ARG ) start_POSTSUPERSCRIPT ∗ end_POSTSUPERSCRIPT ( italic_x ) end_POSTSUPERSCRIPT = 0.

  4. (4)

    0μ(Ω)a(pqpq)(x)𝑑x<superscriptsubscript0𝜇Ωsuperscript𝑎superscript𝑝𝑞𝑝𝑞𝑥differential-d𝑥\int_{0}^{\mu(\Omega)}a^{(\frac{p\,q}{p-q})^{*}(x)}dx<\infty∫ start_POSTSUBSCRIPT 0 end_POSTSUBSCRIPT start_POSTSUPERSCRIPT italic_μ ( roman_Ω ) end_POSTSUPERSCRIPT italic_a start_POSTSUPERSCRIPT ( divide start_ARG italic_p italic_q end_ARG start_ARG italic_p - italic_q end_ARG ) start_POSTSUPERSCRIPT ∗ end_POSTSUPERSCRIPT ( italic_x ) end_POSTSUPERSCRIPT italic_d italic_x < ∞     for every a>1𝑎1a>1italic_a > 1.

Thus the above L𝐿Litalic_L-weak compactness inclusion criteria for bounded exponents in [9] is extended to the general case. The useful limit condition (3) has not been considered earlier. In particular a new weak compactness criterion for inclusions Lp()(μ)L1(μ)superscript𝐿𝑝𝜇superscript𝐿1𝜇{L^{p(\cdot)}(\mu)}\hookrightarrow L^{1}(\mu)italic_L start_POSTSUPERSCRIPT italic_p ( ⋅ ) end_POSTSUPERSCRIPT ( italic_μ ) ↪ italic_L start_POSTSUPERSCRIPT 1 end_POSTSUPERSCRIPT ( italic_μ ) is given (Corollary 4.2). The strict singularity of inclusions  L(μ)Lp()(μ)superscript𝐿𝜇superscript𝐿𝑝𝜇L^{\infty}(\mu)\hookrightarrow{L^{p(\cdot)}(\mu)}italic_L start_POSTSUPERSCRIPT ∞ end_POSTSUPERSCRIPT ( italic_μ ) ↪ italic_L start_POSTSUPERSCRIPT italic_p ( ⋅ ) end_POSTSUPERSCRIPT ( italic_μ )  is also studied obtaining the following criterion

limxμ(Ω)(μ(Ω)x)(1p)(x)= 0,subscript𝑥𝜇superscriptΩsuperscript𝜇Ω𝑥superscript1𝑝𝑥 0\lim_{x\rightarrow\mu(\Omega)^{-}}(\mu(\Omega)-x)^{(\frac{1}{p})^{*}(x)}=\,0,roman_lim start_POSTSUBSCRIPT italic_x → italic_μ ( roman_Ω ) start_POSTSUPERSCRIPT - end_POSTSUPERSCRIPT end_POSTSUBSCRIPT ( italic_μ ( roman_Ω ) - italic_x ) start_POSTSUPERSCRIPT ( divide start_ARG 1 end_ARG start_ARG italic_p end_ARG ) start_POSTSUPERSCRIPT ∗ end_POSTSUPERSCRIPT ( italic_x ) end_POSTSUPERSCRIPT = 0 ,

which is equivalent to  0μ(Ω)ap(x)𝑑x<superscriptsubscript0𝜇Ωsuperscript𝑎superscript𝑝𝑥differential-d𝑥\int_{0}^{\mu(\Omega)}a^{p^{*}(x)}dx<\infty∫ start_POSTSUBSCRIPT 0 end_POSTSUBSCRIPT start_POSTSUPERSCRIPT italic_μ ( roman_Ω ) end_POSTSUPERSCRIPT italic_a start_POSTSUPERSCRIPT italic_p start_POSTSUPERSCRIPT ∗ end_POSTSUPERSCRIPT ( italic_x ) end_POSTSUPERSCRIPT italic_d italic_x < ∞  for every a>1𝑎1a>1italic_a > 1. In other words, the exponent  p()𝑝p(\cdot)italic_p ( ⋅ )  must belong to the Orlicz space  L0expx(μ)superscriptsubscript𝐿0𝑒𝑥𝑝𝑥𝜇L_{0}^{exp\,x}(\mu)italic_L start_POSTSUBSCRIPT 0 end_POSTSUBSCRIPT start_POSTSUPERSCRIPT italic_e italic_x italic_p italic_x end_POSTSUPERSCRIPT ( italic_μ ) (Theorem 4.6). Several illustrative examples are included at the end of this section (Examples at 4.9).

In Section 5, the special exponent class of log-Holder continuous functions is considered, giving a simpler disjoint strict singularity criterion, namely

essinf(p()q())>0.𝑒𝑠𝑠infimum𝑝𝑞0ess\inf(p(\cdot)-q(\cdot))>0\,.italic_e italic_s italic_s roman_inf ( italic_p ( ⋅ ) - italic_q ( ⋅ ) ) > 0 .

Finally, Section 6 is devoted to the infinite measure case. Inclusions   Lp()(μ)Lq()(μ)superscript𝐿𝑝𝜇superscript𝐿𝑞𝜇{L^{p(\cdot)}(\mu)}\hookrightarrow{L^{q(\cdot)}(\mu)}italic_L start_POSTSUPERSCRIPT italic_p ( ⋅ ) end_POSTSUPERSCRIPT ( italic_μ ) ↪ italic_L start_POSTSUPERSCRIPT italic_q ( ⋅ ) end_POSTSUPERSCRIPT ( italic_μ )   for infinite measures  forces that the exponents have a very close asymptotic behavior. This allows to find suitable subspaces generated by disjoint functions with equivalence of norms. Thus for infinite measures all the inclusions  Lp()(μ)Lq()(μ)superscript𝐿𝑝𝜇superscript𝐿𝑞𝜇{L^{p(\cdot)}(\mu)}\hookrightarrow{L^{q(\cdot)}(\mu)}italic_L start_POSTSUPERSCRIPT italic_p ( ⋅ ) end_POSTSUPERSCRIPT ( italic_μ ) ↪ italic_L start_POSTSUPERSCRIPT italic_q ( ⋅ ) end_POSTSUPERSCRIPT ( italic_μ )  are no disjointly strictly singular (Theorem 6.3).

2. Preliminaries

We recall here some basic definitions and fix the notation used in the following sections.

An operator   T:EY:𝑇𝐸𝑌T:E\rightarrow Yitalic_T : italic_E → italic_Y between a Banach lattice E𝐸Eitalic_E and a Banach space F𝐹Fitalic_F is disjointly strictly singular (DSS in short) if the restriction T|[fn]evaluated-at𝑇delimited-[]subscript𝑓𝑛T|_{[f_{n}]}italic_T | start_POSTSUBSCRIPT [ italic_f start_POSTSUBSCRIPT italic_n end_POSTSUBSCRIPT ] end_POSTSUBSCRIPT  is not an isomorphism for any (closed) subspace [fn]delimited-[]subscript𝑓𝑛[f_{n}][ italic_f start_POSTSUBSCRIPT italic_n end_POSTSUBSCRIPT ]  spanned by a normalized pairwise disjoint sequence (fn)subscript𝑓𝑛(f_{n})( italic_f start_POSTSUBSCRIPT italic_n end_POSTSUBSCRIPT ) in E𝐸Eitalic_E. This DSS notion is useful in comparing the lattice structure of Banach lattices and studying strictly singular operators between Banach lattices (cf. [20, 12]). Recall that an operator T𝑇Titalic_T between two Banach spaces E𝐸Eitalic_E and F𝐹Fitalic_F is strictly singular (or Kato) if there is no infinite-dimensional subspace E1subscript𝐸1E_{1}italic_E start_POSTSUBSCRIPT 1 end_POSTSUBSCRIPT of E𝐸Eitalic_E such that the restriction T|E1T_{|E_{1}}italic_T start_POSTSUBSCRIPT | italic_E start_POSTSUBSCRIPT 1 end_POSTSUBSCRIPT end_POSTSUBSCRIPT is an isomorphism. Obviously every strictly singular operator is DSS but the converse is not true. (f.i. the inclusions  Lp[0,1]Lq[0,1]superscript𝐿𝑝01superscript𝐿𝑞01L^{p}[0,1]\hookrightarrow L^{q}[0,1]italic_L start_POSTSUPERSCRIPT italic_p end_POSTSUPERSCRIPT [ 0 , 1 ] ↪ italic_L start_POSTSUPERSCRIPT italic_q end_POSTSUPERSCRIPT [ 0 , 1 ], q<p<𝑞𝑝q<p<\inftyitalic_q < italic_p < ∞).

An operator  T:EF:𝑇𝐸𝐹T:E\rightarrow Fitalic_T : italic_E → italic_F between a Banach function lattice  E𝐸Eitalic_E and a Banach space F𝐹Fitalic_F is said to be M𝑀Mitalic_M-weakly compact whenever  limnT(fn)F=0subscript𝑛subscriptnorm𝑇subscript𝑓𝑛𝐹0\lim_{n\rightarrow\infty}\,||T(f_{n})||_{F}=0roman_lim start_POSTSUBSCRIPT italic_n → ∞ end_POSTSUBSCRIPT | | italic_T ( italic_f start_POSTSUBSCRIPT italic_n end_POSTSUBSCRIPT ) | | start_POSTSUBSCRIPT italic_F end_POSTSUBSCRIPT = 0, where (fn)subscript𝑓𝑛(f_{n})( italic_f start_POSTSUBSCRIPT italic_n end_POSTSUBSCRIPT ) is any norm bounded disjoint sequence in  E𝐸Eitalic_E. It is clear that every M𝑀Mitalic_M-weakly compact operator is a DSS operator. An operator  T:EF:𝑇𝐸𝐹T:E\rightarrow Fitalic_T : italic_E → italic_F between two Banach function lattices E𝐸Eitalic_E and F𝐹Fitalic_F on a measure space (Ω,μ)Ω𝜇(\Omega,\mu)( roman_Ω , italic_μ ) is L𝐿Litalic_L-weakly compact (or almost compact or strict) whenever T(BE)𝑇subscript𝐵𝐸T(B_{E})italic_T ( italic_B start_POSTSUBSCRIPT italic_E end_POSTSUBSCRIPT ) is a equi-integrable subset in  F𝐹Fitalic_F  for BEsubscript𝐵𝐸B_{E}italic_B start_POSTSUBSCRIPT italic_E end_POSTSUBSCRIPT  the unit ball of E𝐸Eitalic_E  i.e.

limnsupfBE{T(f)χAnF}=0subscript𝑛subscriptsupremum𝑓subscript𝐵𝐸subscriptnorm𝑇𝑓subscript𝜒subscript𝐴𝑛𝐹0\lim_{n\rightarrow\infty}\sup_{f\in B_{E}}\{||T(f)\chi_{A_{n}}||_{F}\}=0\,roman_lim start_POSTSUBSCRIPT italic_n → ∞ end_POSTSUBSCRIPT roman_sup start_POSTSUBSCRIPT italic_f ∈ italic_B start_POSTSUBSCRIPT italic_E end_POSTSUBSCRIPT end_POSTSUBSCRIPT { | | italic_T ( italic_f ) italic_χ start_POSTSUBSCRIPT italic_A start_POSTSUBSCRIPT italic_n end_POSTSUBSCRIPT end_POSTSUBSCRIPT | | start_POSTSUBSCRIPT italic_F end_POSTSUBSCRIPT } = 0

for every sequence  (An)subscript𝐴𝑛(A_{n})( italic_A start_POSTSUBSCRIPT italic_n end_POSTSUBSCRIPT )  of measurable sets in ΩΩ\Omegaroman_Ω such that  χAn0subscript𝜒subscript𝐴𝑛0\chi_{A_{n}}\rightarrow 0italic_χ start_POSTSUBSCRIPT italic_A start_POSTSUBSCRIPT italic_n end_POSTSUBSCRIPT end_POSTSUBSCRIPT → 0 μ𝜇\muitalic_μ-a.e. (cf. [2, 6]).

Let (Ω,μ)Ω𝜇(\Omega,\mu)( roman_Ω , italic_μ ) be a measure space. Given an exponent function  p()𝑝p(\cdot)italic_p ( ⋅ )  on ΩΩ\Omegaroman_Ω (i.e. a real measurable function p𝑝pitalic_p on ΩΩ\Omegaroman_Ω with 1p(t)<1𝑝𝑡1\leq p(t)<\infty1 ≤ italic_p ( italic_t ) < ∞) the variable Lebesgue space (or Nakano space)Lp()(μ)superscript𝐿𝑝𝜇{L^{p(\cdot)}(\mu)}italic_L start_POSTSUPERSCRIPT italic_p ( ⋅ ) end_POSTSUPERSCRIPT ( italic_μ )  is the space of all real measurable function classes f𝑓fitalic_f on ΩΩ\Omegaroman_Ω  such that the modular  ρp()(f/r)<subscript𝜌𝑝𝑓𝑟\rho_{p(\cdot)}(f/r)<\inftyitalic_ρ start_POSTSUBSCRIPT italic_p ( ⋅ ) end_POSTSUBSCRIPT ( italic_f / italic_r ) < ∞ for some  r>0𝑟0r>0italic_r > 0, where

ρp()(f)=Ω|f(t)|p(t)𝑑μ.subscript𝜌𝑝𝑓subscriptΩsuperscript𝑓𝑡𝑝𝑡differential-d𝜇\rho_{p(\cdot)}(f)=\int_{\Omega}|f(t)|^{p(t)}d\mu.italic_ρ start_POSTSUBSCRIPT italic_p ( ⋅ ) end_POSTSUBSCRIPT ( italic_f ) = ∫ start_POSTSUBSCRIPT roman_Ω end_POSTSUBSCRIPT | italic_f ( italic_t ) | start_POSTSUPERSCRIPT italic_p ( italic_t ) end_POSTSUPERSCRIPT italic_d italic_μ .

The associated Luxemburg norm is defined by

fp()=inf{r>0:ρp()(fr)1}.subscriptnorm𝑓𝑝infimumconditional-set𝑟0subscript𝜌𝑝𝑓𝑟1||f||_{p(\cdot)}=\inf\{r>0:\rho_{p(\cdot)}\left(\frac{f}{r}\right)\leq 1\}.| | italic_f | | start_POSTSUBSCRIPT italic_p ( ⋅ ) end_POSTSUBSCRIPT = roman_inf { italic_r > 0 : italic_ρ start_POSTSUBSCRIPT italic_p ( ⋅ ) end_POSTSUBSCRIPT ( divide start_ARG italic_f end_ARG start_ARG italic_r end_ARG ) ≤ 1 } .

We denote   p:=essinf{p(t):tΩ}assignsuperscript𝑝essinfimumconditional-set𝑝𝑡𝑡Ωp^{-}:=\text{ess}\inf\{p(t):t\in\Omega\}italic_p start_POSTSUPERSCRIPT - end_POSTSUPERSCRIPT := ess roman_inf { italic_p ( italic_t ) : italic_t ∈ roman_Ω }  and p+:=esssup{p(t):tΩ}assignsuperscript𝑝esssupremumconditional-set𝑝𝑡𝑡Ωp^{+}:=\text{ess}\sup\{p(t):t\in\Omega\}italic_p start_POSTSUPERSCRIPT + end_POSTSUPERSCRIPT := ess roman_sup { italic_p ( italic_t ) : italic_t ∈ roman_Ω }. Equally, p|A+p_{|A}^{+}italic_p start_POSTSUBSCRIPT | italic_A end_POSTSUBSCRIPT start_POSTSUPERSCRIPT + end_POSTSUPERSCRIPT and p|Ap_{|A}^{-}italic_p start_POSTSUBSCRIPT | italic_A end_POSTSUBSCRIPT start_POSTSUPERSCRIPT - end_POSTSUPERSCRIPT denote the essential supremum and infimum of  p()𝑝p(\cdot)italic_p ( ⋅ )  over a measurable subset A𝐴Aitalic_A of ΩΩ\Omegaroman_Ω. When Ω=Ω\Omega=\mathbb{N}roman_Ω = blackboard_N with the counting measure and (pn)subscript𝑝𝑛(p_{n})( italic_p start_POSTSUBSCRIPT italic_n end_POSTSUBSCRIPT ) is a real sequence with 1pn<1subscript𝑝𝑛1\leq p_{n}<\infty1 ≤ italic_p start_POSTSUBSCRIPT italic_n end_POSTSUBSCRIPT < ∞, we get the Nakano sequence space (pn)subscriptsubscript𝑝𝑛\ell_{(p_{n})}roman_ℓ start_POSTSUBSCRIPT ( italic_p start_POSTSUBSCRIPT italic_n end_POSTSUBSCRIPT ) end_POSTSUBSCRIPT i.e. the Banach space

(pn)={(xn):ρ((xn))=n=1|xnr|pn< for some r>0}subscriptsubscript𝑝𝑛conditional-setsubscript𝑥𝑛superscript𝜌subscript𝑥𝑛superscriptsubscript𝑛1superscriptsubscript𝑥𝑛𝑟subscript𝑝𝑛expectation for some 𝑟0\ell_{(p_{n})}=\left\{(x_{n})\in\mathbb{R}^{\mathbb{N}}:\rho((x_{n}))=\sum_{n=% 1}^{\infty}\left|\frac{x_{n}}{r}\right|^{p_{n}}\,<\infty\mbox{ for some }r>0\right\}roman_ℓ start_POSTSUBSCRIPT ( italic_p start_POSTSUBSCRIPT italic_n end_POSTSUBSCRIPT ) end_POSTSUBSCRIPT = { ( italic_x start_POSTSUBSCRIPT italic_n end_POSTSUBSCRIPT ) ∈ blackboard_R start_POSTSUPERSCRIPT blackboard_N end_POSTSUPERSCRIPT : italic_ρ ( ( italic_x start_POSTSUBSCRIPT italic_n end_POSTSUBSCRIPT ) ) = ∑ start_POSTSUBSCRIPT italic_n = 1 end_POSTSUBSCRIPT start_POSTSUPERSCRIPT ∞ end_POSTSUPERSCRIPT | divide start_ARG italic_x start_POSTSUBSCRIPT italic_n end_POSTSUBSCRIPT end_ARG start_ARG italic_r end_ARG | start_POSTSUPERSCRIPT italic_p start_POSTSUBSCRIPT italic_n end_POSTSUBSCRIPT end_POSTSUPERSCRIPT < ∞ for some italic_r > 0 }

equipped with the corresponding Luxemburg norm.

The conjugate exponent function  p()superscript𝑝p^{\prime}(\cdot)italic_p start_POSTSUPERSCRIPT ′ end_POSTSUPERSCRIPT ( ⋅ )  of p()𝑝p(\cdot)italic_p ( ⋅ ) is defined by the equation 1p(t)+1p(t)=11𝑝𝑡1superscript𝑝𝑡1\frac{1}{p(t)}+\frac{1}{p^{\prime}(t)}=1divide start_ARG 1 end_ARG start_ARG italic_p ( italic_t ) end_ARG + divide start_ARG 1 end_ARG start_ARG italic_p start_POSTSUPERSCRIPT ′ end_POSTSUPERSCRIPT ( italic_t ) end_ARG = 1 almost everywhere tΩ𝑡Ωt\in\Omegaitalic_t ∈ roman_Ω. When p+<superscript𝑝p^{+}<\inftyitalic_p start_POSTSUPERSCRIPT + end_POSTSUPERSCRIPT < ∞, the topological dual of the space Lp()(μ)superscript𝐿𝑝𝜇{L^{p(\cdot)}(\mu)}italic_L start_POSTSUPERSCRIPT italic_p ( ⋅ ) end_POSTSUPERSCRIPT ( italic_μ )  is the variable Lebesgue space Lp()(Ω)superscript𝐿superscript𝑝ΩL^{p^{\prime}(\cdot)}(\Omega)italic_L start_POSTSUPERSCRIPT italic_p start_POSTSUPERSCRIPT ′ end_POSTSUPERSCRIPT ( ⋅ ) end_POSTSUPERSCRIPT ( roman_Ω ). An space Lp()(μ)superscript𝐿𝑝𝜇{L^{p(\cdot)}(\mu)}italic_L start_POSTSUPERSCRIPT italic_p ( ⋅ ) end_POSTSUPERSCRIPT ( italic_μ ) is separable if and only if the measure space (Ω,μ)Ω𝜇(\Omega,\mu)( roman_Ω , italic_μ ) is separable and p+<superscript𝑝p^{+}<\inftyitalic_p start_POSTSUPERSCRIPT + end_POSTSUPERSCRIPT < ∞. Moreover, Lp()(μ)superscript𝐿𝑝𝜇{L^{p(\cdot)}(\mu)}italic_L start_POSTSUPERSCRIPT italic_p ( ⋅ ) end_POSTSUPERSCRIPT ( italic_μ ) is reflexive if and only if  1<pp+<1superscript𝑝superscript𝑝1<p^{-}\leq p^{+}<\infty1 < italic_p start_POSTSUPERSCRIPT - end_POSTSUPERSCRIPT ≤ italic_p start_POSTSUPERSCRIPT + end_POSTSUPERSCRIPT < ∞.

Recall that the associated space  (Lp()(μ))superscriptsuperscript𝐿𝑝𝜇({L^{p(\cdot)}(\mu)})^{\prime}( italic_L start_POSTSUPERSCRIPT italic_p ( ⋅ ) end_POSTSUPERSCRIPT ( italic_μ ) ) start_POSTSUPERSCRIPT ′ end_POSTSUPERSCRIPT  is the space of all scalar measurable functions g𝑔gitalic_g on ΩΩ\Omegaroman_Ω such that Ωfg𝑑μ<subscriptΩ𝑓𝑔differential-d𝜇\int_{\Omega}\,fg\,d\mu<\infty∫ start_POSTSUBSCRIPT roman_Ω end_POSTSUBSCRIPT italic_f italic_g italic_d italic_μ < ∞  for every  fLp()(μ)𝑓superscript𝐿𝑝𝜇f\in{L^{p(\cdot)}(\mu)}italic_f ∈ italic_L start_POSTSUPERSCRIPT italic_p ( ⋅ ) end_POSTSUPERSCRIPT ( italic_μ ). If 1<p()<1𝑝1<p(\cdot)<\infty1 < italic_p ( ⋅ ) < ∞ a.e. then (Lp()(μ))=Lp()(μ)superscriptsuperscript𝐿𝑝𝜇superscript𝐿superscript𝑝𝜇(L^{p(\cdot)}(\mu))^{\prime}=L^{p^{\prime}(\cdot)}(\mu)( italic_L start_POSTSUPERSCRIPT italic_p ( ⋅ ) end_POSTSUPERSCRIPT ( italic_μ ) ) start_POSTSUPERSCRIPT ′ end_POSTSUPERSCRIPT = italic_L start_POSTSUPERSCRIPT italic_p start_POSTSUPERSCRIPT ′ end_POSTSUPERSCRIPT ( ⋅ ) end_POSTSUPERSCRIPT ( italic_μ )   (cf. [7, 28]).

A sequence (fn)Lp()(μ)subscript𝑓𝑛superscript𝐿𝑝𝜇(f_{n})\subset{L^{p(\cdot)}(\mu)}( italic_f start_POSTSUBSCRIPT italic_n end_POSTSUBSCRIPT ) ⊂ italic_L start_POSTSUPERSCRIPT italic_p ( ⋅ ) end_POSTSUPERSCRIPT ( italic_μ ) verifies that fnp()0subscriptnormsubscript𝑓𝑛𝑝0||f_{n}||_{p(\cdot)}\rightarrow 0| | italic_f start_POSTSUBSCRIPT italic_n end_POSTSUBSCRIPT | | start_POSTSUBSCRIPT italic_p ( ⋅ ) end_POSTSUBSCRIPT → 0 if and only if ρp()(λfn)0subscript𝜌𝑝𝜆subscript𝑓𝑛0\rho_{p(\cdot)}(\lambda f_{n})\rightarrow 0italic_ρ start_POSTSUBSCRIPT italic_p ( ⋅ ) end_POSTSUBSCRIPT ( italic_λ italic_f start_POSTSUBSCRIPT italic_n end_POSTSUBSCRIPT ) → 0 for every λ>0𝜆0\lambda>0italic_λ > 0. If p+<superscript𝑝p^{+}<\inftyitalic_p start_POSTSUPERSCRIPT + end_POSTSUPERSCRIPT < ∞, then ρp()(fn)0subscript𝜌𝑝subscript𝑓𝑛0\rho_{p(\cdot)}(f_{n})\rightarrow 0italic_ρ start_POSTSUBSCRIPT italic_p ( ⋅ ) end_POSTSUBSCRIPT ( italic_f start_POSTSUBSCRIPT italic_n end_POSTSUBSCRIPT ) → 0 if and only if fnp()0subscriptnormsubscript𝑓𝑛𝑝0||f_{n}||_{p(\cdot)}\rightarrow 0| | italic_f start_POSTSUBSCRIPT italic_n end_POSTSUBSCRIPT | | start_POSTSUBSCRIPT italic_p ( ⋅ ) end_POSTSUBSCRIPT → 0. Furthermore,  ρp()(f)1subscript𝜌𝑝𝑓1\rho_{p(\cdot)}(f)\leq 1italic_ρ start_POSTSUBSCRIPT italic_p ( ⋅ ) end_POSTSUBSCRIPT ( italic_f ) ≤ 1 if and only if   fp()1subscriptnorm𝑓𝑝1||f||_{p(\cdot)}\leq 1| | italic_f | | start_POSTSUBSCRIPT italic_p ( ⋅ ) end_POSTSUBSCRIPT ≤ 1. Also, if fp()>1subscriptnorm𝑓𝑝1||f||_{p(\cdot)}>1| | italic_f | | start_POSTSUBSCRIPT italic_p ( ⋅ ) end_POSTSUBSCRIPT > 1, then 1fp()ρp()(f)1subscriptnorm𝑓𝑝subscript𝜌𝑝𝑓1\leq||f||_{p(\cdot)}\leq\rho_{p(\cdot)}(f)1 ≤ | | italic_f | | start_POSTSUBSCRIPT italic_p ( ⋅ ) end_POSTSUBSCRIPT ≤ italic_ρ start_POSTSUBSCRIPT italic_p ( ⋅ ) end_POSTSUBSCRIPT ( italic_f ) ([8] p.75, [7]). The Hölder inequality ([7] Thm 2.26, [8] Lemma 3.2.20) states that there exists a constant   1<K41𝐾41<K\leq 41 < italic_K ≤ 4  such that for every two measurable functions  f,g:Ω:𝑓𝑔Ωf,g:\Omega\rightarrow\mathbb{R}italic_f , italic_g : roman_Ω → blackboard_R, it holds

Ω|f(t)g(t)|𝑑μKfp()gp().subscriptΩ𝑓𝑡𝑔𝑡differential-d𝜇𝐾subscriptnorm𝑓𝑝subscriptnorm𝑔superscript𝑝\int_{\Omega}|f(t)g(t)|d\mu\leq\,K\,||f||_{p(\cdot)}\,||g||_{p^{\prime}(\cdot)}.∫ start_POSTSUBSCRIPT roman_Ω end_POSTSUBSCRIPT | italic_f ( italic_t ) italic_g ( italic_t ) | italic_d italic_μ ≤ italic_K | | italic_f | | start_POSTSUBSCRIPT italic_p ( ⋅ ) end_POSTSUBSCRIPT | | italic_g | | start_POSTSUBSCRIPT italic_p start_POSTSUPERSCRIPT ′ end_POSTSUPERSCRIPT ( ⋅ ) end_POSTSUBSCRIPT .

A criterion for the inclusion  Lp()(μ)Lq()(μ)superscript𝐿𝑝𝜇superscript𝐿𝑞𝜇{L^{p(\cdot)}(\mu)}\hookrightarrow{L^{q(\cdot)}(\mu)}italic_L start_POSTSUPERSCRIPT italic_p ( ⋅ ) end_POSTSUPERSCRIPT ( italic_μ ) ↪ italic_L start_POSTSUPERSCRIPT italic_q ( ⋅ ) end_POSTSUPERSCRIPT ( italic_μ )  to hold is the following:

Proposition 2.1.

([7] Thm 2.45, [8] Thm 3.3.1) Let (Ω,μ)Ω𝜇(\Omega,\mu)( roman_Ω , italic_μ ) be an atomless infinite measure space and exponents p()𝑝p(\cdot)italic_p ( ⋅ ) and q()𝑞q(\cdot)italic_q ( ⋅ ). The inclusion  Lp()(μ)Lq()(μ)superscript𝐿𝑝𝜇superscript𝐿𝑞𝜇{L^{p(\cdot)}(\mu)}\hookrightarrow{L^{q(\cdot)}(\mu)}italic_L start_POSTSUPERSCRIPT italic_p ( ⋅ ) end_POSTSUPERSCRIPT ( italic_μ ) ↪ italic_L start_POSTSUPERSCRIPT italic_q ( ⋅ ) end_POSTSUPERSCRIPT ( italic_μ )  holds if and only if  q()p()𝑞𝑝q(\cdot)\leq p(\cdot)italic_q ( ⋅ ) ≤ italic_p ( ⋅ )μ𝜇\muitalic_μ-a.e. and there exists λ>1𝜆1\lambda>1italic_λ > 1 such that

Ωdλ(pqpq)(t)𝑑μ<,subscriptsubscriptΩ𝑑superscript𝜆𝑝𝑞𝑝𝑞𝑡differential-d𝜇\int_{\Omega_{d}}\lambda^{-\left(\frac{p\,q}{p-q}\right)(t)}\,d\mu<\infty,∫ start_POSTSUBSCRIPT roman_Ω start_POSTSUBSCRIPT italic_d end_POSTSUBSCRIPT end_POSTSUBSCRIPT italic_λ start_POSTSUPERSCRIPT - ( divide start_ARG italic_p italic_q end_ARG start_ARG italic_p - italic_q end_ARG ) ( italic_t ) end_POSTSUPERSCRIPT italic_d italic_μ < ∞ ,

where  Ωd={tΩ:p(t)>q(t)}subscriptΩ𝑑conditional-set𝑡Ω𝑝𝑡𝑞𝑡\Omega_{d}=\{t\in\Omega:p(t)>q(t)\}roman_Ω start_POSTSUBSCRIPT italic_d end_POSTSUBSCRIPT = { italic_t ∈ roman_Ω : italic_p ( italic_t ) > italic_q ( italic_t ) }.

Note that, in contrast with classical Lebesgue spaces  Lpsuperscript𝐿𝑝L^{p}italic_L start_POSTSUPERSCRIPT italic_p end_POSTSUPERSCRIPT, inclusions between variable Lebesgue spaces on infinite measures  Lp()(μ)Lq()(μ)superscript𝐿𝑝𝜇superscript𝐿𝑞𝜇{L^{p(\cdot)}(\mu)}\hookrightarrow{L^{q(\cdot)}(\mu)}italic_L start_POSTSUPERSCRIPT italic_p ( ⋅ ) end_POSTSUPERSCRIPT ( italic_μ ) ↪ italic_L start_POSTSUPERSCRIPT italic_q ( ⋅ ) end_POSTSUPERSCRIPT ( italic_μ )  can hold. Also, for a finite measure space (Ω,μ)Ω𝜇(\Omega,\mu)( roman_Ω , italic_μ ), the inclusion  Lp()(μ)Lq()(μ)superscript𝐿𝑝𝜇superscript𝐿𝑞𝜇{L^{p(\cdot)}(\mu)}\hookrightarrow{L^{q(\cdot)}(\mu)}italic_L start_POSTSUPERSCRIPT italic_p ( ⋅ ) end_POSTSUPERSCRIPT ( italic_μ ) ↪ italic_L start_POSTSUPERSCRIPT italic_q ( ⋅ ) end_POSTSUPERSCRIPT ( italic_μ )  holds if and only if  q()p()𝑞𝑝q(\cdot)\leq p(\cdot)italic_q ( ⋅ ) ≤ italic_p ( ⋅ )μ𝜇\muitalic_μ-a.e..

If (fn)subscript𝑓𝑛(f_{n})( italic_f start_POSTSUBSCRIPT italic_n end_POSTSUBSCRIPT ) is a disjoint sequence in Lp()(μ)superscript𝐿𝑝𝜇{L^{p(\cdot)}(\mu)}italic_L start_POSTSUPERSCRIPT italic_p ( ⋅ ) end_POSTSUPERSCRIPT ( italic_μ ) and (gn)subscript𝑔𝑛(g_{n})( italic_g start_POSTSUBSCRIPT italic_n end_POSTSUBSCRIPT ) is another sequence such that fngnp()<subscriptnormsubscript𝑓𝑛subscript𝑔𝑛𝑝\sum||f_{n}-g_{n}||_{p(\cdot)}<\infty∑ | | italic_f start_POSTSUBSCRIPT italic_n end_POSTSUBSCRIPT - italic_g start_POSTSUBSCRIPT italic_n end_POSTSUBSCRIPT | | start_POSTSUBSCRIPT italic_p ( ⋅ ) end_POSTSUBSCRIPT < ∞ then (fn)subscript𝑓𝑛(f_{n})( italic_f start_POSTSUBSCRIPT italic_n end_POSTSUBSCRIPT ) and (gn)subscript𝑔𝑛(g_{n})( italic_g start_POSTSUBSCRIPT italic_n end_POSTSUBSCRIPT ) are equivalent (unconditional) basic sequences, i.e. nxnfnLp()(μ)subscript𝑛subscript𝑥𝑛subscript𝑓𝑛superscript𝐿𝑝𝜇\sum_{n}x_{n}f_{n}\in{L^{p(\cdot)}(\mu)}∑ start_POSTSUBSCRIPT italic_n end_POSTSUBSCRIPT italic_x start_POSTSUBSCRIPT italic_n end_POSTSUBSCRIPT italic_f start_POSTSUBSCRIPT italic_n end_POSTSUBSCRIPT ∈ italic_L start_POSTSUPERSCRIPT italic_p ( ⋅ ) end_POSTSUPERSCRIPT ( italic_μ ) if and only if nxngnLp()(μ).subscript𝑛subscript𝑥𝑛subscript𝑔𝑛superscript𝐿𝑝𝜇\sum_{n}x_{n}g_{n}\in{L^{p(\cdot)}(\mu)}.∑ start_POSTSUBSCRIPT italic_n end_POSTSUBSCRIPT italic_x start_POSTSUBSCRIPT italic_n end_POSTSUBSCRIPT italic_g start_POSTSUBSCRIPT italic_n end_POSTSUBSCRIPT ∈ italic_L start_POSTSUPERSCRIPT italic_p ( ⋅ ) end_POSTSUPERSCRIPT ( italic_μ ) .

Proposition 2.2.

([30]) Let 1pn,qn<formulae-sequence1subscript𝑝𝑛subscript𝑞𝑛1\leq p_{n},\,q_{n}<\infty1 ≤ italic_p start_POSTSUBSCRIPT italic_n end_POSTSUBSCRIPT , italic_q start_POSTSUBSCRIPT italic_n end_POSTSUBSCRIPT < ∞. Then  (pn)=(qn)subscriptsubscript𝑝𝑛subscriptsubscript𝑞𝑛\ell_{(p_{n})}=\ell_{(q_{n})}roman_ℓ start_POSTSUBSCRIPT ( italic_p start_POSTSUBSCRIPT italic_n end_POSTSUBSCRIPT ) end_POSTSUBSCRIPT = roman_ℓ start_POSTSUBSCRIPT ( italic_q start_POSTSUBSCRIPT italic_n end_POSTSUBSCRIPT ) end_POSTSUBSCRIPT  if and only if there exists  α>0𝛼0\alpha>0italic_α > 0  such that  n=1αpnqn|pnqn|<superscriptsubscript𝑛1superscript𝛼subscript𝑝𝑛subscript𝑞𝑛subscript𝑝𝑛subscript𝑞𝑛\sum_{n=1}^{\infty}\,\alpha^{\frac{p_{n}q_{n}}{|p_{n}-q_{n}|}}\,<\infty∑ start_POSTSUBSCRIPT italic_n = 1 end_POSTSUBSCRIPT start_POSTSUPERSCRIPT ∞ end_POSTSUPERSCRIPT italic_α start_POSTSUPERSCRIPT divide start_ARG italic_p start_POSTSUBSCRIPT italic_n end_POSTSUBSCRIPT italic_q start_POSTSUBSCRIPT italic_n end_POSTSUBSCRIPT end_ARG start_ARG | italic_p start_POSTSUBSCRIPT italic_n end_POSTSUBSCRIPT - italic_q start_POSTSUBSCRIPT italic_n end_POSTSUBSCRIPT | end_ARG end_POSTSUPERSCRIPT < ∞.

Recall that the decreasing rearrangement (cf. [6, 25, 27]) of a measurable function f𝑓fitalic_f is the real function fsuperscript𝑓f^{*}italic_f start_POSTSUPERSCRIPT ∗ end_POSTSUPERSCRIPT on [0,μ(Ω))0𝜇Ω[0,\mu(\Omega))[ 0 , italic_μ ( roman_Ω ) ) defined by

f(x):=inf{s[0,μ(Ω)]:μf(s)x},assignsuperscript𝑓𝑥infimumconditional-set𝑠0𝜇Ωsubscript𝜇𝑓𝑠𝑥f^{*}(x):=\inf\{s\in[0,\mu(\Omega)]\,:\,\mu_{f}(s)\leq x\},italic_f start_POSTSUPERSCRIPT ∗ end_POSTSUPERSCRIPT ( italic_x ) := roman_inf { italic_s ∈ [ 0 , italic_μ ( roman_Ω ) ] : italic_μ start_POSTSUBSCRIPT italic_f end_POSTSUBSCRIPT ( italic_s ) ≤ italic_x } ,

where μfsubscript𝜇𝑓\mu_{f}italic_μ start_POSTSUBSCRIPT italic_f end_POSTSUBSCRIPT is the distribution function of f𝑓fitalic_f, μf(s):=μ({tΩ:|f(t)|>s})assignsubscript𝜇𝑓𝑠𝜇conditional-set𝑡Ω𝑓𝑡𝑠\mu_{f}(s):=\mu(\{t\in\Omega:|f(t)|>s\})italic_μ start_POSTSUBSCRIPT italic_f end_POSTSUBSCRIPT ( italic_s ) := italic_μ ( { italic_t ∈ roman_Ω : | italic_f ( italic_t ) | > italic_s } ). For a measurable f0𝑓0f\geq 0italic_f ≥ 0 on ΩΩ\Omegaroman_Ω, the functions f𝑓fitalic_f and fsuperscript𝑓f^{*}italic_f start_POSTSUPERSCRIPT ∗ end_POSTSUPERSCRIPT are equi-distributed and

Ωf(t)𝑑μ=0μ(Ω)f(x)𝑑x.subscriptΩ𝑓𝑡differential-d𝜇superscriptsubscript0𝜇Ωsuperscript𝑓𝑥differential-d𝑥\int_{\Omega}f(t)d\mu\,=\,\int_{0}^{\mu(\Omega)}f^{*}(x)dx.∫ start_POSTSUBSCRIPT roman_Ω end_POSTSUBSCRIPT italic_f ( italic_t ) italic_d italic_μ = ∫ start_POSTSUBSCRIPT 0 end_POSTSUBSCRIPT start_POSTSUPERSCRIPT italic_μ ( roman_Ω ) end_POSTSUPERSCRIPT italic_f start_POSTSUPERSCRIPT ∗ end_POSTSUPERSCRIPT ( italic_x ) italic_d italic_x .

A Banach function lattice is said to be rearrangement invariant if every two equi-distributed functions have the same norm. Orlicz spaces (cf. [24, 29]) are examples of rearrangement invariant spaces while variable Lebesgue spaces are not. If φ𝜑\varphiitalic_φ is a non-decreasing unbounded positive convex function on [0,)0[0,\infty)[ 0 , ∞ ) with φ(0)=0𝜑00\varphi(0)=0italic_φ ( 0 ) = 0, the Orlicz spaceLφ(μ)superscript𝐿𝜑𝜇L^{\varphi}(\mu)italic_L start_POSTSUPERSCRIPT italic_φ end_POSTSUPERSCRIPT ( italic_μ )  consists of all measurable functions classes f𝑓fitalic_f on (Ω,μ)Ω𝜇(\Omega,\mu)( roman_Ω , italic_μ ) such that for some r>0𝑟0r>0italic_r > 0

Ωφ(r|f|)𝑑μ<.subscriptΩ𝜑𝑟𝑓differential-d𝜇\int_{\Omega}\varphi(r|f|)d\mu<\infty.∫ start_POSTSUBSCRIPT roman_Ω end_POSTSUBSCRIPT italic_φ ( italic_r | italic_f | ) italic_d italic_μ < ∞ .

In the class of Orlicz spaces there are examples of inclusions  Lφ(μ)Lψ(μ)superscript𝐿𝜑𝜇superscript𝐿𝜓𝜇L^{\varphi}(\mu)\hookrightarrow L^{\psi}(\mu)italic_L start_POSTSUPERSCRIPT italic_φ end_POSTSUPERSCRIPT ( italic_μ ) ↪ italic_L start_POSTSUPERSCRIPT italic_ψ end_POSTSUPERSCRIPT ( italic_μ )  for a finite measure which are DSS but not L𝐿Litalic_L-weakly compact. Let us recall the DSS criterion and the L𝐿Litalic_L-weak compactness criterion for inclusions between Orlicz spaces:

Proposition 2.3.

([20] Prop. 3.2). Let (Ω,μ)Ω𝜇(\Omega,\mu)( roman_Ω , italic_μ ) be an atomless finite measure space and  ψφ𝜓𝜑\psi\leq\varphiitalic_ψ ≤ italic_φ  Orlicz functions with the Δ2subscriptΔ2\Delta_{2}roman_Δ start_POSTSUBSCRIPT 2 end_POSTSUBSCRIPT-condition. An inclusion  Lφ(μ)Lψ(μ)superscript𝐿𝜑𝜇superscript𝐿𝜓𝜇L^{\varphi}(\mu)\hookrightarrow L^{\psi}(\mu)italic_L start_POSTSUPERSCRIPT italic_φ end_POSTSUPERSCRIPT ( italic_μ ) ↪ italic_L start_POSTSUPERSCRIPT italic_ψ end_POSTSUPERSCRIPT ( italic_μ )   is DSS   if and only if for every natural n𝑛nitalic_n and any constant A>0𝐴0A>0italic_A > 0 there exist  1x1<x2<<xn1subscript𝑥1subscript𝑥2subscript𝑥𝑛1\leq x_{1}<x_{2}<...<x_{n}1 ≤ italic_x start_POSTSUBSCRIPT 1 end_POSTSUBSCRIPT < italic_x start_POSTSUBSCRIPT 2 end_POSTSUBSCRIPT < … < italic_x start_POSTSUBSCRIPT italic_n end_POSTSUBSCRIPT  and ci>0subscript𝑐𝑖0c_{i}>0italic_c start_POSTSUBSCRIPT italic_i end_POSTSUBSCRIPT > 0 for i=1,,n𝑖1𝑛i=1,...,nitalic_i = 1 , … , italic_n such that for t1𝑡1t\geq 1italic_t ≥ 1

i=1nciψ(txi)Ai=1nciφ(txi)superscriptsubscript𝑖1𝑛subscript𝑐𝑖𝜓𝑡subscript𝑥𝑖𝐴superscriptsubscript𝑖1𝑛subscript𝑐𝑖𝜑𝑡subscript𝑥𝑖\sum_{i=1}^{n}c_{i}\,\psi(tx_{i})\,\leq\,A\,\sum_{i=1}^{n}\,c_{i}\,\varphi(tx_% {i})∑ start_POSTSUBSCRIPT italic_i = 1 end_POSTSUBSCRIPT start_POSTSUPERSCRIPT italic_n end_POSTSUPERSCRIPT italic_c start_POSTSUBSCRIPT italic_i end_POSTSUBSCRIPT italic_ψ ( italic_t italic_x start_POSTSUBSCRIPT italic_i end_POSTSUBSCRIPT ) ≤ italic_A ∑ start_POSTSUBSCRIPT italic_i = 1 end_POSTSUBSCRIPT start_POSTSUPERSCRIPT italic_n end_POSTSUPERSCRIPT italic_c start_POSTSUBSCRIPT italic_i end_POSTSUBSCRIPT italic_φ ( italic_t italic_x start_POSTSUBSCRIPT italic_i end_POSTSUBSCRIPT )
Proposition 2.4.

([24], [5] p.1369) Let  (Ω,μ)Ω𝜇(\Omega,\mu)( roman_Ω , italic_μ )  be an atomless finite measure space and Orlicz functions  ψφ𝜓𝜑\psi\leq\varphiitalic_ψ ≤ italic_φ. An inclusion  Lφ(μ)Lψ(μ)superscript𝐿𝜑𝜇superscript𝐿𝜓𝜇L^{\varphi}(\mu)\hookrightarrow L^{\psi}(\mu)italic_L start_POSTSUPERSCRIPT italic_φ end_POSTSUPERSCRIPT ( italic_μ ) ↪ italic_L start_POSTSUPERSCRIPT italic_ψ end_POSTSUPERSCRIPT ( italic_μ )  is  L𝐿Litalic_L-weakly compact if and only if

limtφ1(t)ψ1(t)= 0.subscriptmaps-to𝑡superscript𝜑1𝑡superscript𝜓1𝑡 0\lim_{t\mapsto\infty}\,\frac{\varphi^{-1}(t)}{\psi^{-1}(t)}\,=\,0.roman_lim start_POSTSUBSCRIPT italic_t ↦ ∞ end_POSTSUBSCRIPT divide start_ARG italic_φ start_POSTSUPERSCRIPT - 1 end_POSTSUPERSCRIPT ( italic_t ) end_ARG start_ARG italic_ψ start_POSTSUPERSCRIPT - 1 end_POSTSUPERSCRIPT ( italic_t ) end_ARG = 0 .

In particular this condition implies that   limsψ(s)φ(s)=0subscriptmaps-to𝑠𝜓𝑠𝜑𝑠0\lim_{s\mapsto\infty}\,\frac{\psi(s)}{\varphi(s)}=0roman_lim start_POSTSUBSCRIPT italic_s ↦ ∞ end_POSTSUBSCRIPT divide start_ARG italic_ψ ( italic_s ) end_ARG start_ARG italic_φ ( italic_s ) end_ARG = 0. Indeed, given 0<ϵ<10italic-ϵ10<\epsilon<10 < italic_ϵ < 1 there exists s0>0subscript𝑠00s_{0}>0italic_s start_POSTSUBSCRIPT 0 end_POSTSUBSCRIPT > 0 such that   φ1(s)ϵψ1(s)superscript𝜑1𝑠italic-ϵsuperscript𝜓1𝑠\varphi^{-1}(s)\leq\epsilon\psi^{-1}(s)italic_φ start_POSTSUPERSCRIPT - 1 end_POSTSUPERSCRIPT ( italic_s ) ≤ italic_ϵ italic_ψ start_POSTSUPERSCRIPT - 1 end_POSTSUPERSCRIPT ( italic_s )  for s>s0𝑠subscript𝑠0s>s_{0}italic_s > italic_s start_POSTSUBSCRIPT 0 end_POSTSUBSCRIPT. So, for   s=ψ(t)>s0𝑠𝜓𝑡subscript𝑠0s=\psi(t)>s_{0}italic_s = italic_ψ ( italic_t ) > italic_s start_POSTSUBSCRIPT 0 end_POSTSUBSCRIPT, we have φ1(ψ(t))ϵψ1(ψ(t))=ϵsuperscript𝜑1𝜓𝑡italic-ϵsuperscript𝜓1𝜓𝑡italic-ϵ\varphi^{-1}(\psi(t))\leq\epsilon\psi^{-1}(\psi(t))=\epsilonitalic_φ start_POSTSUPERSCRIPT - 1 end_POSTSUPERSCRIPT ( italic_ψ ( italic_t ) ) ≤ italic_ϵ italic_ψ start_POSTSUPERSCRIPT - 1 end_POSTSUPERSCRIPT ( italic_ψ ( italic_t ) ) = italic_ϵ, hence   ψ(t)φ(ϵt)ϵφ(t)𝜓𝑡𝜑italic-ϵ𝑡italic-ϵ𝜑𝑡\psi(t)\leq\varphi(\epsilon t)\,\leq\epsilon\,\varphi(t)italic_ψ ( italic_t ) ≤ italic_φ ( italic_ϵ italic_t ) ≤ italic_ϵ italic_φ ( italic_t )   for  t>φ1(s0)𝑡superscript𝜑1subscript𝑠0t>\varphi^{-1}(s_{0})italic_t > italic_φ start_POSTSUPERSCRIPT - 1 end_POSTSUPERSCRIPT ( italic_s start_POSTSUBSCRIPT 0 end_POSTSUBSCRIPT ).

Consider now the Orlicz function ψ𝜓\psiitalic_ψ defined in ([21] Thm. A) which verifies the inclusion  Lp[0,1]Lψ[0,1]superscript𝐿𝑝01superscript𝐿𝜓01L^{p}[0,1]\hookrightarrow L^{\psi}[0,1]italic_L start_POSTSUPERSCRIPT italic_p end_POSTSUPERSCRIPT [ 0 , 1 ] ↪ italic_L start_POSTSUPERSCRIPT italic_ψ end_POSTSUPERSCRIPT [ 0 , 1 ], for a fixed p>1𝑝1p>1italic_p > 1. Using the above criterion, it is proved that the inclusion   Lp[0,1]Lψ[0,1]superscript𝐿𝑝01superscript𝐿𝜓01L^{p}[0,1]\hookrightarrow L^{\psi}[0,1]italic_L start_POSTSUPERSCRIPT italic_p end_POSTSUPERSCRIPT [ 0 , 1 ] ↪ italic_L start_POSTSUPERSCRIPT italic_ψ end_POSTSUPERSCRIPT [ 0 , 1 ]   is DSS and that

lim suptψ(t)tplim supnψ(2n)2np>0subscriptlimit-supremummaps-to𝑡𝜓𝑡superscript𝑡𝑝subscriptlimit-supremummaps-to𝑛𝜓superscript2𝑛superscript2𝑛𝑝0\limsup_{t\mapsto\infty}\,\,\frac{\psi(t)}{t^{p}}\,\geq\limsup_{n\mapsto\infty% }\,\,\frac{\psi(2^{n})}{2^{np}}\,>0lim sup start_POSTSUBSCRIPT italic_t ↦ ∞ end_POSTSUBSCRIPT divide start_ARG italic_ψ ( italic_t ) end_ARG start_ARG italic_t start_POSTSUPERSCRIPT italic_p end_POSTSUPERSCRIPT end_ARG ≥ lim sup start_POSTSUBSCRIPT italic_n ↦ ∞ end_POSTSUBSCRIPT divide start_ARG italic_ψ ( 2 start_POSTSUPERSCRIPT italic_n end_POSTSUPERSCRIPT ) end_ARG start_ARG 2 start_POSTSUPERSCRIPT italic_n italic_p end_POSTSUPERSCRIPT end_ARG > 0

(see [21] p.184). So we deduce, by above L𝐿Litalic_L-weak compactness criterion, that the inclusion   Lp[0,1]Lψ[0,1]superscript𝐿𝑝01superscript𝐿𝜓01L^{p}[0,1]\hookrightarrow L^{\psi}[0,1]italic_L start_POSTSUPERSCRIPT italic_p end_POSTSUPERSCRIPT [ 0 , 1 ] ↪ italic_L start_POSTSUPERSCRIPT italic_ψ end_POSTSUPERSCRIPT [ 0 , 1 ]   is not L𝐿Litalic_L-weakly compact.

In the setting of variable Lebesgue spaces Lp()(μ)superscript𝐿𝑝𝜇{L^{p(\cdot)}(\mu)}italic_L start_POSTSUPERSCRIPT italic_p ( ⋅ ) end_POSTSUPERSCRIPT ( italic_μ ), a L𝐿Litalic_L-weakly compact inclusion criterion has been given by Edmunds, Gogathisvili and Nekvinda in ([9] Thm 3.4):

Proposition 2.5.

([9]) Let a bounded open subset  ΩnΩsuperscript𝑛\Omega\subset\mathbb{R}^{n}roman_Ω ⊂ blackboard_R start_POSTSUPERSCRIPT italic_n end_POSTSUPERSCRIPT and bounded exponents  q()p()p+<𝑞𝑝superscript𝑝q(\cdot)\leq p(\cdot)\,\leq\,p^{+}<\inftyitalic_q ( ⋅ ) ≤ italic_p ( ⋅ ) ≤ italic_p start_POSTSUPERSCRIPT + end_POSTSUPERSCRIPT < ∞. The inclusion   Lp()(Ω)Lq()(Ω)superscript𝐿𝑝Ωsuperscript𝐿𝑞ΩL^{p(\cdot)}(\Omega)\hookrightarrow L^{q(\cdot)}(\Omega)italic_L start_POSTSUPERSCRIPT italic_p ( ⋅ ) end_POSTSUPERSCRIPT ( roman_Ω ) ↪ italic_L start_POSTSUPERSCRIPT italic_q ( ⋅ ) end_POSTSUPERSCRIPT ( roman_Ω )  is L𝐿Litalic_L-weakly compact if and only if for every  a>1𝑎1a>1italic_a > 1

0|Ω|a(1pq)(x)𝑑x<.superscriptsubscript0Ωsuperscript𝑎superscript1𝑝𝑞𝑥differential-d𝑥\int_{0}^{|\Omega|}a^{\left(\frac{1}{p-q}\right)^{*}(x)}dx<\infty.∫ start_POSTSUBSCRIPT 0 end_POSTSUBSCRIPT start_POSTSUPERSCRIPT | roman_Ω | end_POSTSUPERSCRIPT italic_a start_POSTSUPERSCRIPT ( divide start_ARG 1 end_ARG start_ARG italic_p - italic_q end_ARG ) start_POSTSUPERSCRIPT ∗ end_POSTSUPERSCRIPT ( italic_x ) end_POSTSUPERSCRIPT italic_d italic_x < ∞ .

Other L𝐿Litalic_L-weak compactness inclusion characterization of   Lp()(μ)Lq()(μ)superscript𝐿𝑝𝜇superscript𝐿𝑞𝜇L^{p(\cdot)}(\mu)\hookrightarrow L^{q(\cdot)}(\mu)italic_L start_POSTSUPERSCRIPT italic_p ( ⋅ ) end_POSTSUPERSCRIPT ( italic_μ ) ↪ italic_L start_POSTSUPERSCRIPT italic_q ( ⋅ ) end_POSTSUPERSCRIPT ( italic_μ ) (De la Valleé-Poussin type) is given in [22] (Prop. 3.3), [31].

3. Previous Results

We will study inclusion operators between variable Lebesgue spaces on a finite measure space  Lp()(μ)Lq()(μ)superscript𝐿𝑝𝜇superscript𝐿𝑞𝜇{L^{p(\cdot)}(\mu)}\hookrightarrow{L^{q(\cdot)}(\mu)}italic_L start_POSTSUPERSCRIPT italic_p ( ⋅ ) end_POSTSUPERSCRIPT ( italic_μ ) ↪ italic_L start_POSTSUPERSCRIPT italic_q ( ⋅ ) end_POSTSUPERSCRIPT ( italic_μ ), looking for suitable DSS characterizations in terms of the exponents. In this section we collect some preliminary results. We will assume  q()p()𝑞𝑝q(\cdot)\leq\,p(\cdot)italic_q ( ⋅ ) ≤ italic_p ( ⋅ ) μ𝜇\muitalic_μ-a.e. with  μ{tΩ:q(t)=p(t)}=0𝜇conditional-set𝑡Ω𝑞𝑡𝑝𝑡0\mu\{t\in\Omega:\,q(t)=p(t)\}=0italic_μ { italic_t ∈ roman_Ω : italic_q ( italic_t ) = italic_p ( italic_t ) } = 0  (otherwise the inclusion Lp()(μ)Lq()(μ)superscript𝐿𝑝𝜇superscript𝐿𝑞𝜇{L^{p(\cdot)}(\mu)}\hookrightarrow{L^{q(\cdot)}(\mu)}italic_L start_POSTSUPERSCRIPT italic_p ( ⋅ ) end_POSTSUPERSCRIPT ( italic_μ ) ↪ italic_L start_POSTSUPERSCRIPT italic_q ( ⋅ ) end_POSTSUPERSCRIPT ( italic_μ )  would be trivially non-DSS). No restrictions on the exponents will be assumed.

Let r:Ω[1,):𝑟Ω1r:\Omega\rightarrow[1,\infty)italic_r : roman_Ω → [ 1 , ∞ ) be an exponent𝑒𝑥𝑝𝑜𝑛𝑒𝑛𝑡exponentitalic_e italic_x italic_p italic_o italic_n italic_e italic_n italic_t and consider the function   ar(t)superscript𝑎𝑟𝑡a^{r(t)}italic_a start_POSTSUPERSCRIPT italic_r ( italic_t ) end_POSTSUPERSCRIPT  on ΩΩ\Omegaroman_Ω for some a>1𝑎1a>1italic_a > 1. It holds that   (ar())(x)=ar(x)superscriptsuperscript𝑎𝑟𝑥superscript𝑎superscript𝑟𝑥(a^{r(\cdot)})^{*}(x)=\,a^{r^{*}(x)}( italic_a start_POSTSUPERSCRIPT italic_r ( ⋅ ) end_POSTSUPERSCRIPT ) start_POSTSUPERSCRIPT ∗ end_POSTSUPERSCRIPT ( italic_x ) = italic_a start_POSTSUPERSCRIPT italic_r start_POSTSUPERSCRIPT ∗ end_POSTSUPERSCRIPT ( italic_x ) end_POSTSUPERSCRIPT for all x>0𝑥0x>0italic_x > 0 (cf. [9] Lemma 2.10), hence

Ωar(t)𝑑μ=0μ(Ω)(ar())(x)𝑑x=0μ(Ω)ar(x)𝑑x.subscriptΩsuperscript𝑎𝑟𝑡differential-d𝜇superscriptsubscript0𝜇Ωsuperscriptsuperscript𝑎𝑟𝑥differential-d𝑥superscriptsubscript0𝜇Ωsuperscript𝑎superscript𝑟𝑥differential-d𝑥\int_{\Omega}a^{r(t)}d\mu=\int_{0}^{\mu(\Omega)}(a^{r(\cdot)})^{*}(x)\,dx=\int% _{0}^{\mu(\Omega)}a^{r^{*}(x)}dx.∫ start_POSTSUBSCRIPT roman_Ω end_POSTSUBSCRIPT italic_a start_POSTSUPERSCRIPT italic_r ( italic_t ) end_POSTSUPERSCRIPT italic_d italic_μ = ∫ start_POSTSUBSCRIPT 0 end_POSTSUBSCRIPT start_POSTSUPERSCRIPT italic_μ ( roman_Ω ) end_POSTSUPERSCRIPT ( italic_a start_POSTSUPERSCRIPT italic_r ( ⋅ ) end_POSTSUPERSCRIPT ) start_POSTSUPERSCRIPT ∗ end_POSTSUPERSCRIPT ( italic_x ) italic_d italic_x = ∫ start_POSTSUBSCRIPT 0 end_POSTSUBSCRIPT start_POSTSUPERSCRIPT italic_μ ( roman_Ω ) end_POSTSUPERSCRIPT italic_a start_POSTSUPERSCRIPT italic_r start_POSTSUPERSCRIPT ∗ end_POSTSUPERSCRIPT ( italic_x ) end_POSTSUPERSCRIPT italic_d italic_x .
Lemma 3.1.

Let  (Ω,μ)Ω𝜇(\Omega,\mu)( roman_Ω , italic_μ ) be an atomless finite measure space and r()𝑟r(\cdot)italic_r ( ⋅ ) be an exponent. If there exists   a>1𝑎1a>1italic_a > 1  such that   0μ(Ω)ar(x)𝑑x=superscriptsubscript0𝜇Ωsuperscript𝑎superscript𝑟𝑥differential-d𝑥\int_{0}^{\mu(\Omega)}a^{r^{*}(x)}dx=\infty∫ start_POSTSUBSCRIPT 0 end_POSTSUBSCRIPT start_POSTSUPERSCRIPT italic_μ ( roman_Ω ) end_POSTSUPERSCRIPT italic_a start_POSTSUPERSCRIPT italic_r start_POSTSUPERSCRIPT ∗ end_POSTSUPERSCRIPT ( italic_x ) end_POSTSUPERSCRIPT italic_d italic_x = ∞, then there exist   β>0𝛽0\beta>0italic_β > 0 and a disjoint measurable sequence   (En)n=1superscriptsubscriptsubscript𝐸𝑛𝑛1(E_{n})_{n=1}^{\infty}( italic_E start_POSTSUBSCRIPT italic_n end_POSTSUBSCRIPT ) start_POSTSUBSCRIPT italic_n = 1 end_POSTSUBSCRIPT start_POSTSUPERSCRIPT ∞ end_POSTSUPERSCRIPT   such that, for every natural n𝑛nitalic_n,

χEnr()β.subscriptnormsubscript𝜒subscript𝐸𝑛𝑟𝛽||\chi_{E_{n}}||_{r(\cdot)}\geq\,\beta.| | italic_χ start_POSTSUBSCRIPT italic_E start_POSTSUBSCRIPT italic_n end_POSTSUBSCRIPT end_POSTSUBSCRIPT | | start_POSTSUBSCRIPT italic_r ( ⋅ ) end_POSTSUBSCRIPT ≥ italic_β .
Proof.

Let 0<β<10𝛽10<\beta<10 < italic_β < 1   with   1/β>a1𝛽𝑎1/\beta>a1 / italic_β > italic_a. Since the rearrangement r()superscript𝑟r^{*}(\cdot)italic_r start_POSTSUPERSCRIPT ∗ end_POSTSUPERSCRIPT ( ⋅ ) is decreasing and 0μ(Ω)ar(x)𝑑x=superscriptsubscript0𝜇Ωsuperscript𝑎superscript𝑟𝑥differential-d𝑥\int_{0}^{\mu(\Omega)}a^{r^{*}(x)}dx=\infty∫ start_POSTSUBSCRIPT 0 end_POSTSUBSCRIPT start_POSTSUPERSCRIPT italic_μ ( roman_Ω ) end_POSTSUPERSCRIPT italic_a start_POSTSUPERSCRIPT italic_r start_POSTSUPERSCRIPT ∗ end_POSTSUPERSCRIPT ( italic_x ) end_POSTSUPERSCRIPT italic_d italic_x = ∞ , there exists a positive sequence (tn)subscript𝑡𝑛(t_{n})( italic_t start_POSTSUBSCRIPT italic_n end_POSTSUBSCRIPT ), with tn0subscript𝑡𝑛0t_{n}\searrow 0italic_t start_POSTSUBSCRIPT italic_n end_POSTSUBSCRIPT ↘ 0   and t1=μ(Ω)subscript𝑡1𝜇Ωt_{1}=\mu(\Omega)italic_t start_POSTSUBSCRIPT 1 end_POSTSUBSCRIPT = italic_μ ( roman_Ω ) , such that

tn+1tnar(x)𝑑x>1.superscriptsubscriptsubscript𝑡𝑛1subscript𝑡𝑛superscript𝑎superscript𝑟𝑥differential-d𝑥1\int_{t_{n+1}}^{t_{n}}\,a^{r^{*}(x)}\,dx>1.∫ start_POSTSUBSCRIPT italic_t start_POSTSUBSCRIPT italic_n + 1 end_POSTSUBSCRIPT end_POSTSUBSCRIPT start_POSTSUPERSCRIPT italic_t start_POSTSUBSCRIPT italic_n end_POSTSUBSCRIPT end_POSTSUPERSCRIPT italic_a start_POSTSUPERSCRIPT italic_r start_POSTSUPERSCRIPT ∗ end_POSTSUPERSCRIPT ( italic_x ) end_POSTSUPERSCRIPT italic_d italic_x > 1 .

Now, for each tn>0subscript𝑡𝑛0t_{n}>0italic_t start_POSTSUBSCRIPT italic_n end_POSTSUBSCRIPT > 0 we can find a measurable set FnΩsubscript𝐹𝑛ΩF_{n}\subset\Omegaitalic_F start_POSTSUBSCRIPT italic_n end_POSTSUBSCRIPT ⊂ roman_Ω with  μ(Fn)=tn𝜇subscript𝐹𝑛subscript𝑡𝑛\mu(F_{n})=t_{n}italic_μ ( italic_F start_POSTSUBSCRIPT italic_n end_POSTSUBSCRIPT ) = italic_t start_POSTSUBSCRIPT italic_n end_POSTSUBSCRIPT  such that

Fnar(t)𝑑μ=0tn(ar())(x)𝑑x=0tnar(x)𝑑x.subscriptsubscript𝐹𝑛superscript𝑎𝑟𝑡differential-d𝜇superscriptsubscript0subscript𝑡𝑛superscriptsuperscript𝑎𝑟𝑥differential-d𝑥superscriptsubscript0subscript𝑡𝑛superscript𝑎superscript𝑟𝑥differential-d𝑥\int_{F_{n}}\,a^{r(t)}\,d\mu\,=\,\int_{0}^{t_{n}}(a^{r(\cdot)})^{*}(x)\,dx=\,% \int_{0}^{t_{n}}\,a^{r^{*}(x)}\,dx.∫ start_POSTSUBSCRIPT italic_F start_POSTSUBSCRIPT italic_n end_POSTSUBSCRIPT end_POSTSUBSCRIPT italic_a start_POSTSUPERSCRIPT italic_r ( italic_t ) end_POSTSUPERSCRIPT italic_d italic_μ = ∫ start_POSTSUBSCRIPT 0 end_POSTSUBSCRIPT start_POSTSUPERSCRIPT italic_t start_POSTSUBSCRIPT italic_n end_POSTSUBSCRIPT end_POSTSUPERSCRIPT ( italic_a start_POSTSUPERSCRIPT italic_r ( ⋅ ) end_POSTSUPERSCRIPT ) start_POSTSUPERSCRIPT ∗ end_POSTSUPERSCRIPT ( italic_x ) italic_d italic_x = ∫ start_POSTSUBSCRIPT 0 end_POSTSUBSCRIPT start_POSTSUPERSCRIPT italic_t start_POSTSUBSCRIPT italic_n end_POSTSUBSCRIPT end_POSTSUPERSCRIPT italic_a start_POSTSUPERSCRIPT italic_r start_POSTSUPERSCRIPT ∗ end_POSTSUPERSCRIPT ( italic_x ) end_POSTSUPERSCRIPT italic_d italic_x .

Moreover, the sets (Fn)subscript𝐹𝑛(F_{n})( italic_F start_POSTSUBSCRIPT italic_n end_POSTSUBSCRIPT ) can be defined so that Fn+1Fnsubscript𝐹𝑛1subscript𝐹𝑛F_{n+1}\subset F_{n}italic_F start_POSTSUBSCRIPT italic_n + 1 end_POSTSUBSCRIPT ⊂ italic_F start_POSTSUBSCRIPT italic_n end_POSTSUBSCRIPT   since tn+1<tnsubscript𝑡𝑛1subscript𝑡𝑛t_{n+1}<t_{n}italic_t start_POSTSUBSCRIPT italic_n + 1 end_POSTSUBSCRIPT < italic_t start_POSTSUBSCRIPT italic_n end_POSTSUBSCRIPT  (cf. [6] Lemma 2.2.5).

Consider now the disjoint measurable sequence (En)subscript𝐸𝑛(E_{n})( italic_E start_POSTSUBSCRIPT italic_n end_POSTSUBSCRIPT ) where   En:=Fn\Fn+1assignsubscript𝐸𝑛\subscript𝐹𝑛subscript𝐹𝑛1E_{n}:=F_{n}\backslash F_{n+1}italic_E start_POSTSUBSCRIPT italic_n end_POSTSUBSCRIPT := italic_F start_POSTSUBSCRIPT italic_n end_POSTSUBSCRIPT \ italic_F start_POSTSUBSCRIPT italic_n + 1 end_POSTSUBSCRIPT. Then,

Ω(χEnβ)r(t)𝑑μsubscriptΩsuperscriptsubscript𝜒subscript𝐸𝑛𝛽𝑟𝑡differential-d𝜇absent\displaystyle\int_{\Omega}(\frac{\chi_{E_{n}}}{\beta})^{r(t)}\,d\mu\geq{}∫ start_POSTSUBSCRIPT roman_Ω end_POSTSUBSCRIPT ( divide start_ARG italic_χ start_POSTSUBSCRIPT italic_E start_POSTSUBSCRIPT italic_n end_POSTSUBSCRIPT end_POSTSUBSCRIPT end_ARG start_ARG italic_β end_ARG ) start_POSTSUPERSCRIPT italic_r ( italic_t ) end_POSTSUPERSCRIPT italic_d italic_μ ≥ En(aχEn)r(t)𝑑μ=Fn(aχFn)r(t)𝑑μFn+1(aχFn+1)r(t)𝑑μsubscriptsubscript𝐸𝑛superscript𝑎subscript𝜒subscript𝐸𝑛𝑟𝑡differential-d𝜇subscriptsubscript𝐹𝑛superscript𝑎subscript𝜒subscript𝐹𝑛𝑟𝑡differential-d𝜇subscriptsubscript𝐹𝑛1superscript𝑎subscript𝜒subscript𝐹𝑛1𝑟𝑡differential-d𝜇\displaystyle\int_{E_{n}}(a\chi_{E_{n}})^{r(t)}d\mu\,=\int_{F_{n}}(a\chi_{F_{n% }})^{r(t)}d\mu\,-\int_{F_{n+1}}(a\chi_{F_{n+1}})^{r(t)}d\mu∫ start_POSTSUBSCRIPT italic_E start_POSTSUBSCRIPT italic_n end_POSTSUBSCRIPT end_POSTSUBSCRIPT ( italic_a italic_χ start_POSTSUBSCRIPT italic_E start_POSTSUBSCRIPT italic_n end_POSTSUBSCRIPT end_POSTSUBSCRIPT ) start_POSTSUPERSCRIPT italic_r ( italic_t ) end_POSTSUPERSCRIPT italic_d italic_μ = ∫ start_POSTSUBSCRIPT italic_F start_POSTSUBSCRIPT italic_n end_POSTSUBSCRIPT end_POSTSUBSCRIPT ( italic_a italic_χ start_POSTSUBSCRIPT italic_F start_POSTSUBSCRIPT italic_n end_POSTSUBSCRIPT end_POSTSUBSCRIPT ) start_POSTSUPERSCRIPT italic_r ( italic_t ) end_POSTSUPERSCRIPT italic_d italic_μ - ∫ start_POSTSUBSCRIPT italic_F start_POSTSUBSCRIPT italic_n + 1 end_POSTSUBSCRIPT end_POSTSUBSCRIPT ( italic_a italic_χ start_POSTSUBSCRIPT italic_F start_POSTSUBSCRIPT italic_n + 1 end_POSTSUBSCRIPT end_POSTSUBSCRIPT ) start_POSTSUPERSCRIPT italic_r ( italic_t ) end_POSTSUPERSCRIPT italic_d italic_μ
=\displaystyle={}= 0tnar(x)𝑑x0tn+1ar(x)𝑑x=tn+1tnar(x)𝑑x>  1superscriptsubscript0subscript𝑡𝑛superscript𝑎superscript𝑟𝑥differential-d𝑥superscriptsubscript0subscript𝑡𝑛1superscript𝑎superscript𝑟𝑥differential-d𝑥superscriptsubscriptsubscript𝑡𝑛1subscript𝑡𝑛superscript𝑎superscript𝑟𝑥differential-d𝑥1\displaystyle\int_{0}^{t_{n}}a^{r^{*}(x)}\,dx\,-\int_{0}^{t_{n+1}}\,a^{r^{*}(x% )}dx=\int_{t_{n+1}}^{t_{n}}a^{r^{*}(x)}\,dx\,\,>\,\,1∫ start_POSTSUBSCRIPT 0 end_POSTSUBSCRIPT start_POSTSUPERSCRIPT italic_t start_POSTSUBSCRIPT italic_n end_POSTSUBSCRIPT end_POSTSUPERSCRIPT italic_a start_POSTSUPERSCRIPT italic_r start_POSTSUPERSCRIPT ∗ end_POSTSUPERSCRIPT ( italic_x ) end_POSTSUPERSCRIPT italic_d italic_x - ∫ start_POSTSUBSCRIPT 0 end_POSTSUBSCRIPT start_POSTSUPERSCRIPT italic_t start_POSTSUBSCRIPT italic_n + 1 end_POSTSUBSCRIPT end_POSTSUPERSCRIPT italic_a start_POSTSUPERSCRIPT italic_r start_POSTSUPERSCRIPT ∗ end_POSTSUPERSCRIPT ( italic_x ) end_POSTSUPERSCRIPT italic_d italic_x = ∫ start_POSTSUBSCRIPT italic_t start_POSTSUBSCRIPT italic_n + 1 end_POSTSUBSCRIPT end_POSTSUBSCRIPT start_POSTSUPERSCRIPT italic_t start_POSTSUBSCRIPT italic_n end_POSTSUBSCRIPT end_POSTSUPERSCRIPT italic_a start_POSTSUPERSCRIPT italic_r start_POSTSUPERSCRIPT ∗ end_POSTSUPERSCRIPT ( italic_x ) end_POSTSUPERSCRIPT italic_d italic_x > 1

for every natural n𝑛nitalic_n. Hence, χEnr()βsubscriptnormsubscript𝜒subscript𝐸𝑛𝑟𝛽||\chi_{E_{n}}||_{r(\cdot)}\geq\,\beta| | italic_χ start_POSTSUBSCRIPT italic_E start_POSTSUBSCRIPT italic_n end_POSTSUBSCRIPT end_POSTSUBSCRIPT | | start_POSTSUBSCRIPT italic_r ( ⋅ ) end_POSTSUBSCRIPT ≥ italic_β. ∎

Proposition 3.2.

Let (Ω,μ)Ω𝜇(\Omega,\mu)( roman_Ω , italic_μ ) be an atomless finite measure space and an exponent  r()𝑟r(\cdot)italic_r ( ⋅ ). Then every sequence (En)n=1superscriptsubscriptsubscript𝐸𝑛𝑛1(E_{n})_{n=1}^{\infty}( italic_E start_POSTSUBSCRIPT italic_n end_POSTSUBSCRIPT ) start_POSTSUBSCRIPT italic_n = 1 end_POSTSUBSCRIPT start_POSTSUPERSCRIPT ∞ end_POSTSUPERSCRIPT in  ΩΩ\Omegaroman_Ω with χEn0subscript𝜒subscript𝐸𝑛0\chi_{E_{n}}\rightarrow 0italic_χ start_POSTSUBSCRIPT italic_E start_POSTSUBSCRIPT italic_n end_POSTSUBSCRIPT end_POSTSUBSCRIPT → 0 μ𝜇\muitalic_μ-a.e. satisfies  χEnr()0subscriptnormsubscript𝜒subscript𝐸𝑛𝑟0||\chi_{E_{n}}||_{r(\cdot)}\rightarrow 0| | italic_χ start_POSTSUBSCRIPT italic_E start_POSTSUBSCRIPT italic_n end_POSTSUBSCRIPT end_POSTSUBSCRIPT | | start_POSTSUBSCRIPT italic_r ( ⋅ ) end_POSTSUBSCRIPT → 0   if and only if for every   a>1𝑎1a>1italic_a > 1,

0μ(Ω)ar(x)𝑑x<.superscriptsubscript0𝜇Ωsuperscript𝑎superscript𝑟𝑥differential-d𝑥\int_{0}^{\mu(\Omega)}a^{r^{*}(x)}dx<\infty.∫ start_POSTSUBSCRIPT 0 end_POSTSUBSCRIPT start_POSTSUPERSCRIPT italic_μ ( roman_Ω ) end_POSTSUPERSCRIPT italic_a start_POSTSUPERSCRIPT italic_r start_POSTSUPERSCRIPT ∗ end_POSTSUPERSCRIPT ( italic_x ) end_POSTSUPERSCRIPT italic_d italic_x < ∞ .
Proof.

The direct implication is above Lemma 3.1. Let us show the converse. Assume there exist a measurable sequence (En)subscript𝐸𝑛(E_{n})( italic_E start_POSTSUBSCRIPT italic_n end_POSTSUBSCRIPT ) with  χEn0subscript𝜒subscript𝐸𝑛0\chi_{E_{n}}\rightarrow 0italic_χ start_POSTSUBSCRIPT italic_E start_POSTSUBSCRIPT italic_n end_POSTSUBSCRIPT end_POSTSUBSCRIPT → 0 μ𝜇\muitalic_μ-a.e. (thus  μ(En)0𝜇subscript𝐸𝑛0\mu(E_{n})\rightarrow 0italic_μ ( italic_E start_POSTSUBSCRIPT italic_n end_POSTSUBSCRIPT ) → 0 ) and   0<δ<10𝛿10<\delta<10 < italic_δ < 1   such that χEnr()δsubscriptnormsubscript𝜒subscript𝐸𝑛𝑟𝛿||\chi_{E_{n}}||_{r(\cdot)}\geq\delta| | italic_χ start_POSTSUBSCRIPT italic_E start_POSTSUBSCRIPT italic_n end_POSTSUBSCRIPT end_POSTSUBSCRIPT | | start_POSTSUBSCRIPT italic_r ( ⋅ ) end_POSTSUBSCRIPT ≥ italic_δ for every n𝑛nitalic_n. Taking  0<β<δ0𝛽𝛿0<\beta<\delta0 < italic_β < italic_δ, it follows from the norm definition that

1<Ω(χEnβ)r(t)𝑑μ.1subscriptΩsuperscriptsubscript𝜒subscript𝐸𝑛𝛽𝑟𝑡differential-d𝜇1<\int_{\Omega}(\frac{\chi_{E_{n}}}{\beta})^{r(t)}d\mu.1 < ∫ start_POSTSUBSCRIPT roman_Ω end_POSTSUBSCRIPT ( divide start_ARG italic_χ start_POSTSUBSCRIPT italic_E start_POSTSUBSCRIPT italic_n end_POSTSUBSCRIPT end_POSTSUBSCRIPT end_ARG start_ARG italic_β end_ARG ) start_POSTSUPERSCRIPT italic_r ( italic_t ) end_POSTSUPERSCRIPT italic_d italic_μ .

Now, by the hypotheses,

Ω(χΩβ)r(t)𝑑μ=0μ(Ω)(1β)r(x)𝑑x<subscriptΩsuperscriptsubscript𝜒Ω𝛽𝑟𝑡differential-d𝜇superscriptsubscript0𝜇Ωsuperscript1𝛽superscript𝑟𝑥differential-d𝑥\int_{\Omega}(\frac{\chi_{\Omega}}{\beta})^{r(t)}d\mu=\int_{0}^{\mu(\Omega)}(% \frac{1}{\beta})^{r^{*}(x)}dx<\infty∫ start_POSTSUBSCRIPT roman_Ω end_POSTSUBSCRIPT ( divide start_ARG italic_χ start_POSTSUBSCRIPT roman_Ω end_POSTSUBSCRIPT end_ARG start_ARG italic_β end_ARG ) start_POSTSUPERSCRIPT italic_r ( italic_t ) end_POSTSUPERSCRIPT italic_d italic_μ = ∫ start_POSTSUBSCRIPT 0 end_POSTSUBSCRIPT start_POSTSUPERSCRIPT italic_μ ( roman_Ω ) end_POSTSUPERSCRIPT ( divide start_ARG 1 end_ARG start_ARG italic_β end_ARG ) start_POSTSUPERSCRIPT italic_r start_POSTSUPERSCRIPT ∗ end_POSTSUPERSCRIPT ( italic_x ) end_POSTSUPERSCRIPT italic_d italic_x < ∞

and, as χEnβ0μa.e.subscript𝜒subscript𝐸𝑛𝛽0𝜇a.e.\frac{\chi_{E_{n}}}{\beta}\rightarrow 0\,\mu-\text{a.e.}divide start_ARG italic_χ start_POSTSUBSCRIPT italic_E start_POSTSUBSCRIPT italic_n end_POSTSUBSCRIPT end_POSTSUBSCRIPT end_ARG start_ARG italic_β end_ARG → 0 italic_μ - a.e., we conclude using the dominated convergence theorem that

Ω(χEnβ)r(t)𝑑μ0subscriptΩsuperscriptsubscript𝜒subscript𝐸𝑛𝛽𝑟𝑡differential-d𝜇0\int_{\Omega}(\frac{\chi_{E_{n}}}{\beta})^{r(t)}d\mu\rightarrow 0\,∫ start_POSTSUBSCRIPT roman_Ω end_POSTSUBSCRIPT ( divide start_ARG italic_χ start_POSTSUBSCRIPT italic_E start_POSTSUBSCRIPT italic_n end_POSTSUBSCRIPT end_POSTSUBSCRIPT end_ARG start_ARG italic_β end_ARG ) start_POSTSUPERSCRIPT italic_r ( italic_t ) end_POSTSUPERSCRIPT italic_d italic_μ → 0

as nmaps-to𝑛n\mapsto\inftyitalic_n ↦ ∞, which is a contradiction. ∎

The above equivalence, for bounded exponents, was crucial for proving Theorem 3.4 in [9] (see also [10]).

Proposition 3.3.

Let  (Ω,μ)Ω𝜇(\Omega,\mu)( roman_Ω , italic_μ )  be an atomless finite measure space and exponents  q()p()𝑞𝑝q(\cdot)\leq p(\cdot)italic_q ( ⋅ ) ≤ italic_p ( ⋅ ). If the inclusion  Lp()(μ)Lq()(μ)superscript𝐿𝑝𝜇superscript𝐿𝑞𝜇{L^{p(\cdot)}(\mu)}\hookrightarrow{L^{q(\cdot)}(\mu)}italic_L start_POSTSUPERSCRIPT italic_p ( ⋅ ) end_POSTSUPERSCRIPT ( italic_μ ) ↪ italic_L start_POSTSUPERSCRIPT italic_q ( ⋅ ) end_POSTSUPERSCRIPT ( italic_μ )  is DSS, then

limxμ(Ω)(μ(Ω)x)(pqpq)(x)=0.subscript𝑥𝜇superscriptΩsuperscript𝜇Ω𝑥superscript𝑝𝑞𝑝𝑞𝑥0\lim_{x\rightarrow\mu(\Omega)^{-}}\,(\mu(\Omega)-x)^{(\frac{p-q}{p\,q})^{*}(x)% }=0.roman_lim start_POSTSUBSCRIPT italic_x → italic_μ ( roman_Ω ) start_POSTSUPERSCRIPT - end_POSTSUPERSCRIPT end_POSTSUBSCRIPT ( italic_μ ( roman_Ω ) - italic_x ) start_POSTSUPERSCRIPT ( divide start_ARG italic_p - italic_q end_ARG start_ARG italic_p italic_q end_ARG ) start_POSTSUPERSCRIPT ∗ end_POSTSUPERSCRIPT ( italic_x ) end_POSTSUPERSCRIPT = 0 .
Proof.

Since the inclusion is DSS we have  μ({t:p(t)=q(t)})=0𝜇conditional-set𝑡𝑝𝑡𝑞𝑡0\mu(\{t:\,p(t)=q(t)\})=0italic_μ ( { italic_t : italic_p ( italic_t ) = italic_q ( italic_t ) } ) = 0. First note that if  essinf(pqpq)>0𝑒𝑠𝑠infimumsuperscript𝑝𝑞𝑝𝑞0ess\inf(\frac{p-q}{pq})^{*}>0italic_e italic_s italic_s roman_inf ( divide start_ARG italic_p - italic_q end_ARG start_ARG italic_p italic_q end_ARG ) start_POSTSUPERSCRIPT ∗ end_POSTSUPERSCRIPT > 0, then limxμ(Ω)(μ(Ω)x)(pqpq)(x)=0subscript𝑥𝜇superscriptΩsuperscript𝜇Ω𝑥superscript𝑝𝑞𝑝𝑞𝑥0\lim_{x\rightarrow\mu(\Omega)^{-}}(\mu(\Omega)-x)^{(\frac{p-q}{pq})^{*}(x)}=0roman_lim start_POSTSUBSCRIPT italic_x → italic_μ ( roman_Ω ) start_POSTSUPERSCRIPT - end_POSTSUPERSCRIPT end_POSTSUBSCRIPT ( italic_μ ( roman_Ω ) - italic_x ) start_POSTSUPERSCRIPT ( divide start_ARG italic_p - italic_q end_ARG start_ARG italic_p italic_q end_ARG ) start_POSTSUPERSCRIPT ∗ end_POSTSUPERSCRIPT ( italic_x ) end_POSTSUPERSCRIPT = 0. Hence (since the rearrangement is decreasing), we can suppose

limxμ(Ω)(pqpq)(x)=0.subscript𝑥𝜇superscriptΩsuperscript𝑝𝑞𝑝𝑞𝑥0\lim_{x\rightarrow\mu(\Omega)^{-}}\,(\frac{p-q}{p\,q})^{*}(x)=0.roman_lim start_POSTSUBSCRIPT italic_x → italic_μ ( roman_Ω ) start_POSTSUPERSCRIPT - end_POSTSUPERSCRIPT end_POSTSUBSCRIPT ( divide start_ARG italic_p - italic_q end_ARG start_ARG italic_p italic_q end_ARG ) start_POSTSUPERSCRIPT ∗ end_POSTSUPERSCRIPT ( italic_x ) = 0 .

Let us assume that   lim supxμ(Ω)(μ(Ω)x)(pqpq)(x)>0subscriptlimit-supremum𝑥𝜇superscriptΩsuperscript𝜇Ω𝑥superscript𝑝𝑞𝑝𝑞𝑥0\limsup_{x\rightarrow\mu(\Omega)^{-}}(\mu(\Omega)-x)^{(\frac{p-q}{p\,q})^{*}(x% )}>0lim sup start_POSTSUBSCRIPT italic_x → italic_μ ( roman_Ω ) start_POSTSUPERSCRIPT - end_POSTSUPERSCRIPT end_POSTSUBSCRIPT ( italic_μ ( roman_Ω ) - italic_x ) start_POSTSUPERSCRIPT ( divide start_ARG italic_p - italic_q end_ARG start_ARG italic_p italic_q end_ARG ) start_POSTSUPERSCRIPT ∗ end_POSTSUPERSCRIPT ( italic_x ) end_POSTSUPERSCRIPT > 0. Then there exist r>0𝑟0r>0italic_r > 0 and a scalar sequence   (xn)μ(Ω)subscript𝑥𝑛𝜇Ω(x_{n})\nearrow\mu(\Omega)( italic_x start_POSTSUBSCRIPT italic_n end_POSTSUBSCRIPT ) ↗ italic_μ ( roman_Ω )   such that

(\diamond) (μ(Ω)xn)(pqpq)(xn)rsuperscript𝜇Ωsubscript𝑥𝑛superscript𝑝𝑞𝑝𝑞subscript𝑥𝑛𝑟{}(\mu(\Omega)-x_{n})^{(\frac{p-q}{p\,q})^{*}(x_{n})}\geq\,r( italic_μ ( roman_Ω ) - italic_x start_POSTSUBSCRIPT italic_n end_POSTSUBSCRIPT ) start_POSTSUPERSCRIPT ( divide start_ARG italic_p - italic_q end_ARG start_ARG italic_p italic_q end_ARG ) start_POSTSUPERSCRIPT ∗ end_POSTSUPERSCRIPT ( italic_x start_POSTSUBSCRIPT italic_n end_POSTSUBSCRIPT ) end_POSTSUPERSCRIPT ≥ italic_r

for every natural  n𝑛nitalic_n. Furthermore it can be assumed w.l.o.g. that   xn+μ(Ω)2<xn+1subscript𝑥𝑛𝜇Ω2subscript𝑥𝑛1\frac{x_{n}+\mu(\Omega)}{2}<x_{n+1}divide start_ARG italic_x start_POSTSUBSCRIPT italic_n end_POSTSUBSCRIPT + italic_μ ( roman_Ω ) end_ARG start_ARG 2 end_ARG < italic_x start_POSTSUBSCRIPT italic_n + 1 end_POSTSUBSCRIPT   and

(pqpq)(xn+μ(Ω)2)>(pqpq)(xn+1)superscript𝑝𝑞𝑝𝑞subscript𝑥𝑛𝜇Ω2superscript𝑝𝑞𝑝𝑞subscript𝑥𝑛1(\frac{p-q}{p\,q})^{*}(\frac{x_{n}+\mu(\Omega)}{2})>(\frac{p-q}{p\,q})^{*}(x_{% n+1})( divide start_ARG italic_p - italic_q end_ARG start_ARG italic_p italic_q end_ARG ) start_POSTSUPERSCRIPT ∗ end_POSTSUPERSCRIPT ( divide start_ARG italic_x start_POSTSUBSCRIPT italic_n end_POSTSUBSCRIPT + italic_μ ( roman_Ω ) end_ARG start_ARG 2 end_ARG ) > ( divide start_ARG italic_p - italic_q end_ARG start_ARG italic_p italic_q end_ARG ) start_POSTSUPERSCRIPT ∗ end_POSTSUPERSCRIPT ( italic_x start_POSTSUBSCRIPT italic_n + 1 end_POSTSUBSCRIPT )

for every natural n𝑛nitalic_n. Consider the sets

An=((pqpq))1([(pqpq)(xn+μ(Ω)2),(pqpq)(xn)])[xn,xn+μ(Ω)2].subscript𝐴𝑛superscriptsuperscript𝑝𝑞𝑝𝑞1superscript𝑝𝑞𝑝𝑞subscript𝑥𝑛𝜇Ω2superscript𝑝𝑞𝑝𝑞subscript𝑥𝑛superset-of-or-equalssubscript𝑥𝑛subscript𝑥𝑛𝜇Ω2A_{n}=\big{(}\,(\frac{p-q}{p\,q})^{*}\big{)}^{-1}\left(\,[(\frac{p-q}{p\,q})^{% *}(\frac{x_{n}+\mu(\Omega)}{2}),(\frac{p-q}{p\,q})^{*}(x_{n})]\right)% \supseteqq\;[x_{n},\frac{x_{n}+\mu(\Omega)}{2}].italic_A start_POSTSUBSCRIPT italic_n end_POSTSUBSCRIPT = ( ( divide start_ARG italic_p - italic_q end_ARG start_ARG italic_p italic_q end_ARG ) start_POSTSUPERSCRIPT ∗ end_POSTSUPERSCRIPT ) start_POSTSUPERSCRIPT - 1 end_POSTSUPERSCRIPT ( [ ( divide start_ARG italic_p - italic_q end_ARG start_ARG italic_p italic_q end_ARG ) start_POSTSUPERSCRIPT ∗ end_POSTSUPERSCRIPT ( divide start_ARG italic_x start_POSTSUBSCRIPT italic_n end_POSTSUBSCRIPT + italic_μ ( roman_Ω ) end_ARG start_ARG 2 end_ARG ) , ( divide start_ARG italic_p - italic_q end_ARG start_ARG italic_p italic_q end_ARG ) start_POSTSUPERSCRIPT ∗ end_POSTSUPERSCRIPT ( italic_x start_POSTSUBSCRIPT italic_n end_POSTSUBSCRIPT ) ] ) ⫆ [ italic_x start_POSTSUBSCRIPT italic_n end_POSTSUBSCRIPT , divide start_ARG italic_x start_POSTSUBSCRIPT italic_n end_POSTSUBSCRIPT + italic_μ ( roman_Ω ) end_ARG start_ARG 2 end_ARG ] .

Thus (An)subscript𝐴𝑛(A_{n})( italic_A start_POSTSUBSCRIPT italic_n end_POSTSUBSCRIPT ) is a disjoint measurable sequence with Lebesgue measure

(\diamond\diamond⋄ ⋄) |An|μ(Ω)xn2.subscript𝐴𝑛𝜇Ωsubscript𝑥𝑛2{}|A_{n}|\,\geq\,\frac{\mu(\Omega)-x_{n}}{2}.| italic_A start_POSTSUBSCRIPT italic_n end_POSTSUBSCRIPT | ≥ divide start_ARG italic_μ ( roman_Ω ) - italic_x start_POSTSUBSCRIPT italic_n end_POSTSUBSCRIPT end_ARG start_ARG 2 end_ARG .

Let us define the sets

Bn={tΩ:(pqpq)(xn+μ(Ω)2)pqpq(t)(pqpq)(xn)}.subscript𝐵𝑛conditional-set𝑡Ωsuperscript𝑝𝑞𝑝𝑞subscript𝑥𝑛𝜇Ω2𝑝𝑞𝑝𝑞𝑡superscript𝑝𝑞𝑝𝑞subscript𝑥𝑛B_{n}=\{t\in\Omega\,\,:\,\,(\frac{p-q}{p\,q})^{*}(\frac{x_{n}+\mu(\Omega)}{2})% \leq\frac{p-q}{p\,q}(t)\leq(\frac{p-q}{p\,q})^{*}(x_{n})\}.italic_B start_POSTSUBSCRIPT italic_n end_POSTSUBSCRIPT = { italic_t ∈ roman_Ω : ( divide start_ARG italic_p - italic_q end_ARG start_ARG italic_p italic_q end_ARG ) start_POSTSUPERSCRIPT ∗ end_POSTSUPERSCRIPT ( divide start_ARG italic_x start_POSTSUBSCRIPT italic_n end_POSTSUBSCRIPT + italic_μ ( roman_Ω ) end_ARG start_ARG 2 end_ARG ) ≤ divide start_ARG italic_p - italic_q end_ARG start_ARG italic_p italic_q end_ARG ( italic_t ) ≤ ( divide start_ARG italic_p - italic_q end_ARG start_ARG italic_p italic_q end_ARG ) start_POSTSUPERSCRIPT ∗ end_POSTSUPERSCRIPT ( italic_x start_POSTSUBSCRIPT italic_n end_POSTSUBSCRIPT ) } .

Since the functions  pqpq𝑝𝑞𝑝𝑞\frac{p-q}{p\,q}divide start_ARG italic_p - italic_q end_ARG start_ARG italic_p italic_q end_ARG and (pqpq)superscript𝑝𝑞𝑝𝑞(\frac{p-q}{p\,q})^{*}( divide start_ARG italic_p - italic_q end_ARG start_ARG italic_p italic_q end_ARG ) start_POSTSUPERSCRIPT ∗ end_POSTSUPERSCRIPT are equi-distributed we have  μ(Bn)=|An|𝜇subscript𝐵𝑛subscript𝐴𝑛\mu(B_{n})=|A_{n}|italic_μ ( italic_B start_POSTSUBSCRIPT italic_n end_POSTSUBSCRIPT ) = | italic_A start_POSTSUBSCRIPT italic_n end_POSTSUBSCRIPT |. The sets (Bn)subscript𝐵𝑛(B_{n})( italic_B start_POSTSUBSCRIPT italic_n end_POSTSUBSCRIPT ) are disjoint and it can be assumed   0<μ(Bn)<10𝜇subscript𝐵𝑛10<\mu(B_{n})<10 < italic_μ ( italic_B start_POSTSUBSCRIPT italic_n end_POSTSUBSCRIPT ) < 1 for every n𝑛nitalic_n. Consider the disjoint normalized sequence  (sn)subscript𝑠𝑛(s_{n})( italic_s start_POSTSUBSCRIPT italic_n end_POSTSUBSCRIPT )  in Lp()(μ)superscript𝐿𝑝𝜇{L^{p(\cdot)}(\mu)}italic_L start_POSTSUPERSCRIPT italic_p ( ⋅ ) end_POSTSUPERSCRIPT ( italic_μ )

sn(t):=χBnμ(Bn)1p(t)assignsubscript𝑠𝑛𝑡subscript𝜒subscript𝐵𝑛𝜇superscriptsubscript𝐵𝑛1𝑝𝑡s_{n}(t):=\frac{\chi_{B_{n}}}{\mu(B_{n})^{\frac{1}{p(t)}}}italic_s start_POSTSUBSCRIPT italic_n end_POSTSUBSCRIPT ( italic_t ) := divide start_ARG italic_χ start_POSTSUBSCRIPT italic_B start_POSTSUBSCRIPT italic_n end_POSTSUBSCRIPT end_POSTSUBSCRIPT end_ARG start_ARG italic_μ ( italic_B start_POSTSUBSCRIPT italic_n end_POSTSUBSCRIPT ) start_POSTSUPERSCRIPT divide start_ARG 1 end_ARG start_ARG italic_p ( italic_t ) end_ARG end_POSTSUPERSCRIPT end_ARG

and the (closed) subspace  [sn]p()subscriptdelimited-[]subscript𝑠𝑛𝑝[s_{n}]_{p(\cdot)}[ italic_s start_POSTSUBSCRIPT italic_n end_POSTSUBSCRIPT ] start_POSTSUBSCRIPT italic_p ( ⋅ ) end_POSTSUBSCRIPT. Note that  nansn[sn]p()subscript𝑛subscript𝑎𝑛subscript𝑠𝑛subscriptdelimited-[]subscript𝑠𝑛𝑝\sum_{n}a_{n}s_{n}\in[s_{n}]_{p(\cdot)}∑ start_POSTSUBSCRIPT italic_n end_POSTSUBSCRIPT italic_a start_POSTSUBSCRIPT italic_n end_POSTSUBSCRIPT italic_s start_POSTSUBSCRIPT italic_n end_POSTSUBSCRIPT ∈ [ italic_s start_POSTSUBSCRIPT italic_n end_POSTSUBSCRIPT ] start_POSTSUBSCRIPT italic_p ( ⋅ ) end_POSTSUBSCRIPT  if and only if

ρp()(λn>Nansn)N0for everyλ>0.formulae-sequence𝑁subscript𝜌𝑝𝜆subscript𝑛𝑁subscript𝑎𝑛subscript𝑠𝑛0for every𝜆0\rho_{p(\cdot)}(\lambda\sum_{n>N}a_{n}s_{n})\xrightarrow{{N\rightarrow\infty}}% 0\quad\text{for every}\quad\lambda>0.italic_ρ start_POSTSUBSCRIPT italic_p ( ⋅ ) end_POSTSUBSCRIPT ( italic_λ ∑ start_POSTSUBSCRIPT italic_n > italic_N end_POSTSUBSCRIPT italic_a start_POSTSUBSCRIPT italic_n end_POSTSUBSCRIPT italic_s start_POSTSUBSCRIPT italic_n end_POSTSUBSCRIPT ) start_ARROW start_OVERACCENT italic_N → ∞ end_OVERACCENT → end_ARROW 0 for every italic_λ > 0 .

Let us prove that i|[sn]p()evaluated-at𝑖subscriptdelimited-[]subscript𝑠𝑛𝑝i|_{[s_{n}]_{p(\cdot)}}italic_i | start_POSTSUBSCRIPT [ italic_s start_POSTSUBSCRIPT italic_n end_POSTSUBSCRIPT ] start_POSTSUBSCRIPT italic_p ( ⋅ ) end_POSTSUBSCRIPT end_POSTSUBSCRIPT is an isomorphism showing that  (sn)subscript𝑠𝑛(s_{n})( italic_s start_POSTSUBSCRIPT italic_n end_POSTSUBSCRIPT ) and (isn)𝑖subscript𝑠𝑛(is_{n})( italic_i italic_s start_POSTSUBSCRIPT italic_n end_POSTSUBSCRIPT ) are equivalent basic sequences. Since i𝑖iitalic_i is continuous, we only need to show that ynsn[sn]q()subscript𝑦𝑛subscript𝑠𝑛subscriptdelimited-[]subscript𝑠𝑛𝑞\sum y_{n}s_{n}\in[s_{n}]_{q(\cdot)}∑ italic_y start_POSTSUBSCRIPT italic_n end_POSTSUBSCRIPT italic_s start_POSTSUBSCRIPT italic_n end_POSTSUBSCRIPT ∈ [ italic_s start_POSTSUBSCRIPT italic_n end_POSTSUBSCRIPT ] start_POSTSUBSCRIPT italic_q ( ⋅ ) end_POSTSUBSCRIPT implies ynsn[sn]p().subscript𝑦𝑛subscript𝑠𝑛subscriptdelimited-[]subscript𝑠𝑛𝑝\sum y_{n}s_{n}\in[s_{n}]_{p(\cdot)}.∑ italic_y start_POSTSUBSCRIPT italic_n end_POSTSUBSCRIPT italic_s start_POSTSUBSCRIPT italic_n end_POSTSUBSCRIPT ∈ [ italic_s start_POSTSUBSCRIPT italic_n end_POSTSUBSCRIPT ] start_POSTSUBSCRIPT italic_p ( ⋅ ) end_POSTSUBSCRIPT . First notice that

ρp()(λynsn)=Bn|λyn|p(t)χBnμ(Bn)𝑑μ=Bn|λyn|q(t)|λyn|pq(t)χBn|An|q(t)p(t)|An|1q(t)p(t)𝑑μ.subscript𝜌𝑝𝜆subscript𝑦𝑛subscript𝑠𝑛subscriptsubscript𝐵𝑛superscript𝜆subscript𝑦𝑛𝑝𝑡subscript𝜒subscript𝐵𝑛𝜇subscript𝐵𝑛differential-d𝜇subscriptsubscript𝐵𝑛superscript𝜆subscript𝑦𝑛𝑞𝑡superscript𝜆subscript𝑦𝑛𝑝𝑞𝑡subscript𝜒subscript𝐵𝑛superscriptsubscript𝐴𝑛𝑞𝑡𝑝𝑡superscriptsubscript𝐴𝑛1𝑞𝑡𝑝𝑡differential-d𝜇\rho_{p(\cdot)}\left(\sum\lambda y_{n}s_{n}\right)=\sum\int_{B_{n}}|\lambda y_% {n}|^{p(t)}\frac{\chi_{B_{n}}}{\mu(B_{n})}d\mu=\sum\int_{B_{n}}|\lambda y_{n}|% ^{q(t)}|\lambda y_{n}|^{p-q(t)}\frac{\chi_{B_{n}}}{|A_{n}|^{\frac{q(t)}{p(t)}}% |A_{n}|^{1-\frac{q(t)}{p(t)}}}d\mu.italic_ρ start_POSTSUBSCRIPT italic_p ( ⋅ ) end_POSTSUBSCRIPT ( ∑ italic_λ italic_y start_POSTSUBSCRIPT italic_n end_POSTSUBSCRIPT italic_s start_POSTSUBSCRIPT italic_n end_POSTSUBSCRIPT ) = ∑ ∫ start_POSTSUBSCRIPT italic_B start_POSTSUBSCRIPT italic_n end_POSTSUBSCRIPT end_POSTSUBSCRIPT | italic_λ italic_y start_POSTSUBSCRIPT italic_n end_POSTSUBSCRIPT | start_POSTSUPERSCRIPT italic_p ( italic_t ) end_POSTSUPERSCRIPT divide start_ARG italic_χ start_POSTSUBSCRIPT italic_B start_POSTSUBSCRIPT italic_n end_POSTSUBSCRIPT end_POSTSUBSCRIPT end_ARG start_ARG italic_μ ( italic_B start_POSTSUBSCRIPT italic_n end_POSTSUBSCRIPT ) end_ARG italic_d italic_μ = ∑ ∫ start_POSTSUBSCRIPT italic_B start_POSTSUBSCRIPT italic_n end_POSTSUBSCRIPT end_POSTSUBSCRIPT | italic_λ italic_y start_POSTSUBSCRIPT italic_n end_POSTSUBSCRIPT | start_POSTSUPERSCRIPT italic_q ( italic_t ) end_POSTSUPERSCRIPT | italic_λ italic_y start_POSTSUBSCRIPT italic_n end_POSTSUBSCRIPT | start_POSTSUPERSCRIPT italic_p - italic_q ( italic_t ) end_POSTSUPERSCRIPT divide start_ARG italic_χ start_POSTSUBSCRIPT italic_B start_POSTSUBSCRIPT italic_n end_POSTSUBSCRIPT end_POSTSUBSCRIPT end_ARG start_ARG | italic_A start_POSTSUBSCRIPT italic_n end_POSTSUBSCRIPT | start_POSTSUPERSCRIPT divide start_ARG italic_q ( italic_t ) end_ARG start_ARG italic_p ( italic_t ) end_ARG end_POSTSUPERSCRIPT | italic_A start_POSTSUBSCRIPT italic_n end_POSTSUBSCRIPT | start_POSTSUPERSCRIPT 1 - divide start_ARG italic_q ( italic_t ) end_ARG start_ARG italic_p ( italic_t ) end_ARG end_POSTSUPERSCRIPT end_ARG italic_d italic_μ .

Also, |λyn|<1𝜆subscript𝑦𝑛1|\lambda y_{n}|<1| italic_λ italic_y start_POSTSUBSCRIPT italic_n end_POSTSUBSCRIPT | < 1 up to a finite amount of terms for every λ>0𝜆0\lambda>0italic_λ > 0. Otherwise, since for large enough N𝑁Nitalic_N we have ρq()(λn>Nynsn)<subscript𝜌𝑞𝜆subscript𝑛𝑁subscript𝑦𝑛subscript𝑠𝑛\rho_{q(\cdot)}(\lambda\sum_{n>N}y_{n}s_{n})<\inftyitalic_ρ start_POSTSUBSCRIPT italic_q ( ⋅ ) end_POSTSUBSCRIPT ( italic_λ ∑ start_POSTSUBSCRIPT italic_n > italic_N end_POSTSUBSCRIPT italic_y start_POSTSUBSCRIPT italic_n end_POSTSUBSCRIPT italic_s start_POSTSUBSCRIPT italic_n end_POSTSUBSCRIPT ) < ∞, taking λ0>2rsubscript𝜆02𝑟\lambda_{0}>\frac{2}{r}italic_λ start_POSTSUBSCRIPT 0 end_POSTSUBSCRIPT > divide start_ARG 2 end_ARG start_ARG italic_r end_ARG we get

ρq()(n>Nλ0λynsn)subscript𝜌𝑞subscript𝑛𝑁subscript𝜆0𝜆subscript𝑦𝑛subscript𝑠𝑛absent\displaystyle\rho_{q(\cdot)}(\sum_{n>N}\lambda_{0}\lambda\,y_{n}s_{n})\geq{}italic_ρ start_POSTSUBSCRIPT italic_q ( ⋅ ) end_POSTSUBSCRIPT ( ∑ start_POSTSUBSCRIPT italic_n > italic_N end_POSTSUBSCRIPT italic_λ start_POSTSUBSCRIPT 0 end_POSTSUBSCRIPT italic_λ italic_y start_POSTSUBSCRIPT italic_n end_POSTSUBSCRIPT italic_s start_POSTSUBSCRIPT italic_n end_POSTSUBSCRIPT ) ≥ n>NBnλ0q(t)1|An|q(t)p(t)𝑑μ=n>N1|An|Bnλ0q(t)|An|1q(t)p(t)𝑑μsubscript𝑛𝑁subscriptsubscript𝐵𝑛superscriptsubscript𝜆0𝑞𝑡1superscriptsubscript𝐴𝑛𝑞𝑡𝑝𝑡differential-d𝜇subscript𝑛𝑁1subscript𝐴𝑛subscriptsubscript𝐵𝑛superscriptsubscript𝜆0𝑞𝑡superscriptsubscript𝐴𝑛1𝑞𝑡𝑝𝑡differential-d𝜇\displaystyle\sum_{n>N}\int_{B_{n}}\lambda_{0}^{q(t)}\frac{1}{|A_{n}|^{\frac{q% (t)}{p(t)}}}d\mu=\sum_{n>N}\frac{1}{|A_{n}|}\int_{B_{n}}\lambda_{0}^{q(t)}|A_{% n}|^{1-\frac{q(t)}{p(t)}}d\mu∑ start_POSTSUBSCRIPT italic_n > italic_N end_POSTSUBSCRIPT ∫ start_POSTSUBSCRIPT italic_B start_POSTSUBSCRIPT italic_n end_POSTSUBSCRIPT end_POSTSUBSCRIPT italic_λ start_POSTSUBSCRIPT 0 end_POSTSUBSCRIPT start_POSTSUPERSCRIPT italic_q ( italic_t ) end_POSTSUPERSCRIPT divide start_ARG 1 end_ARG start_ARG | italic_A start_POSTSUBSCRIPT italic_n end_POSTSUBSCRIPT | start_POSTSUPERSCRIPT divide start_ARG italic_q ( italic_t ) end_ARG start_ARG italic_p ( italic_t ) end_ARG end_POSTSUPERSCRIPT end_ARG italic_d italic_μ = ∑ start_POSTSUBSCRIPT italic_n > italic_N end_POSTSUBSCRIPT divide start_ARG 1 end_ARG start_ARG | italic_A start_POSTSUBSCRIPT italic_n end_POSTSUBSCRIPT | end_ARG ∫ start_POSTSUBSCRIPT italic_B start_POSTSUBSCRIPT italic_n end_POSTSUBSCRIPT end_POSTSUBSCRIPT italic_λ start_POSTSUBSCRIPT 0 end_POSTSUBSCRIPT start_POSTSUPERSCRIPT italic_q ( italic_t ) end_POSTSUPERSCRIPT | italic_A start_POSTSUBSCRIPT italic_n end_POSTSUBSCRIPT | start_POSTSUPERSCRIPT 1 - divide start_ARG italic_q ( italic_t ) end_ARG start_ARG italic_p ( italic_t ) end_ARG end_POSTSUPERSCRIPT italic_d italic_μ
=\displaystyle={}= n>N1|An|Bn[λ0|An|p(t)q(t)q(t)p(t)]q(t)𝑑μsubscript𝑛𝑁1subscript𝐴𝑛subscriptsubscript𝐵𝑛superscriptdelimited-[]subscript𝜆0superscriptsubscript𝐴𝑛𝑝𝑡𝑞𝑡𝑞𝑡𝑝𝑡𝑞𝑡differential-d𝜇\displaystyle\sum_{n>N}\frac{1}{|A_{n}|}\int_{B_{n}}\left[\lambda_{0}|A_{n}|^{% \frac{p(t)-q(t)}{q(t)p(t)}}\right]^{q(t)}d\mu∑ start_POSTSUBSCRIPT italic_n > italic_N end_POSTSUBSCRIPT divide start_ARG 1 end_ARG start_ARG | italic_A start_POSTSUBSCRIPT italic_n end_POSTSUBSCRIPT | end_ARG ∫ start_POSTSUBSCRIPT italic_B start_POSTSUBSCRIPT italic_n end_POSTSUBSCRIPT end_POSTSUBSCRIPT [ italic_λ start_POSTSUBSCRIPT 0 end_POSTSUBSCRIPT | italic_A start_POSTSUBSCRIPT italic_n end_POSTSUBSCRIPT | start_POSTSUPERSCRIPT divide start_ARG italic_p ( italic_t ) - italic_q ( italic_t ) end_ARG start_ARG italic_q ( italic_t ) italic_p ( italic_t ) end_ARG end_POSTSUPERSCRIPT ] start_POSTSUPERSCRIPT italic_q ( italic_t ) end_POSTSUPERSCRIPT italic_d italic_μ
and using ()(\diamond\diamond)( ⋄ ⋄ ) and ()(\diamond)( ⋄ ) we have
\displaystyle\geq{} n>N1|An|Bn[λ0(μ(Ω)xn2)p(t)q(t)q(t)p(t)]q(t)𝑑μsubscript𝑛𝑁1subscript𝐴𝑛subscriptsubscript𝐵𝑛superscriptdelimited-[]subscript𝜆0superscript𝜇Ωsubscript𝑥𝑛2𝑝𝑡𝑞𝑡𝑞𝑡𝑝𝑡𝑞𝑡differential-d𝜇\displaystyle\sum_{n>N}\frac{1}{|A_{n}|}\int_{B_{n}}\left[\lambda_{0}(\frac{% \mu(\Omega)-x_{n}}{2})^{\frac{p(t)-q(t)}{q(t)p(t)}}\right]^{q(t)}d\mu∑ start_POSTSUBSCRIPT italic_n > italic_N end_POSTSUBSCRIPT divide start_ARG 1 end_ARG start_ARG | italic_A start_POSTSUBSCRIPT italic_n end_POSTSUBSCRIPT | end_ARG ∫ start_POSTSUBSCRIPT italic_B start_POSTSUBSCRIPT italic_n end_POSTSUBSCRIPT end_POSTSUBSCRIPT [ italic_λ start_POSTSUBSCRIPT 0 end_POSTSUBSCRIPT ( divide start_ARG italic_μ ( roman_Ω ) - italic_x start_POSTSUBSCRIPT italic_n end_POSTSUBSCRIPT end_ARG start_ARG 2 end_ARG ) start_POSTSUPERSCRIPT divide start_ARG italic_p ( italic_t ) - italic_q ( italic_t ) end_ARG start_ARG italic_q ( italic_t ) italic_p ( italic_t ) end_ARG end_POSTSUPERSCRIPT ] start_POSTSUPERSCRIPT italic_q ( italic_t ) end_POSTSUPERSCRIPT italic_d italic_μ
\displaystyle\geq{} n>N1|An|Bn[λ0(μ(Ω)xn2)(p(t)q(t)q(t)p(t))|Bn+]q(t)𝑑μ\displaystyle\sum_{n>N}\frac{1}{|A_{n}|}\int_{B_{n}}\left[\lambda_{0}(\frac{% \mu(\Omega)-x_{n}}{2})^{(\frac{p(t)-q(t)}{q(t)p(t)})_{|_{B_{n}}}^{+}}\right]^{% q(t)}d\mu∑ start_POSTSUBSCRIPT italic_n > italic_N end_POSTSUBSCRIPT divide start_ARG 1 end_ARG start_ARG | italic_A start_POSTSUBSCRIPT italic_n end_POSTSUBSCRIPT | end_ARG ∫ start_POSTSUBSCRIPT italic_B start_POSTSUBSCRIPT italic_n end_POSTSUBSCRIPT end_POSTSUBSCRIPT [ italic_λ start_POSTSUBSCRIPT 0 end_POSTSUBSCRIPT ( divide start_ARG italic_μ ( roman_Ω ) - italic_x start_POSTSUBSCRIPT italic_n end_POSTSUBSCRIPT end_ARG start_ARG 2 end_ARG ) start_POSTSUPERSCRIPT ( divide start_ARG italic_p ( italic_t ) - italic_q ( italic_t ) end_ARG start_ARG italic_q ( italic_t ) italic_p ( italic_t ) end_ARG ) start_POSTSUBSCRIPT | start_POSTSUBSCRIPT italic_B start_POSTSUBSCRIPT italic_n end_POSTSUBSCRIPT end_POSTSUBSCRIPT end_POSTSUBSCRIPT start_POSTSUPERSCRIPT + end_POSTSUPERSCRIPT end_POSTSUPERSCRIPT ] start_POSTSUPERSCRIPT italic_q ( italic_t ) end_POSTSUPERSCRIPT italic_d italic_μ
\displaystyle\geq{} n>N1|An|Bn[λ0(μ(Ω)xn2)(pqqp)(xn)]q(t)𝑑μsubscript𝑛𝑁1subscript𝐴𝑛subscriptsubscript𝐵𝑛superscriptdelimited-[]subscript𝜆0superscript𝜇Ωsubscript𝑥𝑛2superscript𝑝𝑞𝑞𝑝subscript𝑥𝑛𝑞𝑡differential-d𝜇\displaystyle\sum_{n>N}\frac{1}{|A_{n}|}\int_{B_{n}}\left[\lambda_{0}(\frac{% \mu(\Omega)-x_{n}}{2})^{(\frac{p-q}{qp})^{*}(x_{n})}\right]^{q(t)}d\mu∑ start_POSTSUBSCRIPT italic_n > italic_N end_POSTSUBSCRIPT divide start_ARG 1 end_ARG start_ARG | italic_A start_POSTSUBSCRIPT italic_n end_POSTSUBSCRIPT | end_ARG ∫ start_POSTSUBSCRIPT italic_B start_POSTSUBSCRIPT italic_n end_POSTSUBSCRIPT end_POSTSUBSCRIPT [ italic_λ start_POSTSUBSCRIPT 0 end_POSTSUBSCRIPT ( divide start_ARG italic_μ ( roman_Ω ) - italic_x start_POSTSUBSCRIPT italic_n end_POSTSUBSCRIPT end_ARG start_ARG 2 end_ARG ) start_POSTSUPERSCRIPT ( divide start_ARG italic_p - italic_q end_ARG start_ARG italic_q italic_p end_ARG ) start_POSTSUPERSCRIPT ∗ end_POSTSUPERSCRIPT ( italic_x start_POSTSUBSCRIPT italic_n end_POSTSUBSCRIPT ) end_POSTSUPERSCRIPT ] start_POSTSUPERSCRIPT italic_q ( italic_t ) end_POSTSUPERSCRIPT italic_d italic_μ
\displaystyle\geq{} n>N1|An|Bn[λ0r12]q(t)𝑑μ=,subscript𝑛𝑁1subscript𝐴𝑛subscriptsubscript𝐵𝑛superscriptdelimited-[]subscript𝜆0𝑟12𝑞𝑡differential-d𝜇\displaystyle\sum_{n>N}\frac{1}{|A_{n}|}\int_{B_{n}}[\lambda_{0}r\frac{1}{2}]^% {q(t)}d\mu\,=\,\infty,∑ start_POSTSUBSCRIPT italic_n > italic_N end_POSTSUBSCRIPT divide start_ARG 1 end_ARG start_ARG | italic_A start_POSTSUBSCRIPT italic_n end_POSTSUBSCRIPT | end_ARG ∫ start_POSTSUBSCRIPT italic_B start_POSTSUBSCRIPT italic_n end_POSTSUBSCRIPT end_POSTSUBSCRIPT [ italic_λ start_POSTSUBSCRIPT 0 end_POSTSUBSCRIPT italic_r divide start_ARG 1 end_ARG start_ARG 2 end_ARG ] start_POSTSUPERSCRIPT italic_q ( italic_t ) end_POSTSUPERSCRIPT italic_d italic_μ = ∞ ,

which is a contradiction.

Now, using this fact and writing  λ=λλ0𝜆superscript𝜆subscript𝜆0\lambda=\frac{\lambda^{\prime}}{\lambda_{0}}italic_λ = divide start_ARG italic_λ start_POSTSUPERSCRIPT ′ end_POSTSUPERSCRIPT end_ARG start_ARG italic_λ start_POSTSUBSCRIPT 0 end_POSTSUBSCRIPT end_ARG, we have

ρp()(n>Nλynsn)subscript𝜌𝑝subscript𝑛𝑁𝜆subscript𝑦𝑛subscript𝑠𝑛\displaystyle\rho_{p(\cdot)}(\sum_{n>N}\lambda y_{n}s_{n})italic_ρ start_POSTSUBSCRIPT italic_p ( ⋅ ) end_POSTSUBSCRIPT ( ∑ start_POSTSUBSCRIPT italic_n > italic_N end_POSTSUBSCRIPT italic_λ italic_y start_POSTSUBSCRIPT italic_n end_POSTSUBSCRIPT italic_s start_POSTSUBSCRIPT italic_n end_POSTSUBSCRIPT ) =n>NBn|λλ0yn|q(t)|λλ0yn|pq(t)χBn|An|q(t)p(t)|An|1q(t)p(t)𝑑μabsentsubscript𝑛𝑁subscriptsubscript𝐵𝑛superscriptsuperscript𝜆subscript𝜆0subscript𝑦𝑛𝑞𝑡superscriptsuperscript𝜆subscript𝜆0subscript𝑦𝑛𝑝𝑞𝑡subscript𝜒subscript𝐵𝑛superscriptsubscript𝐴𝑛𝑞𝑡𝑝𝑡superscriptsubscript𝐴𝑛1𝑞𝑡𝑝𝑡differential-d𝜇\displaystyle=\sum_{n>N}\int_{B_{n}}|\frac{\lambda^{\prime}}{\lambda_{0}}y_{n}% |^{q(t)}|\frac{\lambda^{\prime}}{\lambda_{0}}y_{n}|^{p-q(t)}\frac{\chi_{B_{n}}% }{|A_{n}|^{\frac{q(t)}{p(t)}}|A_{n}|^{1-\frac{q(t)}{p(t)}}}d\mu= ∑ start_POSTSUBSCRIPT italic_n > italic_N end_POSTSUBSCRIPT ∫ start_POSTSUBSCRIPT italic_B start_POSTSUBSCRIPT italic_n end_POSTSUBSCRIPT end_POSTSUBSCRIPT | divide start_ARG italic_λ start_POSTSUPERSCRIPT ′ end_POSTSUPERSCRIPT end_ARG start_ARG italic_λ start_POSTSUBSCRIPT 0 end_POSTSUBSCRIPT end_ARG italic_y start_POSTSUBSCRIPT italic_n end_POSTSUBSCRIPT | start_POSTSUPERSCRIPT italic_q ( italic_t ) end_POSTSUPERSCRIPT | divide start_ARG italic_λ start_POSTSUPERSCRIPT ′ end_POSTSUPERSCRIPT end_ARG start_ARG italic_λ start_POSTSUBSCRIPT 0 end_POSTSUBSCRIPT end_ARG italic_y start_POSTSUBSCRIPT italic_n end_POSTSUBSCRIPT | start_POSTSUPERSCRIPT italic_p - italic_q ( italic_t ) end_POSTSUPERSCRIPT divide start_ARG italic_χ start_POSTSUBSCRIPT italic_B start_POSTSUBSCRIPT italic_n end_POSTSUBSCRIPT end_POSTSUBSCRIPT end_ARG start_ARG | italic_A start_POSTSUBSCRIPT italic_n end_POSTSUBSCRIPT | start_POSTSUPERSCRIPT divide start_ARG italic_q ( italic_t ) end_ARG start_ARG italic_p ( italic_t ) end_ARG end_POSTSUPERSCRIPT | italic_A start_POSTSUBSCRIPT italic_n end_POSTSUBSCRIPT | start_POSTSUPERSCRIPT 1 - divide start_ARG italic_q ( italic_t ) end_ARG start_ARG italic_p ( italic_t ) end_ARG end_POSTSUPERSCRIPT end_ARG italic_d italic_μ
n>NBn|λyn|q(t)χBn|An|q(t)p(t)[1λ0(|An|)p(t)q(t)p(t)q(t)]q(t)𝑑μabsentsubscript𝑛𝑁subscriptsubscript𝐵𝑛superscriptsuperscript𝜆subscript𝑦𝑛𝑞𝑡subscript𝜒subscript𝐵𝑛superscriptsubscript𝐴𝑛𝑞𝑡𝑝𝑡superscriptdelimited-[]1subscript𝜆0superscriptsubscript𝐴𝑛𝑝𝑡𝑞𝑡𝑝𝑡𝑞𝑡𝑞𝑡differential-d𝜇\displaystyle\leq\sum_{n>N}\int_{B_{n}}|\lambda^{\prime}y_{n}|^{q(t)}\frac{% \chi_{B_{n}}}{|A_{n}|^{\frac{q(t)}{p(t)}}}\left[\frac{1}{\lambda_{0}(|A_{n}|)^% {\frac{p(t)-q(t)}{p(t)q(t)}}}\right]^{q(t)}d\mu≤ ∑ start_POSTSUBSCRIPT italic_n > italic_N end_POSTSUBSCRIPT ∫ start_POSTSUBSCRIPT italic_B start_POSTSUBSCRIPT italic_n end_POSTSUBSCRIPT end_POSTSUBSCRIPT | italic_λ start_POSTSUPERSCRIPT ′ end_POSTSUPERSCRIPT italic_y start_POSTSUBSCRIPT italic_n end_POSTSUBSCRIPT | start_POSTSUPERSCRIPT italic_q ( italic_t ) end_POSTSUPERSCRIPT divide start_ARG italic_χ start_POSTSUBSCRIPT italic_B start_POSTSUBSCRIPT italic_n end_POSTSUBSCRIPT end_POSTSUBSCRIPT end_ARG start_ARG | italic_A start_POSTSUBSCRIPT italic_n end_POSTSUBSCRIPT | start_POSTSUPERSCRIPT divide start_ARG italic_q ( italic_t ) end_ARG start_ARG italic_p ( italic_t ) end_ARG end_POSTSUPERSCRIPT end_ARG [ divide start_ARG 1 end_ARG start_ARG italic_λ start_POSTSUBSCRIPT 0 end_POSTSUBSCRIPT ( | italic_A start_POSTSUBSCRIPT italic_n end_POSTSUBSCRIPT | ) start_POSTSUPERSCRIPT divide start_ARG italic_p ( italic_t ) - italic_q ( italic_t ) end_ARG start_ARG italic_p ( italic_t ) italic_q ( italic_t ) end_ARG end_POSTSUPERSCRIPT end_ARG ] start_POSTSUPERSCRIPT italic_q ( italic_t ) end_POSTSUPERSCRIPT italic_d italic_μ
n>NBn|λyn|q(t)χBn|An|q(t)p(t)[1λ0(|An|)(p(t)q(t)p(t)q(t))|Bn+]q(t)𝑑μ\displaystyle\leq\sum_{n>N}\int_{B_{n}}|\lambda^{\prime}y_{n}|^{q(t)}\frac{% \chi_{B_{n}}}{|A_{n}|^{\frac{q(t)}{p(t)}}}\left[\frac{1}{\lambda_{0}(|A_{n}|)^% {(\frac{p(t)-q(t)}{p(t)q(t)})^{+}_{|_{B_{n}}}}}\right]^{q(t)}d\mu≤ ∑ start_POSTSUBSCRIPT italic_n > italic_N end_POSTSUBSCRIPT ∫ start_POSTSUBSCRIPT italic_B start_POSTSUBSCRIPT italic_n end_POSTSUBSCRIPT end_POSTSUBSCRIPT | italic_λ start_POSTSUPERSCRIPT ′ end_POSTSUPERSCRIPT italic_y start_POSTSUBSCRIPT italic_n end_POSTSUBSCRIPT | start_POSTSUPERSCRIPT italic_q ( italic_t ) end_POSTSUPERSCRIPT divide start_ARG italic_χ start_POSTSUBSCRIPT italic_B start_POSTSUBSCRIPT italic_n end_POSTSUBSCRIPT end_POSTSUBSCRIPT end_ARG start_ARG | italic_A start_POSTSUBSCRIPT italic_n end_POSTSUBSCRIPT | start_POSTSUPERSCRIPT divide start_ARG italic_q ( italic_t ) end_ARG start_ARG italic_p ( italic_t ) end_ARG end_POSTSUPERSCRIPT end_ARG [ divide start_ARG 1 end_ARG start_ARG italic_λ start_POSTSUBSCRIPT 0 end_POSTSUBSCRIPT ( | italic_A start_POSTSUBSCRIPT italic_n end_POSTSUBSCRIPT | ) start_POSTSUPERSCRIPT ( divide start_ARG italic_p ( italic_t ) - italic_q ( italic_t ) end_ARG start_ARG italic_p ( italic_t ) italic_q ( italic_t ) end_ARG ) start_POSTSUPERSCRIPT + end_POSTSUPERSCRIPT start_POSTSUBSCRIPT | start_POSTSUBSCRIPT italic_B start_POSTSUBSCRIPT italic_n end_POSTSUBSCRIPT end_POSTSUBSCRIPT end_POSTSUBSCRIPT end_POSTSUPERSCRIPT end_ARG ] start_POSTSUPERSCRIPT italic_q ( italic_t ) end_POSTSUPERSCRIPT italic_d italic_μ
n>NBn|λyn|q(t)χBn|An|q(t)p(t)[1λ0(μ(Ω)xn2)(pqpq)(xn)]q(t)𝑑μabsentsubscript𝑛𝑁subscriptsubscript𝐵𝑛superscriptsuperscript𝜆subscript𝑦𝑛𝑞𝑡subscript𝜒subscript𝐵𝑛superscriptsubscript𝐴𝑛𝑞𝑡𝑝𝑡superscriptdelimited-[]1subscript𝜆0superscript𝜇Ωsubscript𝑥𝑛2superscript𝑝𝑞𝑝𝑞subscript𝑥𝑛𝑞𝑡differential-d𝜇\displaystyle\leq\sum_{n>N}\int_{B_{n}}|\lambda^{\prime}y_{n}|^{q(t)}\frac{% \chi_{B_{n}}}{|A_{n}|^{\frac{q(t)}{p(t)}}}\left[\frac{1}{\lambda_{0}(\frac{\mu% (\Omega)-x_{n}}{2})^{(\frac{p-q}{pq})^{*}(x_{n})}}\right]^{q(t)}d\mu≤ ∑ start_POSTSUBSCRIPT italic_n > italic_N end_POSTSUBSCRIPT ∫ start_POSTSUBSCRIPT italic_B start_POSTSUBSCRIPT italic_n end_POSTSUBSCRIPT end_POSTSUBSCRIPT | italic_λ start_POSTSUPERSCRIPT ′ end_POSTSUPERSCRIPT italic_y start_POSTSUBSCRIPT italic_n end_POSTSUBSCRIPT | start_POSTSUPERSCRIPT italic_q ( italic_t ) end_POSTSUPERSCRIPT divide start_ARG italic_χ start_POSTSUBSCRIPT italic_B start_POSTSUBSCRIPT italic_n end_POSTSUBSCRIPT end_POSTSUBSCRIPT end_ARG start_ARG | italic_A start_POSTSUBSCRIPT italic_n end_POSTSUBSCRIPT | start_POSTSUPERSCRIPT divide start_ARG italic_q ( italic_t ) end_ARG start_ARG italic_p ( italic_t ) end_ARG end_POSTSUPERSCRIPT end_ARG [ divide start_ARG 1 end_ARG start_ARG italic_λ start_POSTSUBSCRIPT 0 end_POSTSUBSCRIPT ( divide start_ARG italic_μ ( roman_Ω ) - italic_x start_POSTSUBSCRIPT italic_n end_POSTSUBSCRIPT end_ARG start_ARG 2 end_ARG ) start_POSTSUPERSCRIPT ( divide start_ARG italic_p - italic_q end_ARG start_ARG italic_p italic_q end_ARG ) start_POSTSUPERSCRIPT ∗ end_POSTSUPERSCRIPT ( italic_x start_POSTSUBSCRIPT italic_n end_POSTSUBSCRIPT ) end_POSTSUPERSCRIPT end_ARG ] start_POSTSUPERSCRIPT italic_q ( italic_t ) end_POSTSUPERSCRIPT italic_d italic_μ
n>NBn|λyn|q(t)χBn|An|q(t)p(t)[1λ0r(12)(pqpq)(xn)]q(t)𝑑μabsentsubscript𝑛𝑁subscriptsubscript𝐵𝑛superscriptsuperscript𝜆subscript𝑦𝑛𝑞𝑡subscript𝜒subscript𝐵𝑛superscriptsubscript𝐴𝑛𝑞𝑡𝑝𝑡superscriptdelimited-[]1subscript𝜆0𝑟superscript12superscript𝑝𝑞𝑝𝑞subscript𝑥𝑛𝑞𝑡differential-d𝜇\displaystyle\leq\sum_{n>N}\int_{B_{n}}|\lambda^{\prime}y_{n}|^{q(t)}\frac{% \chi_{B_{n}}}{|A_{n}|^{\frac{q(t)}{p(t)}}}\left[\frac{1}{\lambda_{0}r(\frac{1}% {2})^{(\frac{p-q}{pq})^{*}(x_{n})}}\right]^{q(t)}d\mu≤ ∑ start_POSTSUBSCRIPT italic_n > italic_N end_POSTSUBSCRIPT ∫ start_POSTSUBSCRIPT italic_B start_POSTSUBSCRIPT italic_n end_POSTSUBSCRIPT end_POSTSUBSCRIPT | italic_λ start_POSTSUPERSCRIPT ′ end_POSTSUPERSCRIPT italic_y start_POSTSUBSCRIPT italic_n end_POSTSUBSCRIPT | start_POSTSUPERSCRIPT italic_q ( italic_t ) end_POSTSUPERSCRIPT divide start_ARG italic_χ start_POSTSUBSCRIPT italic_B start_POSTSUBSCRIPT italic_n end_POSTSUBSCRIPT end_POSTSUBSCRIPT end_ARG start_ARG | italic_A start_POSTSUBSCRIPT italic_n end_POSTSUBSCRIPT | start_POSTSUPERSCRIPT divide start_ARG italic_q ( italic_t ) end_ARG start_ARG italic_p ( italic_t ) end_ARG end_POSTSUPERSCRIPT end_ARG [ divide start_ARG 1 end_ARG start_ARG italic_λ start_POSTSUBSCRIPT 0 end_POSTSUBSCRIPT italic_r ( divide start_ARG 1 end_ARG start_ARG 2 end_ARG ) start_POSTSUPERSCRIPT ( divide start_ARG italic_p - italic_q end_ARG start_ARG italic_p italic_q end_ARG ) start_POSTSUPERSCRIPT ∗ end_POSTSUPERSCRIPT ( italic_x start_POSTSUBSCRIPT italic_n end_POSTSUBSCRIPT ) end_POSTSUPERSCRIPT end_ARG ] start_POSTSUPERSCRIPT italic_q ( italic_t ) end_POSTSUPERSCRIPT italic_d italic_μ
ρq()(n>Nλynsn)N0.absentsubscript𝜌𝑞subscript𝑛𝑁superscript𝜆subscript𝑦𝑛subscript𝑠𝑛𝑁0\displaystyle\leq\rho_{q(\cdot)}(\sum_{n>N}\lambda^{\prime}y_{n}s_{n})% \xrightarrow{N\rightarrow\infty}0.≤ italic_ρ start_POSTSUBSCRIPT italic_q ( ⋅ ) end_POSTSUBSCRIPT ( ∑ start_POSTSUBSCRIPT italic_n > italic_N end_POSTSUBSCRIPT italic_λ start_POSTSUPERSCRIPT ′ end_POSTSUPERSCRIPT italic_y start_POSTSUBSCRIPT italic_n end_POSTSUBSCRIPT italic_s start_POSTSUBSCRIPT italic_n end_POSTSUBSCRIPT ) start_ARROW start_OVERACCENT italic_N → ∞ end_OVERACCENT → end_ARROW 0 .

This concludes the proof. ∎

Corollary 3.4.

Let  (Ω,μ)Ω𝜇(\Omega,\mu)( roman_Ω , italic_μ )  be an atomless finite measure space and exponents  q()<p()𝑞𝑝q(\cdot)<p(\cdot)italic_q ( ⋅ ) < italic_p ( ⋅ )μ𝜇\muitalic_μ-a.e. If the inclusion  Lp()(μ)Lq()(μ)superscript𝐿𝑝𝜇superscript𝐿𝑞𝜇{L^{p(\cdot)}(\mu)}\hookrightarrow{L^{q(\cdot)}(\mu)}italic_L start_POSTSUPERSCRIPT italic_p ( ⋅ ) end_POSTSUPERSCRIPT ( italic_μ ) ↪ italic_L start_POSTSUPERSCRIPT italic_q ( ⋅ ) end_POSTSUPERSCRIPT ( italic_μ )  is DSS, then

limxμ(Ω)(μ(Ω)x)(pqp)(x)=0.subscript𝑥𝜇superscriptΩsuperscript𝜇Ω𝑥superscript𝑝𝑞𝑝𝑥0\lim_{x\rightarrow\mu(\Omega)^{-}}\,(\mu(\Omega)-x)^{(\frac{p-q}{p})^{*}(x)}=0.roman_lim start_POSTSUBSCRIPT italic_x → italic_μ ( roman_Ω ) start_POSTSUPERSCRIPT - end_POSTSUPERSCRIPT end_POSTSUBSCRIPT ( italic_μ ( roman_Ω ) - italic_x ) start_POSTSUPERSCRIPT ( divide start_ARG italic_p - italic_q end_ARG start_ARG italic_p end_ARG ) start_POSTSUPERSCRIPT ∗ end_POSTSUPERSCRIPT ( italic_x ) end_POSTSUPERSCRIPT = 0 .

Moreover, if p+<superscript𝑝p^{+}<\inftyitalic_p start_POSTSUPERSCRIPT + end_POSTSUPERSCRIPT < ∞, then

limxμ(Ω)(μ(Ω)x)(pq)(x)=0.subscript𝑥𝜇superscriptΩsuperscript𝜇Ω𝑥superscript𝑝𝑞𝑥0\lim_{x\rightarrow\mu(\Omega)^{-}}\,(\mu(\Omega)-x)^{(p-q)^{*}(x)}=0.roman_lim start_POSTSUBSCRIPT italic_x → italic_μ ( roman_Ω ) start_POSTSUPERSCRIPT - end_POSTSUPERSCRIPT end_POSTSUBSCRIPT ( italic_μ ( roman_Ω ) - italic_x ) start_POSTSUPERSCRIPT ( italic_p - italic_q ) start_POSTSUPERSCRIPT ∗ end_POSTSUPERSCRIPT ( italic_x ) end_POSTSUPERSCRIPT = 0 .
Proof.

If follows from above proposition and when p+<superscript𝑝p^{+}<\inftyitalic_p start_POSTSUPERSCRIPT + end_POSTSUPERSCRIPT < ∞ using that   (pqp+)()(pqp)()(pqp)()𝑝𝑞superscript𝑝𝑝𝑞𝑝𝑝𝑞superscript𝑝(\frac{p-q}{p^{+}})(\cdot)\leq(\frac{p-q}{p})(\cdot)\leq(\frac{p-q}{p^{-}})(\cdot)( divide start_ARG italic_p - italic_q end_ARG start_ARG italic_p start_POSTSUPERSCRIPT + end_POSTSUPERSCRIPT end_ARG ) ( ⋅ ) ≤ ( divide start_ARG italic_p - italic_q end_ARG start_ARG italic_p end_ARG ) ( ⋅ ) ≤ ( divide start_ARG italic_p - italic_q end_ARG start_ARG italic_p start_POSTSUPERSCRIPT - end_POSTSUPERSCRIPT end_ARG ) ( ⋅ ). ∎

The converse of the above proposition will be proved later. We will need some basic Lemmas:

Lemma 3.5.

Let  (Ω,μ)Ω𝜇(\Omega,\mu)( roman_Ω , italic_μ ) be a finite measure space and f:Ω(0,):𝑓Ω0f:\Omega\rightarrow(0,\infty)italic_f : roman_Ω → ( 0 , ∞ ) be a measurable function. Then the functions 1/f1𝑓1/f1 / italic_f and 1/f1superscript𝑓1/f^{*}1 / italic_f start_POSTSUPERSCRIPT ∗ end_POSTSUPERSCRIPT are equi-distributed. Hence   (1/f)=(1/f)superscript1𝑓superscript1superscript𝑓(1/f)^{*}\,=\,(1/f^{*})^{*}( 1 / italic_f ) start_POSTSUPERSCRIPT ∗ end_POSTSUPERSCRIPT = ( 1 / italic_f start_POSTSUPERSCRIPT ∗ end_POSTSUPERSCRIPT ) start_POSTSUPERSCRIPT ∗ end_POSTSUPERSCRIPT.

Proof.

Let λ>0𝜆0\lambda>0italic_λ > 0. Since

μ1/f(λ)=subscript𝜇1𝑓𝜆absent\displaystyle\mu_{1/f}(\lambda)\,={}italic_μ start_POSTSUBSCRIPT 1 / italic_f end_POSTSUBSCRIPT ( italic_λ ) = μ({tΩ:1/f(t)>λ})=μ({tΩ:  1/λ>f(t)})𝜇conditional-set𝑡Ω1𝑓𝑡𝜆𝜇conditional-set𝑡Ω1𝜆𝑓𝑡\displaystyle\mu(\{t\in\Omega\,:1/f(t)>\lambda\,\})\,\,=\,\mu(\{t\in\Omega\,\,% :\,\,1/\lambda>f(t)\,\})italic_μ ( { italic_t ∈ roman_Ω : 1 / italic_f ( italic_t ) > italic_λ } ) = italic_μ ( { italic_t ∈ roman_Ω : 1 / italic_λ > italic_f ( italic_t ) } )
=\displaystyle={}= μ(Ω)μ({tΩ:f(t)1/λ})=μ(Ω)|{t[0,μ(Ω)]:f(t)1/λ}|𝜇Ω𝜇conditional-set𝑡Ω𝑓𝑡1𝜆𝜇Ωconditional-set𝑡0𝜇Ωsuperscript𝑓𝑡1𝜆\displaystyle\mu(\Omega)-\mu(\{t\in\Omega:\,f(t)\geq 1/\lambda\,\})=\mu(\Omega% )-|\{t\in[0,\mu(\Omega)]\,:\,f^{*}(t)\geq 1/\lambda\,\}|italic_μ ( roman_Ω ) - italic_μ ( { italic_t ∈ roman_Ω : italic_f ( italic_t ) ≥ 1 / italic_λ } ) = italic_μ ( roman_Ω ) - | { italic_t ∈ [ 0 , italic_μ ( roman_Ω ) ] : italic_f start_POSTSUPERSCRIPT ∗ end_POSTSUPERSCRIPT ( italic_t ) ≥ 1 / italic_λ } |
=\displaystyle={}= |{t[0,μ(Ω)]:f(t)<1/λ}|=|{t[0,μ(Ω)]: 1/f(t)>λ}|=||1/f(λ),\displaystyle|\,\{t\in[0,\mu(\Omega)]\,:\,f^{*}(t)<1/\lambda\,\}|=|\{t\in[0,% \mu(\Omega)]\,:\,1/f^{*}(t)>\lambda\,\}|=|\,\,|_{1/f^{*}}(\lambda),| { italic_t ∈ [ 0 , italic_μ ( roman_Ω ) ] : italic_f start_POSTSUPERSCRIPT ∗ end_POSTSUPERSCRIPT ( italic_t ) < 1 / italic_λ } | = | { italic_t ∈ [ 0 , italic_μ ( roman_Ω ) ] : 1 / italic_f start_POSTSUPERSCRIPT ∗ end_POSTSUPERSCRIPT ( italic_t ) > italic_λ } | = | | start_POSTSUBSCRIPT 1 / italic_f start_POSTSUPERSCRIPT ∗ end_POSTSUPERSCRIPT end_POSTSUBSCRIPT ( italic_λ ) ,

the functions  1/f1𝑓1/f1 / italic_f and 1/f1superscript𝑓1/f^{*}1 / italic_f start_POSTSUPERSCRIPT ∗ end_POSTSUPERSCRIPT are equi-distributed. ∎

Lemma 3.6.

If f:[0,μ(Ω)](0,):𝑓0𝜇Ω0f:[0,\mu(\Omega)]\rightarrow(0,\infty)italic_f : [ 0 , italic_μ ( roman_Ω ) ] → ( 0 , ∞ ) is an increasing measurable function then   f(t)=f(μ(Ω)t)superscript𝑓𝑡𝑓𝜇Ω𝑡f^{*}(t)=f(\mu(\Omega)-t)italic_f start_POSTSUPERSCRIPT ∗ end_POSTSUPERSCRIPT ( italic_t ) = italic_f ( italic_μ ( roman_Ω ) - italic_t ).

Proof.

Let λ>0𝜆0\lambda>0italic_λ > 0 and consider sλ:=inf{t[0,μ(Ω)]:f(t)>λ}assignsubscript𝑠𝜆infimumconditional-set𝑡0𝜇Ω𝑓𝑡𝜆s_{\lambda}:=\inf\{t\in[0,\mu(\Omega)]\,:\,f(t)>\lambda\,\}italic_s start_POSTSUBSCRIPT italic_λ end_POSTSUBSCRIPT := roman_inf { italic_t ∈ [ 0 , italic_μ ( roman_Ω ) ] : italic_f ( italic_t ) > italic_λ }. Since f𝑓fitalic_f is increasing we have   μf(λ)=μ(Ω)sλsubscript𝜇𝑓𝜆𝜇Ωsubscript𝑠𝜆\mu_{f}(\lambda)=\mu(\Omega)-s_{\lambda}italic_μ start_POSTSUBSCRIPT italic_f end_POSTSUBSCRIPT ( italic_λ ) = italic_μ ( roman_Ω ) - italic_s start_POSTSUBSCRIPT italic_λ end_POSTSUBSCRIPT. On the other hand, the function  h(t):=f(μ(Ω)t)assign𝑡𝑓𝜇Ω𝑡h(t):=f(\mu(\Omega)-t)italic_h ( italic_t ) := italic_f ( italic_μ ( roman_Ω ) - italic_t ) is decreasing and

{t[0,μ(Ω)]:h(t)>λ}=(0,μ(Ω)sλ).conditional-set𝑡0𝜇Ω𝑡𝜆0𝜇Ωsubscript𝑠𝜆\{t\in[0,\mu(\Omega)]\,\,:\,\,h(t)>\lambda\,\}=(0\,,\mu(\Omega)-s_{\lambda}).{ italic_t ∈ [ 0 , italic_μ ( roman_Ω ) ] : italic_h ( italic_t ) > italic_λ } = ( 0 , italic_μ ( roman_Ω ) - italic_s start_POSTSUBSCRIPT italic_λ end_POSTSUBSCRIPT ) .

Lemma 3.7.

Let  f:[0,b)(0,):𝑓0𝑏0f:[0,b)\rightarrow(0,\infty)italic_f : [ 0 , italic_b ) → ( 0 , ∞ )   be a decreasing measurable function with  f(0)>0𝑓00f(0)>0italic_f ( 0 ) > 0   and   limxbf(x)= 0subscript𝑥superscript𝑏𝑓𝑥 0\lim_{x\rightarrow b^{-}}\,f(x)=\,0roman_lim start_POSTSUBSCRIPT italic_x → italic_b start_POSTSUPERSCRIPT - end_POSTSUPERSCRIPT end_POSTSUBSCRIPT italic_f ( italic_x ) = 0. If   limxb(bx)f(x)=0subscript𝑥superscript𝑏superscript𝑏𝑥𝑓𝑥0\lim_{x\rightarrow b^{-}}\,(b-x)^{f(x)}=0roman_lim start_POSTSUBSCRIPT italic_x → italic_b start_POSTSUPERSCRIPT - end_POSTSUPERSCRIPT end_POSTSUBSCRIPT ( italic_b - italic_x ) start_POSTSUPERSCRIPT italic_f ( italic_x ) end_POSTSUPERSCRIPT = 0, then, for every   a>1𝑎1a>1italic_a > 1,

0ba1f(x)𝑑x<.superscriptsubscript0𝑏superscript𝑎1𝑓𝑥differential-d𝑥\int_{0}^{b}a^{\frac{1}{f(x)}}\,dx\,<\,\infty.∫ start_POSTSUBSCRIPT 0 end_POSTSUBSCRIPT start_POSTSUPERSCRIPT italic_b end_POSTSUPERSCRIPT italic_a start_POSTSUPERSCRIPT divide start_ARG 1 end_ARG start_ARG italic_f ( italic_x ) end_ARG end_POSTSUPERSCRIPT italic_d italic_x < ∞ .
Proof.

Given a>1𝑎1a>1italic_a > 1, consider a natural N𝑁Nitalic_N such that  a<eN𝑎superscript𝑒𝑁a<e^{N}italic_a < italic_e start_POSTSUPERSCRIPT italic_N end_POSTSUPERSCRIPT. Let us see 0beN/f(x)𝑑x<superscriptsubscript0𝑏superscript𝑒𝑁𝑓𝑥differential-d𝑥\int_{0}^{b}\,e^{N/f(x)}\,dx<\infty∫ start_POSTSUBSCRIPT 0 end_POSTSUBSCRIPT start_POSTSUPERSCRIPT italic_b end_POSTSUPERSCRIPT italic_e start_POSTSUPERSCRIPT italic_N / italic_f ( italic_x ) end_POSTSUPERSCRIPT italic_d italic_x < ∞.

From the hypothesis it follows that   limxbf(x)ln(bx)=subscript𝑥superscript𝑏𝑓𝑥𝑏𝑥\lim_{x\rightarrow b^{-}}-f(x)\ln(b-x)=\inftyroman_lim start_POSTSUBSCRIPT italic_x → italic_b start_POSTSUPERSCRIPT - end_POSTSUPERSCRIPT end_POSTSUBSCRIPT - italic_f ( italic_x ) roman_ln ( italic_b - italic_x ) = ∞ . Hence there exists 0<δN<10subscript𝛿𝑁10<\delta_{N}<10 < italic_δ start_POSTSUBSCRIPT italic_N end_POSTSUBSCRIPT < 1 such that   f(x)ln(bx)2N𝑓𝑥𝑏𝑥2𝑁-f(x)\ln(b-x)\geq 2N- italic_f ( italic_x ) roman_ln ( italic_b - italic_x ) ≥ 2 italic_N  for every x(bδN,b)𝑥𝑏subscript𝛿𝑁𝑏x\in(b-\delta_{N},b)italic_x ∈ ( italic_b - italic_δ start_POSTSUBSCRIPT italic_N end_POSTSUBSCRIPT , italic_b ), i.e. 12N1f(x)ln(bx)12𝑁1𝑓𝑥𝑏𝑥\,\frac{1}{2N}\geq\frac{1}{-f(x)\ln(b-x)}divide start_ARG 1 end_ARG start_ARG 2 italic_N end_ARG ≥ divide start_ARG 1 end_ARG start_ARG - italic_f ( italic_x ) roman_ln ( italic_b - italic_x ) end_ARG. Thus

0ba1/f(x)𝑑x0bδNa1/f(x)𝑑x+bδNb(eN)1/f(x)𝑑x<,superscriptsubscript0𝑏superscript𝑎1𝑓𝑥differential-d𝑥superscriptsubscript0𝑏subscript𝛿𝑁superscript𝑎1𝑓𝑥differential-d𝑥superscriptsubscript𝑏subscript𝛿𝑁𝑏superscriptsuperscript𝑒𝑁1𝑓𝑥differential-d𝑥\int_{0}^{b}\,a^{1/f(x)}dx\leq\,\int_{0}^{b-\delta_{N}}a^{1/f(x)}dx+\int_{b-% \delta_{N}}^{b}(e^{N})^{1/f(x)}dx\,<\,\infty,∫ start_POSTSUBSCRIPT 0 end_POSTSUBSCRIPT start_POSTSUPERSCRIPT italic_b end_POSTSUPERSCRIPT italic_a start_POSTSUPERSCRIPT 1 / italic_f ( italic_x ) end_POSTSUPERSCRIPT italic_d italic_x ≤ ∫ start_POSTSUBSCRIPT 0 end_POSTSUBSCRIPT start_POSTSUPERSCRIPT italic_b - italic_δ start_POSTSUBSCRIPT italic_N end_POSTSUBSCRIPT end_POSTSUPERSCRIPT italic_a start_POSTSUPERSCRIPT 1 / italic_f ( italic_x ) end_POSTSUPERSCRIPT italic_d italic_x + ∫ start_POSTSUBSCRIPT italic_b - italic_δ start_POSTSUBSCRIPT italic_N end_POSTSUBSCRIPT end_POSTSUBSCRIPT start_POSTSUPERSCRIPT italic_b end_POSTSUPERSCRIPT ( italic_e start_POSTSUPERSCRIPT italic_N end_POSTSUPERSCRIPT ) start_POSTSUPERSCRIPT 1 / italic_f ( italic_x ) end_POSTSUPERSCRIPT italic_d italic_x < ∞ ,

since

bδNb(eN)1/f(x)𝑑x=superscriptsubscript𝑏subscript𝛿𝑁𝑏superscriptsuperscript𝑒𝑁1𝑓𝑥differential-d𝑥absent\displaystyle\int_{b-\delta_{N}}^{b}(e^{N})^{1/f(x)}dx={}∫ start_POSTSUBSCRIPT italic_b - italic_δ start_POSTSUBSCRIPT italic_N end_POSTSUBSCRIPT end_POSTSUBSCRIPT start_POSTSUPERSCRIPT italic_b end_POSTSUPERSCRIPT ( italic_e start_POSTSUPERSCRIPT italic_N end_POSTSUPERSCRIPT ) start_POSTSUPERSCRIPT 1 / italic_f ( italic_x ) end_POSTSUPERSCRIPT italic_d italic_x = bδNb(eN)ln(bx)f(x)ln(bx)𝑑xbδNb(e)ln(bx)N(12N)𝑑x\displaystyle\int_{b-\delta_{N}}^{b}\left(e^{N}\right)^{\frac{-\ln(b-x)}{-f(x)% \ln(b-x)}}dx\leq\int_{b-\delta_{N}}^{b}\left(e\right)^{-\ln(b-x)^{N}(\frac{1}{% 2N})}dx∫ start_POSTSUBSCRIPT italic_b - italic_δ start_POSTSUBSCRIPT italic_N end_POSTSUBSCRIPT end_POSTSUBSCRIPT start_POSTSUPERSCRIPT italic_b end_POSTSUPERSCRIPT ( italic_e start_POSTSUPERSCRIPT italic_N end_POSTSUPERSCRIPT ) start_POSTSUPERSCRIPT divide start_ARG - roman_ln ( italic_b - italic_x ) end_ARG start_ARG - italic_f ( italic_x ) roman_ln ( italic_b - italic_x ) end_ARG end_POSTSUPERSCRIPT italic_d italic_x ≤ ∫ start_POSTSUBSCRIPT italic_b - italic_δ start_POSTSUBSCRIPT italic_N end_POSTSUBSCRIPT end_POSTSUBSCRIPT start_POSTSUPERSCRIPT italic_b end_POSTSUPERSCRIPT ( italic_e ) start_POSTSUPERSCRIPT - roman_ln ( italic_b - italic_x ) start_POSTSUPERSCRIPT italic_N end_POSTSUPERSCRIPT ( divide start_ARG 1 end_ARG start_ARG 2 italic_N end_ARG ) end_POSTSUPERSCRIPT italic_d italic_x
=\displaystyle={}= bδNb(1(bx)N)12N𝑑x=bδNb1bx𝑑x<.superscriptsubscript𝑏subscript𝛿𝑁𝑏superscript1superscript𝑏𝑥𝑁12𝑁differential-d𝑥superscriptsubscript𝑏subscript𝛿𝑁𝑏1𝑏𝑥differential-d𝑥\displaystyle\int_{b-\delta_{N}}^{b}\left(\frac{1}{(b-x)^{N}}\right)^{\frac{1}% {2N}}dx=\int_{b-\delta_{N}}^{b}\frac{1}{\sqrt{b-x}}dx\,\,<\infty.∫ start_POSTSUBSCRIPT italic_b - italic_δ start_POSTSUBSCRIPT italic_N end_POSTSUBSCRIPT end_POSTSUBSCRIPT start_POSTSUPERSCRIPT italic_b end_POSTSUPERSCRIPT ( divide start_ARG 1 end_ARG start_ARG ( italic_b - italic_x ) start_POSTSUPERSCRIPT italic_N end_POSTSUPERSCRIPT end_ARG ) start_POSTSUPERSCRIPT divide start_ARG 1 end_ARG start_ARG 2 italic_N end_ARG end_POSTSUPERSCRIPT italic_d italic_x = ∫ start_POSTSUBSCRIPT italic_b - italic_δ start_POSTSUBSCRIPT italic_N end_POSTSUBSCRIPT end_POSTSUBSCRIPT start_POSTSUPERSCRIPT italic_b end_POSTSUPERSCRIPT divide start_ARG 1 end_ARG start_ARG square-root start_ARG italic_b - italic_x end_ARG end_ARG italic_d italic_x < ∞ .

4. The finite measure case

In this section we give suitable criteria in terms of the exponents for the inclusions between variable exponent Lebesgue spaces over finite measure spaces be DSS. First we consider the case of inclusions Lp()(μ)Lq()(μ)superscript𝐿𝑝𝜇superscript𝐿𝑞𝜇{L^{p(\cdot)}(\mu)}\hookrightarrow{L^{q(\cdot)}(\mu)}italic_L start_POSTSUPERSCRIPT italic_p ( ⋅ ) end_POSTSUPERSCRIPT ( italic_μ ) ↪ italic_L start_POSTSUPERSCRIPT italic_q ( ⋅ ) end_POSTSUPERSCRIPT ( italic_μ ) when q()𝑞q(\cdot)italic_q ( ⋅ ) is a bounded exponent. After that, we will do the general case.

In particular we get the equivalence of the L𝐿Litalic_L-weak compactness and the DSS property for inclusions between variable Lebesgue spaces (recall that this equivalence does not happen in Orlicz spaces, see Section 2).

Theorem 4.1.

Let (Ω,μ)Ω𝜇(\Omega,\mu)( roman_Ω , italic_μ ) be an atomless finite measure space and exponents  q()<p()𝑞𝑝q(\cdot)<p(\cdot)italic_q ( ⋅ ) < italic_p ( ⋅ ) μ𝜇\muitalic_μ-a.e. with q+<superscript𝑞q^{+}<\inftyitalic_q start_POSTSUPERSCRIPT + end_POSTSUPERSCRIPT < ∞. Denote i𝑖iitalic_i the inclusion i:Lp()(μ)Lq()(μ):𝑖superscript𝐿𝑝𝜇superscript𝐿𝑞𝜇i:{L^{p(\cdot)}(\mu)}\hookrightarrow{L^{q(\cdot)}(\mu)}italic_i : italic_L start_POSTSUPERSCRIPT italic_p ( ⋅ ) end_POSTSUPERSCRIPT ( italic_μ ) ↪ italic_L start_POSTSUPERSCRIPT italic_q ( ⋅ ) end_POSTSUPERSCRIPT ( italic_μ ). TFAE:

  1. (1)

    0μ(Ω)a(ppq)(x)𝑑x<superscriptsubscript0𝜇Ωsuperscript𝑎superscript𝑝𝑝𝑞𝑥differential-d𝑥\int_{0}^{\mu(\Omega)}a^{(\frac{p}{p-q})^{*}(x)}dx<\infty∫ start_POSTSUBSCRIPT 0 end_POSTSUBSCRIPT start_POSTSUPERSCRIPT italic_μ ( roman_Ω ) end_POSTSUPERSCRIPT italic_a start_POSTSUPERSCRIPT ( divide start_ARG italic_p end_ARG start_ARG italic_p - italic_q end_ARG ) start_POSTSUPERSCRIPT ∗ end_POSTSUPERSCRIPT ( italic_x ) end_POSTSUPERSCRIPT italic_d italic_x < ∞     for every a>1𝑎1a>1italic_a > 1.

  2. (2)

    i𝑖iitalic_i is L𝐿Litalic_L-weakly compact.

  3. (3)

    i𝑖iitalic_i is M𝑀Mitalic_M-weakly compact.

  4. (4)

    i𝑖iitalic_i is DSS𝐷𝑆𝑆DSSitalic_D italic_S italic_S.

  5. (5)

    The restriction of the inclusion i𝑖iitalic_i on any subspace spanned by a disjoint sequence  (χEnμ(En)1p(t))subscript𝜒subscript𝐸𝑛𝜇superscriptsubscript𝐸𝑛1𝑝𝑡(\frac{\chi_{E_{n}}}{\mu(E_{n})^{\frac{1}{p(t)}}})( divide start_ARG italic_χ start_POSTSUBSCRIPT italic_E start_POSTSUBSCRIPT italic_n end_POSTSUBSCRIPT end_POSTSUBSCRIPT end_ARG start_ARG italic_μ ( italic_E start_POSTSUBSCRIPT italic_n end_POSTSUBSCRIPT ) start_POSTSUPERSCRIPT divide start_ARG 1 end_ARG start_ARG italic_p ( italic_t ) end_ARG end_POSTSUPERSCRIPT end_ARG )  is not an isomorphism.

  6. (6)

    limxμ(Ω)(μ(Ω)x)(pqp)(x)=0subscript𝑥𝜇superscriptΩsuperscript𝜇Ω𝑥superscript𝑝𝑞𝑝𝑥0\lim_{x\rightarrow\mu(\Omega)^{-}}\,\,(\mu(\Omega)-x)^{(\frac{p-q}{p})^{*}(x)}=0roman_lim start_POSTSUBSCRIPT italic_x → italic_μ ( roman_Ω ) start_POSTSUPERSCRIPT - end_POSTSUPERSCRIPT end_POSTSUBSCRIPT ( italic_μ ( roman_Ω ) - italic_x ) start_POSTSUPERSCRIPT ( divide start_ARG italic_p - italic_q end_ARG start_ARG italic_p end_ARG ) start_POSTSUPERSCRIPT ∗ end_POSTSUPERSCRIPT ( italic_x ) end_POSTSUPERSCRIPT = 0.

Proof.

(1)(2)12(1)\Rightarrow(2)( 1 ) ⇒ ( 2 ) It is similar to Lemma 3.3 in [9]. Suppose that (1)1(1)( 1 ) is true and (2)2(2)( 2 ) is not. Hence

limnsupfBLp()(μ){fχAnq()}0subscript𝑛subscriptsupremum𝑓subscript𝐵superscript𝐿𝑝𝜇subscriptnorm𝑓subscript𝜒subscript𝐴𝑛𝑞0\lim_{n\rightarrow\infty}\sup_{f\in B_{{L^{p(\cdot)}(\mu)}}}\{||f\chi_{A_{n}}|% |_{q(\cdot)}\}\neq 0roman_lim start_POSTSUBSCRIPT italic_n → ∞ end_POSTSUBSCRIPT roman_sup start_POSTSUBSCRIPT italic_f ∈ italic_B start_POSTSUBSCRIPT italic_L start_POSTSUPERSCRIPT italic_p ( ⋅ ) end_POSTSUPERSCRIPT ( italic_μ ) end_POSTSUBSCRIPT end_POSTSUBSCRIPT { | | italic_f italic_χ start_POSTSUBSCRIPT italic_A start_POSTSUBSCRIPT italic_n end_POSTSUBSCRIPT end_POSTSUBSCRIPT | | start_POSTSUBSCRIPT italic_q ( ⋅ ) end_POSTSUBSCRIPT } ≠ 0

for certain sequence (An)subscript𝐴𝑛(A_{n})( italic_A start_POSTSUBSCRIPT italic_n end_POSTSUBSCRIPT ) in ΩΩ\Omegaroman_Ω  such that χAn0subscript𝜒subscript𝐴𝑛0\chi_{A_{n}}\rightarrow 0italic_χ start_POSTSUBSCRIPT italic_A start_POSTSUBSCRIPT italic_n end_POSTSUBSCRIPT end_POSTSUBSCRIPT → 0 μ𝜇\muitalic_μ-a.e.. Then there exist   0<δ<10𝛿10<\delta<10 < italic_δ < 1, a sequence (fk)kBLp()(μ)subscriptsubscript𝑓𝑘𝑘subscript𝐵superscript𝐿𝑝𝜇(f_{k})_{k}\subset B_{{L^{p(\cdot)}(\mu)}}( italic_f start_POSTSUBSCRIPT italic_k end_POSTSUBSCRIPT ) start_POSTSUBSCRIPT italic_k end_POSTSUBSCRIPT ⊂ italic_B start_POSTSUBSCRIPT italic_L start_POSTSUPERSCRIPT italic_p ( ⋅ ) end_POSTSUPERSCRIPT ( italic_μ ) end_POSTSUBSCRIPT and a subsequence (Ank)ksubscriptsubscript𝐴subscript𝑛𝑘𝑘(A_{n_{k}})_{k}( italic_A start_POSTSUBSCRIPT italic_n start_POSTSUBSCRIPT italic_k end_POSTSUBSCRIPT end_POSTSUBSCRIPT ) start_POSTSUBSCRIPT italic_k end_POSTSUBSCRIPT such that

fkχAnkq()>δsubscriptnormsubscript𝑓𝑘subscript𝜒subscript𝐴subscript𝑛𝑘𝑞𝛿||f_{k}\chi_{A_{n_{k}}}||_{q(\cdot)}\,>\,\delta| | italic_f start_POSTSUBSCRIPT italic_k end_POSTSUBSCRIPT italic_χ start_POSTSUBSCRIPT italic_A start_POSTSUBSCRIPT italic_n start_POSTSUBSCRIPT italic_k end_POSTSUBSCRIPT end_POSTSUBSCRIPT end_POSTSUBSCRIPT | | start_POSTSUBSCRIPT italic_q ( ⋅ ) end_POSTSUBSCRIPT > italic_δ

for every natural k𝑘kitalic_k. This implies

1<fkχAnkδq()ρq()(fkχAnkδ)1δq+ρq()(fkχAnk).1subscriptnormsubscript𝑓𝑘subscript𝜒subscript𝐴subscript𝑛𝑘𝛿𝑞subscript𝜌𝑞subscript𝑓𝑘subscript𝜒subscript𝐴subscript𝑛𝑘𝛿1superscript𝛿superscript𝑞subscript𝜌𝑞subscript𝑓𝑘subscript𝜒subscript𝐴subscript𝑛𝑘1<\|\frac{f_{k}\chi_{A_{n_{k}}}}{\delta}\|_{q(\cdot)}\leq\rho_{q(\cdot)}(\frac% {f_{k}\chi_{A_{n_{k}}}}{\delta})\,\leq\,\frac{1}{\delta^{q^{+}}}\,\rho_{q(% \cdot)}(f_{k}\chi_{A_{n_{k}}}).1 < ∥ divide start_ARG italic_f start_POSTSUBSCRIPT italic_k end_POSTSUBSCRIPT italic_χ start_POSTSUBSCRIPT italic_A start_POSTSUBSCRIPT italic_n start_POSTSUBSCRIPT italic_k end_POSTSUBSCRIPT end_POSTSUBSCRIPT end_POSTSUBSCRIPT end_ARG start_ARG italic_δ end_ARG ∥ start_POSTSUBSCRIPT italic_q ( ⋅ ) end_POSTSUBSCRIPT ≤ italic_ρ start_POSTSUBSCRIPT italic_q ( ⋅ ) end_POSTSUBSCRIPT ( divide start_ARG italic_f start_POSTSUBSCRIPT italic_k end_POSTSUBSCRIPT italic_χ start_POSTSUBSCRIPT italic_A start_POSTSUBSCRIPT italic_n start_POSTSUBSCRIPT italic_k end_POSTSUBSCRIPT end_POSTSUBSCRIPT end_POSTSUBSCRIPT end_ARG start_ARG italic_δ end_ARG ) ≤ divide start_ARG 1 end_ARG start_ARG italic_δ start_POSTSUPERSCRIPT italic_q start_POSTSUPERSCRIPT + end_POSTSUPERSCRIPT end_POSTSUPERSCRIPT end_ARG italic_ρ start_POSTSUBSCRIPT italic_q ( ⋅ ) end_POSTSUBSCRIPT ( italic_f start_POSTSUBSCRIPT italic_k end_POSTSUBSCRIPT italic_χ start_POSTSUBSCRIPT italic_A start_POSTSUBSCRIPT italic_n start_POSTSUBSCRIPT italic_k end_POSTSUBSCRIPT end_POSTSUBSCRIPT end_POSTSUBSCRIPT ) .

Hence,

δq+<ρq()(fkχAnk)=Ω|fkχAnk(t)|q(t)𝑑μsuperscript𝛿superscript𝑞subscript𝜌𝑞subscript𝑓𝑘subscript𝜒subscript𝐴subscript𝑛𝑘subscriptΩsuperscriptsubscript𝑓𝑘subscript𝜒subscript𝐴subscript𝑛𝑘𝑡𝑞𝑡differential-d𝜇\delta^{q^{+}}\,<\,\rho_{q(\cdot)}(f_{k}\chi_{A_{n_{k}}})=\int_{\Omega}|f_{k}% \chi_{A_{n_{k}}}(t)|^{q(t)}d\muitalic_δ start_POSTSUPERSCRIPT italic_q start_POSTSUPERSCRIPT + end_POSTSUPERSCRIPT end_POSTSUPERSCRIPT < italic_ρ start_POSTSUBSCRIPT italic_q ( ⋅ ) end_POSTSUBSCRIPT ( italic_f start_POSTSUBSCRIPT italic_k end_POSTSUBSCRIPT italic_χ start_POSTSUBSCRIPT italic_A start_POSTSUBSCRIPT italic_n start_POSTSUBSCRIPT italic_k end_POSTSUBSCRIPT end_POSTSUBSCRIPT end_POSTSUBSCRIPT ) = ∫ start_POSTSUBSCRIPT roman_Ω end_POSTSUBSCRIPT | italic_f start_POSTSUBSCRIPT italic_k end_POSTSUBSCRIPT italic_χ start_POSTSUBSCRIPT italic_A start_POSTSUBSCRIPT italic_n start_POSTSUBSCRIPT italic_k end_POSTSUBSCRIPT end_POSTSUBSCRIPT end_POSTSUBSCRIPT ( italic_t ) | start_POSTSUPERSCRIPT italic_q ( italic_t ) end_POSTSUPERSCRIPT italic_d italic_μ

for every k𝑘kitalic_k. Now considering the exponent  r(t):=p(t)q(t)>1assign𝑟𝑡𝑝𝑡𝑞𝑡1r(t):=\frac{p(t)}{q(t)}>1italic_r ( italic_t ) := divide start_ARG italic_p ( italic_t ) end_ARG start_ARG italic_q ( italic_t ) end_ARG > 1μ𝜇\muitalic_μ-a.e. and its conjugate  r(t)=p(t)p(t)q(t)superscript𝑟𝑡𝑝𝑡𝑝𝑡𝑞𝑡r^{\prime}(t)=\frac{p(t)}{p(t)-q(t)}italic_r start_POSTSUPERSCRIPT ′ end_POSTSUPERSCRIPT ( italic_t ) = divide start_ARG italic_p ( italic_t ) end_ARG start_ARG italic_p ( italic_t ) - italic_q ( italic_t ) end_ARG we have, by Hölder inequality, that

ρq()(fkχAnk)KχAnkr()fkq()r().subscript𝜌𝑞subscript𝑓𝑘subscript𝜒subscript𝐴subscript𝑛𝑘𝐾subscriptnormsubscript𝜒𝐴subscript𝑛𝑘superscript𝑟subscriptnormsuperscriptsubscript𝑓𝑘𝑞𝑟\rho_{q(\cdot)}(f_{k}\chi_{A_{n_{k}}})\leq\,K\,||\,\chi_{An_{k}}||_{r^{\prime}% (\cdot)}\,\,||f_{k}^{q(\cdot)}||_{r(\cdot)}.italic_ρ start_POSTSUBSCRIPT italic_q ( ⋅ ) end_POSTSUBSCRIPT ( italic_f start_POSTSUBSCRIPT italic_k end_POSTSUBSCRIPT italic_χ start_POSTSUBSCRIPT italic_A start_POSTSUBSCRIPT italic_n start_POSTSUBSCRIPT italic_k end_POSTSUBSCRIPT end_POSTSUBSCRIPT end_POSTSUBSCRIPT ) ≤ italic_K | | italic_χ start_POSTSUBSCRIPT italic_A italic_n start_POSTSUBSCRIPT italic_k end_POSTSUBSCRIPT end_POSTSUBSCRIPT | | start_POSTSUBSCRIPT italic_r start_POSTSUPERSCRIPT ′ end_POSTSUPERSCRIPT ( ⋅ ) end_POSTSUBSCRIPT | | italic_f start_POSTSUBSCRIPT italic_k end_POSTSUBSCRIPT start_POSTSUPERSCRIPT italic_q ( ⋅ ) end_POSTSUPERSCRIPT | | start_POSTSUBSCRIPT italic_r ( ⋅ ) end_POSTSUBSCRIPT .

And, since   fkp()1subscriptnormsubscript𝑓𝑘𝑝1||\,f_{k}\,||_{p(\cdot)}\leq 1| | italic_f start_POSTSUBSCRIPT italic_k end_POSTSUBSCRIPT | | start_POSTSUBSCRIPT italic_p ( ⋅ ) end_POSTSUBSCRIPT ≤ 1, we have   fkq()r()1subscriptnormsuperscriptsubscript𝑓𝑘𝑞𝑟1||\,f_{k}^{q(\cdot)}\,||_{r(\cdot)}\leq 1| | italic_f start_POSTSUBSCRIPT italic_k end_POSTSUBSCRIPT start_POSTSUPERSCRIPT italic_q ( ⋅ ) end_POSTSUPERSCRIPT | | start_POSTSUBSCRIPT italic_r ( ⋅ ) end_POSTSUBSCRIPT ≤ 1. Indeed, if ρp()(fλ)1subscript𝜌𝑝𝑓𝜆1\rho_{p(\cdot)}(\frac{f}{\lambda})\leq 1italic_ρ start_POSTSUBSCRIPT italic_p ( ⋅ ) end_POSTSUBSCRIPT ( divide start_ARG italic_f end_ARG start_ARG italic_λ end_ARG ) ≤ 1 for every λ>1𝜆1\lambda>1italic_λ > 1, then

ρr()(fqλ)Ω|f(t)|p(t)(λ1/q+)p(t)𝑑μ1.subscript𝜌𝑟superscript𝑓𝑞𝜆subscriptΩsuperscript𝑓𝑡𝑝𝑡superscriptsuperscript𝜆1superscript𝑞𝑝𝑡differential-d𝜇1\rho_{r(\cdot)}\left(\frac{f^{q}}{\lambda}\right)\leq\int_{\Omega}\frac{|f(t)|% ^{p(t)}}{(\lambda^{1/q^{+}})^{p(t)}}d\mu\leq 1.italic_ρ start_POSTSUBSCRIPT italic_r ( ⋅ ) end_POSTSUBSCRIPT ( divide start_ARG italic_f start_POSTSUPERSCRIPT italic_q end_POSTSUPERSCRIPT end_ARG start_ARG italic_λ end_ARG ) ≤ ∫ start_POSTSUBSCRIPT roman_Ω end_POSTSUBSCRIPT divide start_ARG | italic_f ( italic_t ) | start_POSTSUPERSCRIPT italic_p ( italic_t ) end_POSTSUPERSCRIPT end_ARG start_ARG ( italic_λ start_POSTSUPERSCRIPT 1 / italic_q start_POSTSUPERSCRIPT + end_POSTSUPERSCRIPT end_POSTSUPERSCRIPT ) start_POSTSUPERSCRIPT italic_p ( italic_t ) end_POSTSUPERSCRIPT end_ARG italic_d italic_μ ≤ 1 .

Thus,

δq+KχAnkr().superscript𝛿superscript𝑞𝐾subscriptnormsubscript𝜒𝐴subscript𝑛𝑘superscript𝑟\delta^{q^{+}}\leq\,K\,||\chi_{An_{k}}||_{r^{\prime}(\cdot)}.italic_δ start_POSTSUPERSCRIPT italic_q start_POSTSUPERSCRIPT + end_POSTSUPERSCRIPT end_POSTSUPERSCRIPT ≤ italic_K | | italic_χ start_POSTSUBSCRIPT italic_A italic_n start_POSTSUBSCRIPT italic_k end_POSTSUBSCRIPT end_POSTSUBSCRIPT | | start_POSTSUBSCRIPT italic_r start_POSTSUPERSCRIPT ′ end_POSTSUPERSCRIPT ( ⋅ ) end_POSTSUBSCRIPT .

But by hypotheses (1)1(1)( 1 ) and Proposition 3.2 we get  χAnkr()0subscriptnormsubscript𝜒𝐴subscript𝑛𝑘superscript𝑟0||\chi_{An_{k}}||_{r^{\prime}(\cdot)}\rightarrow 0| | italic_χ start_POSTSUBSCRIPT italic_A italic_n start_POSTSUBSCRIPT italic_k end_POSTSUBSCRIPT end_POSTSUBSCRIPT | | start_POSTSUBSCRIPT italic_r start_POSTSUPERSCRIPT ′ end_POSTSUPERSCRIPT ( ⋅ ) end_POSTSUBSCRIPT → 0, which is a contradiction.

(2)(3)23(2)\Rightarrow(3)( 2 ) ⇒ ( 3 ) Let  (fn)subscript𝑓𝑛(f_{n})( italic_f start_POSTSUBSCRIPT italic_n end_POSTSUBSCRIPT )  be a pairwise disjoint normalized sequence in Lp()(μ)superscript𝐿𝑝𝜇{L^{p(\cdot)}(\mu)}italic_L start_POSTSUPERSCRIPT italic_p ( ⋅ ) end_POSTSUPERSCRIPT ( italic_μ ). As  μ(Ω)<𝜇Ω\mu(\Omega)<\inftyitalic_μ ( roman_Ω ) < ∞  we have  μ(supp(fn))0𝜇𝑠𝑢𝑝𝑝subscript𝑓𝑛0\mu(supp(f_{n}))\rightarrow 0italic_μ ( italic_s italic_u italic_p italic_p ( italic_f start_POSTSUBSCRIPT italic_n end_POSTSUBSCRIPT ) ) → 0 as n𝑛n\rightarrow\inftyitalic_n → ∞, hence

limnfnq()limnsupkfkχsupp(fn)q()limμ(A)0supkfkχAq()=0.subscript𝑛subscriptnormsubscript𝑓𝑛𝑞subscriptmaps-to𝑛subscriptsupremum𝑘subscriptnormsubscript𝑓𝑘subscript𝜒𝑠𝑢𝑝𝑝subscript𝑓𝑛𝑞subscript𝜇𝐴0subscriptsupremum𝑘subscriptnormsubscript𝑓𝑘subscript𝜒𝐴𝑞0\lim_{n\rightarrow\infty}\|f_{n}\|_{q(\cdot)}\leq\lim_{n\mapsto\infty}\sup_{k}% \,\|f_{k}\chi_{supp(f_{n})}\|_{q(\cdot)}\,\leq\,\lim_{\mu(A)\rightarrow 0}\sup% _{k}\,\|f_{k}\chi_{A}\|_{q(\cdot)}=0.roman_lim start_POSTSUBSCRIPT italic_n → ∞ end_POSTSUBSCRIPT ∥ italic_f start_POSTSUBSCRIPT italic_n end_POSTSUBSCRIPT ∥ start_POSTSUBSCRIPT italic_q ( ⋅ ) end_POSTSUBSCRIPT ≤ roman_lim start_POSTSUBSCRIPT italic_n ↦ ∞ end_POSTSUBSCRIPT roman_sup start_POSTSUBSCRIPT italic_k end_POSTSUBSCRIPT ∥ italic_f start_POSTSUBSCRIPT italic_k end_POSTSUBSCRIPT italic_χ start_POSTSUBSCRIPT italic_s italic_u italic_p italic_p ( italic_f start_POSTSUBSCRIPT italic_n end_POSTSUBSCRIPT ) end_POSTSUBSCRIPT ∥ start_POSTSUBSCRIPT italic_q ( ⋅ ) end_POSTSUBSCRIPT ≤ roman_lim start_POSTSUBSCRIPT italic_μ ( italic_A ) → 0 end_POSTSUBSCRIPT roman_sup start_POSTSUBSCRIPT italic_k end_POSTSUBSCRIPT ∥ italic_f start_POSTSUBSCRIPT italic_k end_POSTSUBSCRIPT italic_χ start_POSTSUBSCRIPT italic_A end_POSTSUBSCRIPT ∥ start_POSTSUBSCRIPT italic_q ( ⋅ ) end_POSTSUBSCRIPT = 0 .

It is clear that (3)(4)(5)345(3)\Rightarrow(4)\Rightarrow(5)( 3 ) ⇒ ( 4 ) ⇒ ( 5 ).

(5)(6)56(5)\Rightarrow(6)( 5 ) ⇒ ( 6 ) It is the proof of Proposition 3.3 (and Corollary 3.4)

(6)(1)61(6)\Rightarrow(1)( 6 ) ⇒ ( 1 ) Consider the function  f(t)=(pqp)(t)𝑓𝑡𝑝𝑞𝑝𝑡f(t)=(\frac{p-q}{p})(t)italic_f ( italic_t ) = ( divide start_ARG italic_p - italic_q end_ARG start_ARG italic_p end_ARG ) ( italic_t ). From Lemma 3.5, we have that the functions   1(pqp)(x)1superscript𝑝𝑞𝑝𝑥\frac{1}{(\frac{p-q}{p})^{*}(x)}divide start_ARG 1 end_ARG start_ARG ( divide start_ARG italic_p - italic_q end_ARG start_ARG italic_p end_ARG ) start_POSTSUPERSCRIPT ∗ end_POSTSUPERSCRIPT ( italic_x ) end_ARG   and   1(pqp)(t)=(ppq)(t)1𝑝𝑞𝑝𝑡𝑝𝑝𝑞𝑡\frac{1}{(\frac{p-q}{p})(t)}=(\frac{p}{p-q})(t)divide start_ARG 1 end_ARG start_ARG ( divide start_ARG italic_p - italic_q end_ARG start_ARG italic_p end_ARG ) ( italic_t ) end_ARG = ( divide start_ARG italic_p end_ARG start_ARG italic_p - italic_q end_ARG ) ( italic_t )  are equi-distributed. Thus   (ppq)=(1/f)superscript𝑝𝑝𝑞superscript1superscript𝑓(\frac{p}{p-q})^{*}=(1/f^{*})^{*}( divide start_ARG italic_p end_ARG start_ARG italic_p - italic_q end_ARG ) start_POSTSUPERSCRIPT ∗ end_POSTSUPERSCRIPT = ( 1 / italic_f start_POSTSUPERSCRIPT ∗ end_POSTSUPERSCRIPT ) start_POSTSUPERSCRIPT ∗ end_POSTSUPERSCRIPT  and

0μ(Ω)a(ppq)(x)𝑑x=0μ(Ω)a(1(pqp))(x)𝑑x,superscriptsubscript0𝜇Ωsuperscript𝑎superscript𝑝𝑝𝑞𝑥differential-d𝑥superscriptsubscript0𝜇Ωsuperscript𝑎superscript1superscript𝑝𝑞𝑝𝑥differential-d𝑥\int_{0}^{\mu(\Omega)}a^{\left(\frac{p}{p-q}\right)^{*}(x)}dx=\int_{0}^{\mu(% \Omega)}a^{\left(\frac{1}{(\frac{p-q}{p})^{*}}\right)^{*}(x)}dx,∫ start_POSTSUBSCRIPT 0 end_POSTSUBSCRIPT start_POSTSUPERSCRIPT italic_μ ( roman_Ω ) end_POSTSUPERSCRIPT italic_a start_POSTSUPERSCRIPT ( divide start_ARG italic_p end_ARG start_ARG italic_p - italic_q end_ARG ) start_POSTSUPERSCRIPT ∗ end_POSTSUPERSCRIPT ( italic_x ) end_POSTSUPERSCRIPT italic_d italic_x = ∫ start_POSTSUBSCRIPT 0 end_POSTSUBSCRIPT start_POSTSUPERSCRIPT italic_μ ( roman_Ω ) end_POSTSUPERSCRIPT italic_a start_POSTSUPERSCRIPT ( divide start_ARG 1 end_ARG start_ARG ( divide start_ARG italic_p - italic_q end_ARG start_ARG italic_p end_ARG ) start_POSTSUPERSCRIPT ∗ end_POSTSUPERSCRIPT end_ARG ) start_POSTSUPERSCRIPT ∗ end_POSTSUPERSCRIPT ( italic_x ) end_POSTSUPERSCRIPT italic_d italic_x ,

Now, as 1/f1superscript𝑓1/f^{*}1 / italic_f start_POSTSUPERSCRIPT ∗ end_POSTSUPERSCRIPT is an increasing function, it follows by Lemma 3.6 that  (1/f)(x)=(1/f)(x)=(1/f)(μ(Ω)x)superscript1𝑓𝑥superscript1superscript𝑓𝑥1superscript𝑓𝜇Ω𝑥(1/f)^{*}(x)=(1/f^{*})^{*}(x)=(1/f^{*})(\mu(\Omega)-x)( 1 / italic_f ) start_POSTSUPERSCRIPT ∗ end_POSTSUPERSCRIPT ( italic_x ) = ( 1 / italic_f start_POSTSUPERSCRIPT ∗ end_POSTSUPERSCRIPT ) start_POSTSUPERSCRIPT ∗ end_POSTSUPERSCRIPT ( italic_x ) = ( 1 / italic_f start_POSTSUPERSCRIPT ∗ end_POSTSUPERSCRIPT ) ( italic_μ ( roman_Ω ) - italic_x )  so we have

=0μ(Ω)a1(pqp)(μ(Ω)x)𝑑x=0μ(Ω)a1(pqp)(y)𝑑y.absentsuperscriptsubscript0𝜇Ωsuperscript𝑎1superscript𝑝𝑞𝑝𝜇Ω𝑥differential-d𝑥superscriptsubscript0𝜇Ωsuperscript𝑎1superscript𝑝𝑞𝑝𝑦differential-d𝑦=\,\int_{0}^{\mu(\Omega)}a^{\frac{1}{(\frac{p-q}{p})^{*}}(\mu(\Omega)-x)}dx\,=% \,\int_{0}^{\mu(\Omega)}a^{\frac{1}{(\frac{p-q}{p})^{*}(y)}}dy.= ∫ start_POSTSUBSCRIPT 0 end_POSTSUBSCRIPT start_POSTSUPERSCRIPT italic_μ ( roman_Ω ) end_POSTSUPERSCRIPT italic_a start_POSTSUPERSCRIPT divide start_ARG 1 end_ARG start_ARG ( divide start_ARG italic_p - italic_q end_ARG start_ARG italic_p end_ARG ) start_POSTSUPERSCRIPT ∗ end_POSTSUPERSCRIPT end_ARG ( italic_μ ( roman_Ω ) - italic_x ) end_POSTSUPERSCRIPT italic_d italic_x = ∫ start_POSTSUBSCRIPT 0 end_POSTSUBSCRIPT start_POSTSUPERSCRIPT italic_μ ( roman_Ω ) end_POSTSUPERSCRIPT italic_a start_POSTSUPERSCRIPT divide start_ARG 1 end_ARG start_ARG ( divide start_ARG italic_p - italic_q end_ARG start_ARG italic_p end_ARG ) start_POSTSUPERSCRIPT ∗ end_POSTSUPERSCRIPT ( italic_y ) end_ARG end_POSTSUPERSCRIPT italic_d italic_y .

Finally the boundedness of this integral follows from Lemma 3.7 for μ(Ω)=b𝜇Ω𝑏\mu(\Omega)=bitalic_μ ( roman_Ω ) = italic_b, since   (pqp)()superscript𝑝𝑞𝑝(\frac{p-q}{p})^{*}(\cdot)( divide start_ARG italic_p - italic_q end_ARG start_ARG italic_p end_ARG ) start_POSTSUPERSCRIPT ∗ end_POSTSUPERSCRIPT ( ⋅ ) is decreasing and the hypothesis. ∎

It is clear that under the hypotheses of  essinf(pq)>0𝑒𝑠𝑠infimum𝑝𝑞0ess\inf(p-q)>0italic_e italic_s italic_s roman_inf ( italic_p - italic_q ) > 0   and  p+<superscript𝑝p^{+}<\inftyitalic_p start_POSTSUPERSCRIPT + end_POSTSUPERSCRIPT < ∞ we have

limxμ(Ω)(μ(Ω)x)(pqp)(x)=0,subscript𝑥𝜇superscriptΩsuperscript𝜇Ω𝑥superscript𝑝𝑞𝑝𝑥0\lim_{x\rightarrow\mu(\Omega)^{-}}(\mu(\Omega)-x)^{(\frac{p-q}{p})^{*}(x)}=0,roman_lim start_POSTSUBSCRIPT italic_x → italic_μ ( roman_Ω ) start_POSTSUPERSCRIPT - end_POSTSUPERSCRIPT end_POSTSUBSCRIPT ( italic_μ ( roman_Ω ) - italic_x ) start_POSTSUPERSCRIPT ( divide start_ARG italic_p - italic_q end_ARG start_ARG italic_p end_ARG ) start_POSTSUPERSCRIPT ∗ end_POSTSUPERSCRIPT ( italic_x ) end_POSTSUPERSCRIPT = 0 ,

so the inclusion Lp()(μ)Lq()(μ)superscript𝐿𝑝𝜇superscript𝐿𝑞𝜇{L^{p(\cdot)}(\mu)}\hookrightarrow{L^{q(\cdot)}(\mu)}italic_L start_POSTSUPERSCRIPT italic_p ( ⋅ ) end_POSTSUPERSCRIPT ( italic_μ ) ↪ italic_L start_POSTSUPERSCRIPT italic_q ( ⋅ ) end_POSTSUPERSCRIPT ( italic_μ ) is DSS (theses hypotheses are not necessary conditions, see Example 4.9).

For variable Lebesgue spaces on finite measures  Lp()(μ)superscript𝐿𝑝𝜇{L^{p(\cdot)}(\mu)}italic_L start_POSTSUPERSCRIPT italic_p ( ⋅ ) end_POSTSUPERSCRIPT ( italic_μ )  we have the canonical extreme inclusions  L(μ)Lp()(μ)L1(μ)superscript𝐿𝜇superscript𝐿𝑝𝜇superscript𝐿1𝜇L^{\infty}(\mu)\hookrightarrow{L^{p(\cdot)}(\mu)}\hookrightarrow L^{1}(\mu)italic_L start_POSTSUPERSCRIPT ∞ end_POSTSUPERSCRIPT ( italic_μ ) ↪ italic_L start_POSTSUPERSCRIPT italic_p ( ⋅ ) end_POSTSUPERSCRIPT ( italic_μ ) ↪ italic_L start_POSTSUPERSCRIPT 1 end_POSTSUPERSCRIPT ( italic_μ )  (cf. [8, 7]).

If follows from the above a DSS criterion for the right extreme case of  L1(μ)Lq()(μ)superscript𝐿1𝜇superscript𝐿𝑞𝜇L^{1}(\mu)\equiv{L^{q(\cdot)}(\mu)}italic_L start_POSTSUPERSCRIPT 1 end_POSTSUPERSCRIPT ( italic_μ ) ≡ italic_L start_POSTSUPERSCRIPT italic_q ( ⋅ ) end_POSTSUPERSCRIPT ( italic_μ ) (recall that weakly compact subsets in  L1(μ)superscript𝐿1𝜇L^{1}(\mu)italic_L start_POSTSUPERSCRIPT 1 end_POSTSUPERSCRIPT ( italic_μ )  are the same as equi-integrable sets by Dunford-Pettis Theorem, cf. [1]):

Corollary 4.2.

Let (Ω,μ)Ω𝜇(\Omega,\mu)( roman_Ω , italic_μ ) be an atomless finite measure space and an exponent p()𝑝p(\cdot)italic_p ( ⋅ ). TFAE:

  1. (1)

    0μ(Ω)a(pp1)(x)𝑑x<superscriptsubscript0𝜇Ωsuperscript𝑎superscript𝑝𝑝1𝑥differential-d𝑥\int_{0}^{\mu(\Omega)}a^{(\frac{p}{p-1})^{*}(x)}\,dx<\infty∫ start_POSTSUBSCRIPT 0 end_POSTSUBSCRIPT start_POSTSUPERSCRIPT italic_μ ( roman_Ω ) end_POSTSUPERSCRIPT italic_a start_POSTSUPERSCRIPT ( divide start_ARG italic_p end_ARG start_ARG italic_p - 1 end_ARG ) start_POSTSUPERSCRIPT ∗ end_POSTSUPERSCRIPT ( italic_x ) end_POSTSUPERSCRIPT italic_d italic_x < ∞     for every a>1𝑎1a>1italic_a > 1.

  2. (2)

    The inclusion   Lp()(μ)L1(μ)superscript𝐿𝑝𝜇superscript𝐿1𝜇{L^{p(\cdot)}(\mu)}\hookrightarrow L^{1}(\mu)italic_L start_POSTSUPERSCRIPT italic_p ( ⋅ ) end_POSTSUPERSCRIPT ( italic_μ ) ↪ italic_L start_POSTSUPERSCRIPT 1 end_POSTSUPERSCRIPT ( italic_μ )   is weakly compact.

  3. (3)

    The inclusion   Lp()(μ)L1(μ)superscript𝐿𝑝𝜇superscript𝐿1𝜇{L^{p(\cdot)}(\mu)}\hookrightarrow L^{1}(\mu)italic_L start_POSTSUPERSCRIPT italic_p ( ⋅ ) end_POSTSUPERSCRIPT ( italic_μ ) ↪ italic_L start_POSTSUPERSCRIPT 1 end_POSTSUPERSCRIPT ( italic_μ )   is DSS.

  4. (4)

    limxμ(Ω)(μ(Ω)x)(p1p)(x)=0subscript𝑥𝜇superscriptΩsuperscript𝜇Ω𝑥superscript𝑝1𝑝𝑥0\lim_{x\rightarrow\mu(\Omega)^{-}}(\mu(\Omega)-x)^{(\frac{p-1}{p})^{*}(x)}=0roman_lim start_POSTSUBSCRIPT italic_x → italic_μ ( roman_Ω ) start_POSTSUPERSCRIPT - end_POSTSUPERSCRIPT end_POSTSUBSCRIPT ( italic_μ ( roman_Ω ) - italic_x ) start_POSTSUPERSCRIPT ( divide start_ARG italic_p - 1 end_ARG start_ARG italic_p end_ARG ) start_POSTSUPERSCRIPT ∗ end_POSTSUPERSCRIPT ( italic_x ) end_POSTSUPERSCRIPT = 0.

Remark 4.3.

Notice that the above inclusions i:Lp()(μ)L1(μ):𝑖superscript𝐿𝑝𝜇superscript𝐿1𝜇i:{L^{p(\cdot)}(\mu)}\hookrightarrow L^{1}(\mu)italic_i : italic_L start_POSTSUPERSCRIPT italic_p ( ⋅ ) end_POSTSUPERSCRIPT ( italic_μ ) ↪ italic_L start_POSTSUPERSCRIPT 1 end_POSTSUPERSCRIPT ( italic_μ ), are not strictly singular.

Indeed, in the constant exponent case p=p+subscript𝑝subscript𝑝p_{-}=p_{+}italic_p start_POSTSUBSCRIPT - end_POSTSUBSCRIPT = italic_p start_POSTSUBSCRIPT + end_POSTSUBSCRIPT it is well-known (Khintchine inequality, cf. [26] p.66). Assume now   pr<p+subscript𝑝𝑟subscript𝑝p_{-}\leq r<p_{+}\leq\inftyitalic_p start_POSTSUBSCRIPT - end_POSTSUBSCRIPT ≤ italic_r < italic_p start_POSTSUBSCRIPT + end_POSTSUBSCRIPT ≤ ∞ and consider the set  Ωr={tΩ:p(t)<r}subscriptΩ𝑟conditional-set𝑡Ω𝑝𝑡𝑟\Omega_{r}=\{t\in\Omega:p(t)<r\}roman_Ω start_POSTSUBSCRIPT italic_r end_POSTSUBSCRIPT = { italic_t ∈ roman_Ω : italic_p ( italic_t ) < italic_r }, which have  μ(Ωr)>0𝜇subscriptΩ𝑟0\mu(\Omega_{r})>0italic_μ ( roman_Ω start_POSTSUBSCRIPT italic_r end_POSTSUBSCRIPT ) > 0. The restricted variable Lebesgue space  Lp()(Ωr)superscript𝐿𝑝subscriptΩ𝑟L^{p(\cdot)}(\Omega_{r})italic_L start_POSTSUPERSCRIPT italic_p ( ⋅ ) end_POSTSUPERSCRIPT ( roman_Ω start_POSTSUBSCRIPT italic_r end_POSTSUBSCRIPT )  can be canonically identified with a closed band-subspace of  Lp()(μ)superscript𝐿𝑝𝜇{L^{p(\cdot)}(\mu)}italic_L start_POSTSUPERSCRIPT italic_p ( ⋅ ) end_POSTSUPERSCRIPT ( italic_μ ). Now as  p|Ωr+r<evaluated-at𝑝subscriptΩ𝑟𝑟p|_{\Omega_{r}}^{+}\leq r<\inftyitalic_p | start_POSTSUBSCRIPT roman_Ω start_POSTSUBSCRIPT italic_r end_POSTSUBSCRIPT end_POSTSUBSCRIPT start_POSTSUPERSCRIPT + end_POSTSUPERSCRIPT ≤ italic_r < ∞  it follows easily from Khintchine inequalities in Lpsuperscript𝐿𝑝L^{p}italic_L start_POSTSUPERSCRIPT italic_p end_POSTSUPERSCRIPT-spaces (cf. [3, 26])  that the Rademacher function system  (rn)subscript𝑟𝑛(r_{n})( italic_r start_POSTSUBSCRIPT italic_n end_POSTSUBSCRIPT )  in   Lp()(Ωr)superscript𝐿𝑝subscriptΩ𝑟L^{p(\cdot)}(\Omega_{r})italic_L start_POSTSUPERSCRIPT italic_p ( ⋅ ) end_POSTSUPERSCRIPT ( roman_Ω start_POSTSUBSCRIPT italic_r end_POSTSUBSCRIPT )  and in L1(μ)superscript𝐿1𝜇L^{1}(\mu)italic_L start_POSTSUPERSCRIPT 1 end_POSTSUPERSCRIPT ( italic_μ )  are equivalent to the canonical basis of  2subscript2\ell_{2}roman_ℓ start_POSTSUBSCRIPT 2 end_POSTSUBSCRIPT. Hence the inclusion   Lp()(μ)L1(μ)superscript𝐿𝑝𝜇superscript𝐿1𝜇{L^{p(\cdot)}(\mu)}\hookrightarrow L^{1}(\mu)italic_L start_POSTSUPERSCRIPT italic_p ( ⋅ ) end_POSTSUPERSCRIPT ( italic_μ ) ↪ italic_L start_POSTSUPERSCRIPT 1 end_POSTSUPERSCRIPT ( italic_μ )  is not strictly singular.

A similar argument shows also that all the inclusions  Lp()(μ)Lq()(μ)superscript𝐿𝑝𝜇superscript𝐿𝑞𝜇{L^{p(\cdot)}(\mu)}\hookrightarrow{L^{q(\cdot)}(\mu)}italic_L start_POSTSUPERSCRIPT italic_p ( ⋅ ) end_POSTSUPERSCRIPT ( italic_μ ) ↪ italic_L start_POSTSUPERSCRIPT italic_q ( ⋅ ) end_POSTSUPERSCRIPT ( italic_μ )  (for q()p()𝑞𝑝q(\cdot)\leq p(\cdot)italic_q ( ⋅ ) ≤ italic_p ( ⋅ )) are not strictly singular.

We consider now the left extreme inclusions  L(μ)Lp()(μ)superscript𝐿𝜇superscript𝐿𝑝𝜇L^{\infty}(\mu)\hookrightarrow{L^{p(\cdot)}(\mu)}italic_L start_POSTSUPERSCRIPT ∞ end_POSTSUPERSCRIPT ( italic_μ ) ↪ italic_L start_POSTSUPERSCRIPT italic_p ( ⋅ ) end_POSTSUPERSCRIPT ( italic_μ ). In this case the DSS property is equivalent to the strict singularity:

Theorem 4.4.

Let (Ω,μ)Ω𝜇(\Omega,\mu)( roman_Ω , italic_μ ) be an atomless finite measure space and an exponent  p(t)𝑝𝑡p(t)italic_p ( italic_t ). Denote i𝑖iitalic_i the inclusion i:L(μ)Lp()(μ):𝑖superscript𝐿𝜇superscript𝐿𝑝𝜇i:L^{\infty}(\mu)\hookrightarrow{L^{p(\cdot)}(\mu)}italic_i : italic_L start_POSTSUPERSCRIPT ∞ end_POSTSUPERSCRIPT ( italic_μ ) ↪ italic_L start_POSTSUPERSCRIPT italic_p ( ⋅ ) end_POSTSUPERSCRIPT ( italic_μ ). TFAE:

  1. (1)

    0μ(Ω)ap(x)𝑑x<superscriptsubscript0𝜇Ωsuperscript𝑎superscript𝑝𝑥differential-d𝑥\int_{0}^{\mu(\Omega)}a^{p^{*}(x)}dx<\infty∫ start_POSTSUBSCRIPT 0 end_POSTSUBSCRIPT start_POSTSUPERSCRIPT italic_μ ( roman_Ω ) end_POSTSUPERSCRIPT italic_a start_POSTSUPERSCRIPT italic_p start_POSTSUPERSCRIPT ∗ end_POSTSUPERSCRIPT ( italic_x ) end_POSTSUPERSCRIPT italic_d italic_x < ∞     for every a>1𝑎1a>1italic_a > 1.

  2. (2)

    Every measurable set sequence (En)nsubscriptsubscript𝐸𝑛𝑛(E_{n})_{n}( italic_E start_POSTSUBSCRIPT italic_n end_POSTSUBSCRIPT ) start_POSTSUBSCRIPT italic_n end_POSTSUBSCRIPT with χEn0subscript𝜒subscript𝐸𝑛0\chi_{E_{n}}\rightarrow 0italic_χ start_POSTSUBSCRIPT italic_E start_POSTSUBSCRIPT italic_n end_POSTSUBSCRIPT end_POSTSUBSCRIPT → 0 a.e. satisfies χEnp()0subscriptnormsubscript𝜒subscript𝐸𝑛𝑝0||\chi_{E_{n}}||_{p(\cdot)}\rightarrow 0| | italic_χ start_POSTSUBSCRIPT italic_E start_POSTSUBSCRIPT italic_n end_POSTSUBSCRIPT end_POSTSUBSCRIPT | | start_POSTSUBSCRIPT italic_p ( ⋅ ) end_POSTSUBSCRIPT → 0.

  3. (3)

    i𝑖iitalic_i is L𝐿Litalic_L-weakly compact.

  4. (4)

    i𝑖iitalic_i is M𝑀Mitalic_M-weakly compact.

  5. (5)

    i𝑖iitalic_i is weakly compact.

  6. (6)

    i𝑖iitalic_i is strictly singular.

  7. (7)

    i𝑖iitalic_i is DSS.

Proof.

(1)(2)12(1)\Leftrightarrow(2)( 1 ) ⇔ ( 2 ). It is Proposition 3.2.

(2)(3).23(2)\Rightarrow(3).( 2 ) ⇒ ( 3 ) . If fL(μ)𝑓superscript𝐿𝜇f\in{L^{\infty}(\mu)}italic_f ∈ italic_L start_POSTSUPERSCRIPT ∞ end_POSTSUPERSCRIPT ( italic_μ ), as fp()(f)χΩp()=fχΩp()subscriptnorm𝑓𝑝subscriptdelimited-∥∥subscriptnorm𝑓subscript𝜒Ω𝑝subscriptnorm𝑓subscriptnormsubscript𝜒Ω𝑝||f||_{p(\cdot)}\leq\left\lVert(||f||_{\infty})\chi_{\Omega}\right\rVert_{p(% \cdot)}=||f||_{\infty}||\chi_{\Omega}||_{p(\cdot)}| | italic_f | | start_POSTSUBSCRIPT italic_p ( ⋅ ) end_POSTSUBSCRIPT ≤ ∥ ( | | italic_f | | start_POSTSUBSCRIPT ∞ end_POSTSUBSCRIPT ) italic_χ start_POSTSUBSCRIPT roman_Ω end_POSTSUBSCRIPT ∥ start_POSTSUBSCRIPT italic_p ( ⋅ ) end_POSTSUBSCRIPT = | | italic_f | | start_POSTSUBSCRIPT ∞ end_POSTSUBSCRIPT | | italic_χ start_POSTSUBSCRIPT roman_Ω end_POSTSUBSCRIPT | | start_POSTSUBSCRIPT italic_p ( ⋅ ) end_POSTSUBSCRIPT, we have

limμ(A)0sup{||fχA||p():||f||1}limμ(A)0||χA||p()=0.\lim_{\mu(A)\rightarrow 0}\sup\{||f\chi_{A}||_{p(\cdot)}\,\,:\,\,||f||_{\infty% }\leq 1\,\}\leq\lim_{\mu(A)\rightarrow 0}||\chi_{A}||_{p(\cdot)}=0.roman_lim start_POSTSUBSCRIPT italic_μ ( italic_A ) → 0 end_POSTSUBSCRIPT roman_sup { | | italic_f italic_χ start_POSTSUBSCRIPT italic_A end_POSTSUBSCRIPT | | start_POSTSUBSCRIPT italic_p ( ⋅ ) end_POSTSUBSCRIPT : | | italic_f | | start_POSTSUBSCRIPT ∞ end_POSTSUBSCRIPT ≤ 1 } ≤ roman_lim start_POSTSUBSCRIPT italic_μ ( italic_A ) → 0 end_POSTSUBSCRIPT | | italic_χ start_POSTSUBSCRIPT italic_A end_POSTSUBSCRIPT | | start_POSTSUBSCRIPT italic_p ( ⋅ ) end_POSTSUBSCRIPT = 0 .

(3)(4).34(3)\Rightarrow(4).( 3 ) ⇒ ( 4 ) . It is clear.

(4)(5)45(4)\Leftrightarrow(5)( 4 ) ⇔ ( 5 ). Since L(μ)superscript𝐿𝜇{L^{\infty}(\mu)}italic_L start_POSTSUPERSCRIPT ∞ end_POSTSUPERSCRIPT ( italic_μ ) is an AM𝐴𝑀AMitalic_A italic_M-space (cf. [2] Thm. 18.11),

(5)(6).56(5)\Rightarrow(6).( 5 ) ⇒ ( 6 ) . Since L(μ)superscript𝐿𝜇{L^{\infty}(\mu)}italic_L start_POSTSUPERSCRIPT ∞ end_POSTSUPERSCRIPT ( italic_μ ) has the Dunford-Pettis property and the inclusion i𝑖iitalic_i is weakly compact (cf. [16] Thm. 3.3.5).

(6)(7).67(6)\Rightarrow(7).( 6 ) ⇒ ( 7 ) . It is obvious.

(7)(1).71(7)\Rightarrow(1).( 7 ) ⇒ ( 1 ) . Assume that there exists   a>1𝑎1a>1italic_a > 1  such that 0μ(Ω)ap(x)𝑑x=superscriptsubscript0𝜇Ωsuperscript𝑎superscript𝑝𝑥differential-d𝑥\int_{0}^{\mu(\Omega)}a^{p^{*}(x)}dx=\infty∫ start_POSTSUBSCRIPT 0 end_POSTSUBSCRIPT start_POSTSUPERSCRIPT italic_μ ( roman_Ω ) end_POSTSUPERSCRIPT italic_a start_POSTSUPERSCRIPT italic_p start_POSTSUPERSCRIPT ∗ end_POSTSUPERSCRIPT ( italic_x ) end_POSTSUPERSCRIPT italic_d italic_x = ∞. Then, by Lemma 3.1, there exists   β>0𝛽0\beta>0italic_β > 0  and a sequence of disjoint measurable sets   (En)nsubscriptsubscript𝐸𝑛𝑛(E_{n})_{n}( italic_E start_POSTSUBSCRIPT italic_n end_POSTSUBSCRIPT ) start_POSTSUBSCRIPT italic_n end_POSTSUBSCRIPT  verifying   χEnp()βsubscriptnormsubscript𝜒subscript𝐸𝑛𝑝𝛽||\chi_{E_{n}}||_{p(\cdot)}\geq\beta| | italic_χ start_POSTSUBSCRIPT italic_E start_POSTSUBSCRIPT italic_n end_POSTSUBSCRIPT end_POSTSUBSCRIPT | | start_POSTSUBSCRIPT italic_p ( ⋅ ) end_POSTSUBSCRIPT ≥ italic_β  for every n𝑛nitalic_n. Now the basic sequence (χEn)nsubscriptsubscript𝜒subscript𝐸𝑛𝑛(\chi_{E_{n}})_{n}( italic_χ start_POSTSUBSCRIPT italic_E start_POSTSUBSCRIPT italic_n end_POSTSUBSCRIPT end_POSTSUBSCRIPT ) start_POSTSUBSCRIPT italic_n end_POSTSUBSCRIPT in L(μ)superscript𝐿𝜇L^{\infty}(\mu)italic_L start_POSTSUPERSCRIPT ∞ end_POSTSUPERSCRIPT ( italic_μ ) and in Lp()(μ)superscript𝐿𝑝𝜇{L^{p(\cdot)}(\mu)}italic_L start_POSTSUPERSCRIPT italic_p ( ⋅ ) end_POSTSUPERSCRIPT ( italic_μ ) are equivalent. Indeed, since the inclusion  i:L(μ)Lp()(μ):𝑖superscript𝐿𝜇superscript𝐿𝑝𝜇i:L^{\infty}(\mu)\hookrightarrow{L^{p(\cdot)}(\mu)}italic_i : italic_L start_POSTSUPERSCRIPT ∞ end_POSTSUPERSCRIPT ( italic_μ ) ↪ italic_L start_POSTSUPERSCRIPT italic_p ( ⋅ ) end_POSTSUPERSCRIPT ( italic_μ )  is continuous, we have   n=1anχEnp()Cn=1anχEnsubscriptnormsuperscriptsubscript𝑛1subscript𝑎𝑛subscript𝜒subscript𝐸𝑛𝑝𝐶subscriptnormsuperscriptsubscript𝑛1subscript𝑎𝑛subscript𝜒subscript𝐸𝑛||\sum_{n=1}^{\infty}a_{n}\chi_{E_{n}}||_{p(\cdot)}\,\leq C||\sum_{n=1}^{% \infty}a_{n}\chi_{E_{n}}||_{\infty}| | ∑ start_POSTSUBSCRIPT italic_n = 1 end_POSTSUBSCRIPT start_POSTSUPERSCRIPT ∞ end_POSTSUPERSCRIPT italic_a start_POSTSUBSCRIPT italic_n end_POSTSUBSCRIPT italic_χ start_POSTSUBSCRIPT italic_E start_POSTSUBSCRIPT italic_n end_POSTSUBSCRIPT end_POSTSUBSCRIPT | | start_POSTSUBSCRIPT italic_p ( ⋅ ) end_POSTSUBSCRIPT ≤ italic_C | | ∑ start_POSTSUBSCRIPT italic_n = 1 end_POSTSUBSCRIPT start_POSTSUPERSCRIPT ∞ end_POSTSUPERSCRIPT italic_a start_POSTSUBSCRIPT italic_n end_POSTSUBSCRIPT italic_χ start_POSTSUBSCRIPT italic_E start_POSTSUBSCRIPT italic_n end_POSTSUBSCRIPT end_POSTSUBSCRIPT | | start_POSTSUBSCRIPT ∞ end_POSTSUBSCRIPT   for some C>0𝐶0C>0italic_C > 0. And for every natural k𝑘kitalic_k we have

n=1kanχEn=max1nk|an|1βn=1kanχEnp().subscriptnormsuperscriptsubscript𝑛1𝑘subscript𝑎𝑛subscript𝜒subscript𝐸𝑛subscript1𝑛𝑘subscript𝑎𝑛1𝛽subscriptnormsuperscriptsubscript𝑛1𝑘subscript𝑎𝑛subscript𝜒subscript𝐸𝑛𝑝||\sum_{n=1}^{k}a_{n}\chi_{E_{n}}||_{\infty}=\max_{1\leq n\leq k}|a_{n}|\,\leq% \,\,\frac{1}{\beta}\,\,||\sum_{n=1}^{k}a_{n}\chi_{E_{n}}||_{p(\cdot)}.| | ∑ start_POSTSUBSCRIPT italic_n = 1 end_POSTSUBSCRIPT start_POSTSUPERSCRIPT italic_k end_POSTSUPERSCRIPT italic_a start_POSTSUBSCRIPT italic_n end_POSTSUBSCRIPT italic_χ start_POSTSUBSCRIPT italic_E start_POSTSUBSCRIPT italic_n end_POSTSUBSCRIPT end_POSTSUBSCRIPT | | start_POSTSUBSCRIPT ∞ end_POSTSUBSCRIPT = roman_max start_POSTSUBSCRIPT 1 ≤ italic_n ≤ italic_k end_POSTSUBSCRIPT | italic_a start_POSTSUBSCRIPT italic_n end_POSTSUBSCRIPT | ≤ divide start_ARG 1 end_ARG start_ARG italic_β end_ARG | | ∑ start_POSTSUBSCRIPT italic_n = 1 end_POSTSUBSCRIPT start_POSTSUPERSCRIPT italic_k end_POSTSUPERSCRIPT italic_a start_POSTSUBSCRIPT italic_n end_POSTSUBSCRIPT italic_χ start_POSTSUBSCRIPT italic_E start_POSTSUBSCRIPT italic_n end_POSTSUBSCRIPT end_POSTSUBSCRIPT | | start_POSTSUBSCRIPT italic_p ( ⋅ ) end_POSTSUBSCRIPT .

This contradicts the DSS property. ∎

A direct consequence, by factorization, is the following:

Corollary 4.5.

Let (Ω,μ)Ω𝜇(\Omega,\mu)( roman_Ω , italic_μ ) be an atomless finite measure space and exponents  q(t)p(t)𝑞𝑡𝑝𝑡q(t)\leq p(t)italic_q ( italic_t ) ≤ italic_p ( italic_t ) μ𝜇\muitalic_μ-a.e.. If

0μ(Ω)aq(x)𝑑x=for some a>1,formulae-sequencesuperscriptsubscript0𝜇Ωsuperscript𝑎superscript𝑞𝑥differential-d𝑥for some 𝑎1\int_{0}^{\mu(\Omega)}a^{q^{*}(x)}dx=\infty\qquad\text{for some }a>1,∫ start_POSTSUBSCRIPT 0 end_POSTSUBSCRIPT start_POSTSUPERSCRIPT italic_μ ( roman_Ω ) end_POSTSUPERSCRIPT italic_a start_POSTSUPERSCRIPT italic_q start_POSTSUPERSCRIPT ∗ end_POSTSUPERSCRIPT ( italic_x ) end_POSTSUPERSCRIPT italic_d italic_x = ∞ for some italic_a > 1 ,

then the inclusion Lp()(μ)Lq()(μ)superscript𝐿𝑝𝜇superscript𝐿𝑞𝜇{L^{p(\cdot)}(\mu)}\hookrightarrow{L^{q(\cdot)}(\mu)}italic_L start_POSTSUPERSCRIPT italic_p ( ⋅ ) end_POSTSUPERSCRIPT ( italic_μ ) ↪ italic_L start_POSTSUPERSCRIPT italic_q ( ⋅ ) end_POSTSUPERSCRIPT ( italic_μ ) is not DSS.

Let us give another strictly singular criteria for inclusions  L(μ)Lp()(μ)superscript𝐿𝜇superscript𝐿𝑝𝜇L^{\infty}(\mu)\hookrightarrow{L^{p(\cdot)}(\mu)}italic_L start_POSTSUPERSCRIPT ∞ end_POSTSUPERSCRIPT ( italic_μ ) ↪ italic_L start_POSTSUPERSCRIPT italic_p ( ⋅ ) end_POSTSUPERSCRIPT ( italic_μ ). Recall that the exponential Orlicz space  L0exp(μ)subscriptsuperscript𝐿𝑒𝑥𝑝0𝜇L^{exp}_{0}(\mu)italic_L start_POSTSUPERSCRIPT italic_e italic_x italic_p end_POSTSUPERSCRIPT start_POSTSUBSCRIPT 0 end_POSTSUBSCRIPT ( italic_μ )   defined by the function  φ(x)=ex1𝜑𝑥superscript𝑒𝑥1\varphi(x)=e^{x}-1italic_φ ( italic_x ) = italic_e start_POSTSUPERSCRIPT italic_x end_POSTSUPERSCRIPT - 1  and a finite measure is the space of all measurable functions f𝑓fitalic_f such that

Ωe|rf(t)|𝑑μ< for every r>0.formulae-sequencesubscriptΩsuperscript𝑒𝑟𝑓𝑡differential-d𝜇 for every 𝑟0\,\,\int_{\Omega}e^{|rf(t)|}\,d\mu<\infty\quad\text{ for every }r>0.∫ start_POSTSUBSCRIPT roman_Ω end_POSTSUBSCRIPT italic_e start_POSTSUPERSCRIPT | italic_r italic_f ( italic_t ) | end_POSTSUPERSCRIPT italic_d italic_μ < ∞ for every italic_r > 0 .

Then, we have:

Theorem 4.6.

Let (Ω,μ)Ω𝜇(\Omega,\mu)( roman_Ω , italic_μ ) be an atomless finite measure space and an exponent p(t)𝑝𝑡p(t)italic_p ( italic_t ). TFAE:

  1. (1)

    The inclusion  L(μ)Lp()(μ)superscript𝐿𝜇superscript𝐿𝑝𝜇\displaystyle L^{\infty}(\mu)\hookrightarrow{L^{p(\cdot)}(\mu)}italic_L start_POSTSUPERSCRIPT ∞ end_POSTSUPERSCRIPT ( italic_μ ) ↪ italic_L start_POSTSUPERSCRIPT italic_p ( ⋅ ) end_POSTSUPERSCRIPT ( italic_μ )   is strictly singular.

  2. (2)

    p()L0exp(μ)𝑝subscriptsuperscript𝐿𝑒𝑥𝑝0𝜇\displaystyle p(\cdot)\in\,L^{exp}_{0}(\mu)italic_p ( ⋅ ) ∈ italic_L start_POSTSUPERSCRIPT italic_e italic_x italic_p end_POSTSUPERSCRIPT start_POSTSUBSCRIPT 0 end_POSTSUBSCRIPT ( italic_μ ).

  3. (3)

    limx00xp(s)𝑑sxln(ex)= 0subscript𝑥0superscriptsubscript0𝑥superscript𝑝𝑠differential-d𝑠𝑥𝑒𝑥 0\displaystyle\,\lim_{x\rightarrow 0}\,\frac{\int_{0}^{x}p^{*}(s)ds}{x\ln(\frac% {e}{x})}\,=\,0roman_lim start_POSTSUBSCRIPT italic_x → 0 end_POSTSUBSCRIPT divide start_ARG ∫ start_POSTSUBSCRIPT 0 end_POSTSUBSCRIPT start_POSTSUPERSCRIPT italic_x end_POSTSUPERSCRIPT italic_p start_POSTSUPERSCRIPT ∗ end_POSTSUPERSCRIPT ( italic_s ) italic_d italic_s end_ARG start_ARG italic_x roman_ln ( divide start_ARG italic_e end_ARG start_ARG italic_x end_ARG ) end_ARG = 0.

  4. (4)

    limxμ(Ω)(μ(Ω)x)(1p)(x)= 0.subscript𝑥𝜇superscriptΩsuperscript𝜇Ω𝑥superscript1𝑝𝑥 0\displaystyle\lim_{x\rightarrow\mu(\Omega)^{-}}(\mu(\Omega)-x)^{(\frac{1}{p})^% {*}(x)}=\,0.roman_lim start_POSTSUBSCRIPT italic_x → italic_μ ( roman_Ω ) start_POSTSUPERSCRIPT - end_POSTSUPERSCRIPT end_POSTSUBSCRIPT ( italic_μ ( roman_Ω ) - italic_x ) start_POSTSUPERSCRIPT ( divide start_ARG 1 end_ARG start_ARG italic_p end_ARG ) start_POSTSUPERSCRIPT ∗ end_POSTSUPERSCRIPT ( italic_x ) end_POSTSUPERSCRIPT = 0 .

Proof.

(1)(2)12(1)\Leftrightarrow(2)( 1 ) ⇔ ( 2 ). Assume that the inclusion L(μ)Lp()(μ)superscript𝐿𝜇superscript𝐿𝑝𝜇\displaystyle L^{\infty}(\mu)\hookrightarrow{L^{p(\cdot)}(\mu)}italic_L start_POSTSUPERSCRIPT ∞ end_POSTSUPERSCRIPT ( italic_μ ) ↪ italic_L start_POSTSUPERSCRIPT italic_p ( ⋅ ) end_POSTSUPERSCRIPT ( italic_μ ) is strictly singular. Then, by the above characterization,

0μ(Ω)ap(x)𝑑x=Ωap(t)𝑑μ=0μ(Ω)(ap())(x)𝑑x<superscriptsubscript0𝜇Ωsuperscript𝑎superscript𝑝𝑥differential-d𝑥subscriptΩsuperscript𝑎𝑝𝑡differential-d𝜇superscriptsubscript0𝜇Ωsuperscriptsuperscript𝑎𝑝𝑥differential-d𝑥\int_{0}^{\mu(\Omega)}a^{p^{*}(x)}dx=\int_{\Omega}a^{p(t)}d\mu=\int_{0}^{\mu(% \Omega)}(a^{p(\cdot)})^{*}(x)\,dx<\infty∫ start_POSTSUBSCRIPT 0 end_POSTSUBSCRIPT start_POSTSUPERSCRIPT italic_μ ( roman_Ω ) end_POSTSUPERSCRIPT italic_a start_POSTSUPERSCRIPT italic_p start_POSTSUPERSCRIPT ∗ end_POSTSUPERSCRIPT ( italic_x ) end_POSTSUPERSCRIPT italic_d italic_x = ∫ start_POSTSUBSCRIPT roman_Ω end_POSTSUBSCRIPT italic_a start_POSTSUPERSCRIPT italic_p ( italic_t ) end_POSTSUPERSCRIPT italic_d italic_μ = ∫ start_POSTSUBSCRIPT 0 end_POSTSUBSCRIPT start_POSTSUPERSCRIPT italic_μ ( roman_Ω ) end_POSTSUPERSCRIPT ( italic_a start_POSTSUPERSCRIPT italic_p ( ⋅ ) end_POSTSUPERSCRIPT ) start_POSTSUPERSCRIPT ∗ end_POSTSUPERSCRIPT ( italic_x ) italic_d italic_x < ∞

for every a>1.𝑎1a>1.italic_a > 1 . Since 1<a=e1s1𝑎superscript𝑒1𝑠1<a=e^{\frac{1}{s}}1 < italic_a = italic_e start_POSTSUPERSCRIPT divide start_ARG 1 end_ARG start_ARG italic_s end_ARG end_POSTSUPERSCRIPT for some  0<s<0𝑠0<s<\infty0 < italic_s < ∞, we have

Ωap(t)𝑑μ=Ωep(t)s𝑑μ<subscriptΩsuperscript𝑎𝑝𝑡differential-d𝜇subscriptΩsuperscript𝑒𝑝𝑡𝑠differential-d𝜇\int_{\Omega}a^{p(t)}d\mu=\int_{\Omega}e^{\frac{p(t)}{s}}d\mu<\infty∫ start_POSTSUBSCRIPT roman_Ω end_POSTSUBSCRIPT italic_a start_POSTSUPERSCRIPT italic_p ( italic_t ) end_POSTSUPERSCRIPT italic_d italic_μ = ∫ start_POSTSUBSCRIPT roman_Ω end_POSTSUBSCRIPT italic_e start_POSTSUPERSCRIPT divide start_ARG italic_p ( italic_t ) end_ARG start_ARG italic_s end_ARG end_POSTSUPERSCRIPT italic_d italic_μ < ∞

for every s>0𝑠0s>0italic_s > 0. Therefore p()L0exp(μ)𝑝subscriptsuperscript𝐿𝑒𝑥𝑝0𝜇p(\cdot)\in L^{exp}_{0}(\mu)italic_p ( ⋅ ) ∈ italic_L start_POSTSUPERSCRIPT italic_e italic_x italic_p end_POSTSUPERSCRIPT start_POSTSUBSCRIPT 0 end_POSTSUBSCRIPT ( italic_μ ). The converse is equal.

The equivalence (2)(3)23(2)\Leftrightarrow(3)( 2 ) ⇔ ( 3 ) follows from the following known fact: the order continuous exponential Orlicz space  L0exp(Ω)subscriptsuperscript𝐿𝑒𝑥𝑝0ΩL^{exp}_{0}(\Omega)italic_L start_POSTSUPERSCRIPT italic_e italic_x italic_p end_POSTSUPERSCRIPT start_POSTSUBSCRIPT 0 end_POSTSUBSCRIPT ( roman_Ω )  coincides with the order-continuous Marcinkiwiecz space  M0(φ)subscript𝑀0𝜑M_{0}(\varphi)italic_M start_POSTSUBSCRIPT 0 end_POSTSUBSCRIPT ( italic_φ )  defined by the function   φ(x)=xln(ex)𝜑𝑥𝑥𝑒𝑥\varphi(x)=x\ln(\frac{e}{x})italic_φ ( italic_x ) = italic_x roman_ln ( divide start_ARG italic_e end_ARG start_ARG italic_x end_ARG )  (cf. [25] p.116). Hence the exponent p()L0exp(Ω)𝑝subscriptsuperscript𝐿𝑒𝑥𝑝0Ωp(\cdot)\in L^{exp}_{0}(\Omega)italic_p ( ⋅ ) ∈ italic_L start_POSTSUPERSCRIPT italic_e italic_x italic_p end_POSTSUPERSCRIPT start_POSTSUBSCRIPT 0 end_POSTSUBSCRIPT ( roman_Ω )  satisfies the condition

limx00xp(s)𝑑sxln(ex)= 0.subscript𝑥0superscriptsubscript0𝑥superscript𝑝𝑠differential-d𝑠𝑥𝑒𝑥 0\lim_{x\rightarrow 0}\,\frac{\int_{0}^{x}p^{*}(s)ds}{x\,\ln(\frac{e}{x})}\,=\,0.roman_lim start_POSTSUBSCRIPT italic_x → 0 end_POSTSUBSCRIPT divide start_ARG ∫ start_POSTSUBSCRIPT 0 end_POSTSUBSCRIPT start_POSTSUPERSCRIPT italic_x end_POSTSUPERSCRIPT italic_p start_POSTSUPERSCRIPT ∗ end_POSTSUPERSCRIPT ( italic_s ) italic_d italic_s end_ARG start_ARG italic_x roman_ln ( divide start_ARG italic_e end_ARG start_ARG italic_x end_ARG ) end_ARG = 0 .

(1)(4)14(1)\Rightarrow(4)( 1 ) ⇒ ( 4 ). First assume 1<p(t)1𝑝𝑡1<p(t)1 < italic_p ( italic_t )μ𝜇\muitalic_μ-a.e.. Then, since  1<p(t)=p(t)p(t)1<1superscript𝑝𝑡𝑝𝑡𝑝𝑡11<p^{\prime}(t)=\frac{p(t)}{p(t)-1}<\infty1 < italic_p start_POSTSUPERSCRIPT ′ end_POSTSUPERSCRIPT ( italic_t ) = divide start_ARG italic_p ( italic_t ) end_ARG start_ARG italic_p ( italic_t ) - 1 end_ARG < ∞, (1)1(1)( 1 ) says that 0μ(Ω)a(pp1)(x)𝑑x<superscriptsubscript0𝜇Ωsuperscript𝑎superscriptsuperscript𝑝superscript𝑝1𝑥differential-d𝑥\int_{0}^{\mu(\Omega)}a^{(\frac{p^{\prime}}{p^{\prime}-1})^{*}(x)}dx<\infty∫ start_POSTSUBSCRIPT 0 end_POSTSUBSCRIPT start_POSTSUPERSCRIPT italic_μ ( roman_Ω ) end_POSTSUPERSCRIPT italic_a start_POSTSUPERSCRIPT ( divide start_ARG italic_p start_POSTSUPERSCRIPT ′ end_POSTSUPERSCRIPT end_ARG start_ARG italic_p start_POSTSUPERSCRIPT ′ end_POSTSUPERSCRIPT - 1 end_ARG ) start_POSTSUPERSCRIPT ∗ end_POSTSUPERSCRIPT ( italic_x ) end_POSTSUPERSCRIPT italic_d italic_x < ∞ for every a>1𝑎1a>1italic_a > 1. And Corollary 4.2 allows to get

limxμ(Ω)(μ(Ω)x)(p1p)(x)= 0.subscript𝑥𝜇superscriptΩsuperscript𝜇Ω𝑥superscriptsuperscript𝑝1superscript𝑝𝑥 0\lim_{x\rightarrow\mu(\Omega)^{-}}(\mu(\Omega)-x)^{(\frac{p^{\prime}-1}{p^{% \prime}})^{*}(x)}=\,0.roman_lim start_POSTSUBSCRIPT italic_x → italic_μ ( roman_Ω ) start_POSTSUPERSCRIPT - end_POSTSUPERSCRIPT end_POSTSUBSCRIPT ( italic_μ ( roman_Ω ) - italic_x ) start_POSTSUPERSCRIPT ( divide start_ARG italic_p start_POSTSUPERSCRIPT ′ end_POSTSUPERSCRIPT - 1 end_ARG start_ARG italic_p start_POSTSUPERSCRIPT ′ end_POSTSUPERSCRIPT end_ARG ) start_POSTSUPERSCRIPT ∗ end_POSTSUPERSCRIPT ( italic_x ) end_POSTSUPERSCRIPT = 0 .

Assume now that Ω1={tΩ:p(t)=1}subscriptΩ1conditional-set𝑡Ω𝑝𝑡1\Omega_{1}=\{t\in\Omega:p(t)=1\}roman_Ω start_POSTSUBSCRIPT 1 end_POSTSUBSCRIPT = { italic_t ∈ roman_Ω : italic_p ( italic_t ) = 1 } has positive measure, take then the exponent  r(t)=2χΩ1+p(t)χΩ1c𝑟𝑡2subscript𝜒subscriptΩ1𝑝𝑡subscript𝜒superscriptsubscriptΩ1𝑐r(t)=2\chi_{\Omega_{1}}+p(t)\chi_{\Omega_{1}^{c}}italic_r ( italic_t ) = 2 italic_χ start_POSTSUBSCRIPT roman_Ω start_POSTSUBSCRIPT 1 end_POSTSUBSCRIPT end_POSTSUBSCRIPT + italic_p ( italic_t ) italic_χ start_POSTSUBSCRIPT roman_Ω start_POSTSUBSCRIPT 1 end_POSTSUBSCRIPT start_POSTSUPERSCRIPT italic_c end_POSTSUPERSCRIPT end_POSTSUBSCRIPT. Thus

0μ(Ω)ar(x)𝑑x=Ωar(t)𝑑μΩ1a2𝑑μ+Ω\Ω1ap(t)𝑑μ<.superscriptsubscript0𝜇Ωsuperscript𝑎superscript𝑟𝑥differential-d𝑥subscriptΩsuperscript𝑎𝑟𝑡differential-d𝜇subscriptsubscriptΩ1superscript𝑎2differential-d𝜇subscript\ΩsubscriptΩ1superscript𝑎𝑝𝑡differential-d𝜇\int_{0}^{\mu(\Omega)}a^{r^{*}(x)}dx=\int_{\Omega}a^{r(t)}d\mu\leq\int_{\Omega% _{1}}a^{2}d\mu+\int_{\Omega\backslash\Omega_{1}}a^{p(t)}d\mu<\infty.∫ start_POSTSUBSCRIPT 0 end_POSTSUBSCRIPT start_POSTSUPERSCRIPT italic_μ ( roman_Ω ) end_POSTSUPERSCRIPT italic_a start_POSTSUPERSCRIPT italic_r start_POSTSUPERSCRIPT ∗ end_POSTSUPERSCRIPT ( italic_x ) end_POSTSUPERSCRIPT italic_d italic_x = ∫ start_POSTSUBSCRIPT roman_Ω end_POSTSUBSCRIPT italic_a start_POSTSUPERSCRIPT italic_r ( italic_t ) end_POSTSUPERSCRIPT italic_d italic_μ ≤ ∫ start_POSTSUBSCRIPT roman_Ω start_POSTSUBSCRIPT 1 end_POSTSUBSCRIPT end_POSTSUBSCRIPT italic_a start_POSTSUPERSCRIPT 2 end_POSTSUPERSCRIPT italic_d italic_μ + ∫ start_POSTSUBSCRIPT roman_Ω \ roman_Ω start_POSTSUBSCRIPT 1 end_POSTSUBSCRIPT end_POSTSUBSCRIPT italic_a start_POSTSUPERSCRIPT italic_p ( italic_t ) end_POSTSUPERSCRIPT italic_d italic_μ < ∞ .

Now, since limxμ(Ω)(μ(Ω)x)(1r)(x)= 0subscript𝑥𝜇superscriptΩsuperscript𝜇Ω𝑥superscript1𝑟𝑥 0\displaystyle\lim_{x\rightarrow\mu(\Omega)^{-}}(\mu(\Omega)-x)^{(\frac{1}{r})^% {*}(x)}=\,0roman_lim start_POSTSUBSCRIPT italic_x → italic_μ ( roman_Ω ) start_POSTSUPERSCRIPT - end_POSTSUPERSCRIPT end_POSTSUBSCRIPT ( italic_μ ( roman_Ω ) - italic_x ) start_POSTSUPERSCRIPT ( divide start_ARG 1 end_ARG start_ARG italic_r end_ARG ) start_POSTSUPERSCRIPT ∗ end_POSTSUPERSCRIPT ( italic_x ) end_POSTSUPERSCRIPT = 0 and (1r)()(1p)()superscript1𝑟superscript1𝑝(\frac{1}{r})^{*}(\cdot)\leq(\frac{1}{p})^{*}(\cdot)( divide start_ARG 1 end_ARG start_ARG italic_r end_ARG ) start_POSTSUPERSCRIPT ∗ end_POSTSUPERSCRIPT ( ⋅ ) ≤ ( divide start_ARG 1 end_ARG start_ARG italic_p end_ARG ) start_POSTSUPERSCRIPT ∗ end_POSTSUPERSCRIPT ( ⋅ ), we conclude that

limxμ(Ω)(μ(Ω)x)(1p)(x)= 0.subscript𝑥𝜇superscriptΩsuperscript𝜇Ω𝑥superscript1𝑝𝑥 0\displaystyle\lim_{x\rightarrow\mu(\Omega)^{-}}(\mu(\Omega)-x)^{(\frac{1}{p})^% {*}(x)}=\,0.roman_lim start_POSTSUBSCRIPT italic_x → italic_μ ( roman_Ω ) start_POSTSUPERSCRIPT - end_POSTSUPERSCRIPT end_POSTSUBSCRIPT ( italic_μ ( roman_Ω ) - italic_x ) start_POSTSUPERSCRIPT ( divide start_ARG 1 end_ARG start_ARG italic_p end_ARG ) start_POSTSUPERSCRIPT ∗ end_POSTSUPERSCRIPT ( italic_x ) end_POSTSUPERSCRIPT = 0 .

(4)(1)41(4)\Rightarrow(1)( 4 ) ⇒ ( 1 ) The case p+<superscript𝑝p^{+}<\inftyitalic_p start_POSTSUPERSCRIPT + end_POSTSUPERSCRIPT < ∞ is trivial. Assume now that p+=.superscript𝑝p^{+}=\infty.italic_p start_POSTSUPERSCRIPT + end_POSTSUPERSCRIPT = ∞ . Since (1p)superscript1𝑝(\frac{1}{p})^{*}( divide start_ARG 1 end_ARG start_ARG italic_p end_ARG ) start_POSTSUPERSCRIPT ∗ end_POSTSUPERSCRIPT is decreasing and   limxμ(Ω)(1p)(x)=0subscript𝑥𝜇Ωsuperscript1𝑝𝑥0\lim_{x\rightarrow\mu(\Omega)}(\frac{1}{p})^{*}(x)=0roman_lim start_POSTSUBSCRIPT italic_x → italic_μ ( roman_Ω ) end_POSTSUBSCRIPT ( divide start_ARG 1 end_ARG start_ARG italic_p end_ARG ) start_POSTSUPERSCRIPT ∗ end_POSTSUPERSCRIPT ( italic_x ) = 0, it follows from Lemma 3.7 that

0μ(Ω)a1(1p)(x)𝑑x<for every a>1.formulae-sequencesuperscriptsubscript0𝜇Ωsuperscript𝑎1superscript1𝑝𝑥differential-d𝑥for every 𝑎1\int_{0}^{\mu(\Omega)}a^{\frac{1}{(\frac{1}{p})^{*}}(x)}dx<\infty\qquad\text{% for every }a>1.∫ start_POSTSUBSCRIPT 0 end_POSTSUBSCRIPT start_POSTSUPERSCRIPT italic_μ ( roman_Ω ) end_POSTSUPERSCRIPT italic_a start_POSTSUPERSCRIPT divide start_ARG 1 end_ARG start_ARG ( divide start_ARG 1 end_ARG start_ARG italic_p end_ARG ) start_POSTSUPERSCRIPT ∗ end_POSTSUPERSCRIPT end_ARG ( italic_x ) end_POSTSUPERSCRIPT italic_d italic_x < ∞ for every italic_a > 1 .

Now, using Lemma 3.5, the functions  p=11p𝑝11𝑝p=\frac{1}{\frac{1}{p}}italic_p = divide start_ARG 1 end_ARG start_ARG divide start_ARG 1 end_ARG start_ARG italic_p end_ARG end_ARG and 1(1p)1superscript1𝑝\frac{1}{(\frac{1}{p})^{*}}divide start_ARG 1 end_ARG start_ARG ( divide start_ARG 1 end_ARG start_ARG italic_p end_ARG ) start_POSTSUPERSCRIPT ∗ end_POSTSUPERSCRIPT end_ARG are equi-distributed, thus

0μ(Ω)ap(x)𝑑x=0μ(Ω)a(1(1p))(x)𝑑x=0μ(Ω)a1(1p)(x)𝑑x<.superscriptsubscript0𝜇Ωsuperscript𝑎superscript𝑝𝑥differential-d𝑥superscriptsubscript0𝜇Ωsuperscript𝑎superscript1superscript1𝑝𝑥differential-d𝑥superscriptsubscript0𝜇Ωsuperscript𝑎1superscript1𝑝𝑥differential-d𝑥\int_{0}^{\mu(\Omega)}a^{p^{*}(x)}dx=\int_{0}^{\mu(\Omega)}a^{(\frac{1}{(\frac% {1}{p})^{*}})^{*}(x)}dx=\int_{0}^{\mu(\Omega)}a^{\frac{1}{(\frac{1}{p})^{*}}(x% )}dx<\infty.∫ start_POSTSUBSCRIPT 0 end_POSTSUBSCRIPT start_POSTSUPERSCRIPT italic_μ ( roman_Ω ) end_POSTSUPERSCRIPT italic_a start_POSTSUPERSCRIPT italic_p start_POSTSUPERSCRIPT ∗ end_POSTSUPERSCRIPT ( italic_x ) end_POSTSUPERSCRIPT italic_d italic_x = ∫ start_POSTSUBSCRIPT 0 end_POSTSUBSCRIPT start_POSTSUPERSCRIPT italic_μ ( roman_Ω ) end_POSTSUPERSCRIPT italic_a start_POSTSUPERSCRIPT ( divide start_ARG 1 end_ARG start_ARG ( divide start_ARG 1 end_ARG start_ARG italic_p end_ARG ) start_POSTSUPERSCRIPT ∗ end_POSTSUPERSCRIPT end_ARG ) start_POSTSUPERSCRIPT ∗ end_POSTSUPERSCRIPT ( italic_x ) end_POSTSUPERSCRIPT italic_d italic_x = ∫ start_POSTSUBSCRIPT 0 end_POSTSUBSCRIPT start_POSTSUPERSCRIPT italic_μ ( roman_Ω ) end_POSTSUPERSCRIPT italic_a start_POSTSUPERSCRIPT divide start_ARG 1 end_ARG start_ARG ( divide start_ARG 1 end_ARG start_ARG italic_p end_ARG ) start_POSTSUPERSCRIPT ∗ end_POSTSUPERSCRIPT end_ARG ( italic_x ) end_POSTSUPERSCRIPT italic_d italic_x < ∞ .

We pass now to study the general case of DSS inclusions  Lp()(μ)Lq()(μ)superscript𝐿𝑝𝜇superscript𝐿𝑞𝜇{L^{p(\cdot)}(\mu)}\hookrightarrow{L^{q(\cdot)}(\mu)}italic_L start_POSTSUPERSCRIPT italic_p ( ⋅ ) end_POSTSUPERSCRIPT ( italic_μ ) ↪ italic_L start_POSTSUPERSCRIPT italic_q ( ⋅ ) end_POSTSUPERSCRIPT ( italic_μ )  for unbounded exponents. First notice that now the condition (6) in above Theorem 4.1 is no enough for getting disjoint strict singularity, as the following example shows:

Example 4.7.

Let q(x)=1x𝑞𝑥1𝑥q(x)=\frac{1}{x}italic_q ( italic_x ) = divide start_ARG 1 end_ARG start_ARG italic_x end_ARG with x(0,1)𝑥01x\in(0,1)italic_x ∈ ( 0 , 1 ) and  p(x)=(1+ϵ)q(x)𝑝𝑥1italic-ϵ𝑞𝑥p(x)=(1+\epsilon)q(x)italic_p ( italic_x ) = ( 1 + italic_ϵ ) italic_q ( italic_x )   for some ϵ>0italic-ϵ0\epsilon>0italic_ϵ > 0. Since 01a1x𝑑x=superscriptsubscript01superscript𝑎1𝑥differential-d𝑥\int_{0}^{1}a^{\frac{1}{x}}dx=\infty∫ start_POSTSUBSCRIPT 0 end_POSTSUBSCRIPT start_POSTSUPERSCRIPT 1 end_POSTSUPERSCRIPT italic_a start_POSTSUPERSCRIPT divide start_ARG 1 end_ARG start_ARG italic_x end_ARG end_POSTSUPERSCRIPT italic_d italic_x = ∞ for every a>1𝑎1a>1italic_a > 1, we have by Corollary 4.4 that the inclusion Lp()Lq()superscript𝐿𝑝superscript𝐿𝑞L^{p(\cdot)}\hookrightarrow L^{q(\cdot)}italic_L start_POSTSUPERSCRIPT italic_p ( ⋅ ) end_POSTSUPERSCRIPT ↪ italic_L start_POSTSUPERSCRIPT italic_q ( ⋅ ) end_POSTSUPERSCRIPT  is not DSS, in spite of

limx1(1x)(pq/p)(x)=limx1(1x)ϵ/1+ϵ= 0.subscript𝑥superscript1superscript1𝑥superscript𝑝𝑞𝑝𝑥subscript𝑥superscript1superscript1𝑥italic-ϵ1italic-ϵ 0\lim_{x\rightarrow 1^{-}}(1-x)^{(p-q/p)^{*}(x)}=\lim_{x\rightarrow 1^{-}}(1-x)% ^{\epsilon/1+\epsilon}\,=\,0.roman_lim start_POSTSUBSCRIPT italic_x → 1 start_POSTSUPERSCRIPT - end_POSTSUPERSCRIPT end_POSTSUBSCRIPT ( 1 - italic_x ) start_POSTSUPERSCRIPT ( italic_p - italic_q / italic_p ) start_POSTSUPERSCRIPT ∗ end_POSTSUPERSCRIPT ( italic_x ) end_POSTSUPERSCRIPT = roman_lim start_POSTSUBSCRIPT italic_x → 1 start_POSTSUPERSCRIPT - end_POSTSUPERSCRIPT end_POSTSUBSCRIPT ( 1 - italic_x ) start_POSTSUPERSCRIPT italic_ϵ / 1 + italic_ϵ end_POSTSUPERSCRIPT = 0 .

We wonder now what happens with the inclusions Lp()(μ)Lq()(μ)superscript𝐿𝑝𝜇superscript𝐿𝑞𝜇{L^{p(\cdot)}(\mu)}\hookrightarrow{L^{q(\cdot)}(\mu)}italic_L start_POSTSUPERSCRIPT italic_p ( ⋅ ) end_POSTSUPERSCRIPT ( italic_μ ) ↪ italic_L start_POSTSUPERSCRIPT italic_q ( ⋅ ) end_POSTSUPERSCRIPT ( italic_μ )   whether q+=superscript𝑞q^{+}=\inftyitalic_q start_POSTSUPERSCRIPT + end_POSTSUPERSCRIPT = ∞ and 0μ(Ω)aq(x)𝑑x<superscriptsubscript0𝜇Ωsuperscript𝑎superscript𝑞𝑥differential-d𝑥\displaystyle\int_{0}^{\mu(\Omega)}a^{q^{*}(x)}dx<\infty∫ start_POSTSUBSCRIPT 0 end_POSTSUBSCRIPT start_POSTSUPERSCRIPT italic_μ ( roman_Ω ) end_POSTSUPERSCRIPT italic_a start_POSTSUPERSCRIPT italic_q start_POSTSUPERSCRIPT ∗ end_POSTSUPERSCRIPT ( italic_x ) end_POSTSUPERSCRIPT italic_d italic_x < ∞ for every a>1𝑎1a>1italic_a > 1. Notice that, if for a>1𝑎1a>1italic_a > 1

0μ(Ω)a(pqpq)(x)𝑑x<,superscriptsubscript0𝜇Ωsuperscript𝑎superscript𝑝𝑞𝑝𝑞𝑥differential-d𝑥\int_{0}^{\mu(\Omega)}a^{(\frac{p\,q}{p-q})^{*}(x)}dx<\infty,∫ start_POSTSUBSCRIPT 0 end_POSTSUBSCRIPT start_POSTSUPERSCRIPT italic_μ ( roman_Ω ) end_POSTSUPERSCRIPT italic_a start_POSTSUPERSCRIPT ( divide start_ARG italic_p italic_q end_ARG start_ARG italic_p - italic_q end_ARG ) start_POSTSUPERSCRIPT ∗ end_POSTSUPERSCRIPT ( italic_x ) end_POSTSUPERSCRIPT italic_d italic_x < ∞ ,

then also 0μ(Ω)aq(x)𝑑x<superscriptsubscript0𝜇Ωsuperscript𝑎superscript𝑞𝑥differential-d𝑥\int_{0}^{\mu(\Omega)}a^{q^{*}(x)}dx<\infty∫ start_POSTSUBSCRIPT 0 end_POSTSUBSCRIPT start_POSTSUPERSCRIPT italic_μ ( roman_Ω ) end_POSTSUPERSCRIPT italic_a start_POSTSUPERSCRIPT italic_q start_POSTSUPERSCRIPT ∗ end_POSTSUPERSCRIPT ( italic_x ) end_POSTSUPERSCRIPT italic_d italic_x < ∞. Indeed, for  1<f(t)=pq(t)1𝑓𝑡𝑝𝑞𝑡1<f(t)=\frac{p}{q}(t)1 < italic_f ( italic_t ) = divide start_ARG italic_p end_ARG start_ARG italic_q end_ARG ( italic_t ) we have

pqpq(t)=f(t)f(t)1q(t)>q(t),𝑝𝑞𝑝𝑞𝑡𝑓𝑡𝑓𝑡1𝑞𝑡𝑞𝑡\frac{p\,q}{p-q}(t)=\frac{f(t)}{f(t)-1}\,q(t)>q(t),divide start_ARG italic_p italic_q end_ARG start_ARG italic_p - italic_q end_ARG ( italic_t ) = divide start_ARG italic_f ( italic_t ) end_ARG start_ARG italic_f ( italic_t ) - 1 end_ARG italic_q ( italic_t ) > italic_q ( italic_t ) ,

so  (pqpq)(x)q(x)superscript𝑝𝑞𝑝𝑞𝑥superscript𝑞𝑥(\frac{p\,q}{p-q})^{*}(x)\geq q^{*}(x)( divide start_ARG italic_p italic_q end_ARG start_ARG italic_p - italic_q end_ARG ) start_POSTSUPERSCRIPT ∗ end_POSTSUPERSCRIPT ( italic_x ) ≥ italic_q start_POSTSUPERSCRIPT ∗ end_POSTSUPERSCRIPT ( italic_x ). This leads to the following:

Theorem 4.8.

Let (Ω,μ)Ω𝜇(\Omega,\mu)( roman_Ω , italic_μ ) be an atomless finite measure space and exponents   q()<p()𝑞𝑝q(\cdot)<p(\cdot)italic_q ( ⋅ ) < italic_p ( ⋅ ) μ𝜇\muitalic_μ-a.e.. TFAE:

  1. (1)

    0μ(Ω)a(pqpq)(x)𝑑x<superscriptsubscript0𝜇Ωsuperscript𝑎superscript𝑝𝑞𝑝𝑞𝑥differential-d𝑥\int_{0}^{\mu(\Omega)}a^{(\frac{p\,q}{p-q})^{*}(x)}dx<\infty∫ start_POSTSUBSCRIPT 0 end_POSTSUBSCRIPT start_POSTSUPERSCRIPT italic_μ ( roman_Ω ) end_POSTSUPERSCRIPT italic_a start_POSTSUPERSCRIPT ( divide start_ARG italic_p italic_q end_ARG start_ARG italic_p - italic_q end_ARG ) start_POSTSUPERSCRIPT ∗ end_POSTSUPERSCRIPT ( italic_x ) end_POSTSUPERSCRIPT italic_d italic_x < ∞     for every a>1𝑎1a>1italic_a > 1.

  2. (2)

    The inclusion  Lp()(μ)Lq()(μ)superscript𝐿𝑝𝜇superscript𝐿𝑞𝜇{L^{p(\cdot)}(\mu)}\hookrightarrow{L^{q(\cdot)}(\mu)}italic_L start_POSTSUPERSCRIPT italic_p ( ⋅ ) end_POSTSUPERSCRIPT ( italic_μ ) ↪ italic_L start_POSTSUPERSCRIPT italic_q ( ⋅ ) end_POSTSUPERSCRIPT ( italic_μ )  is L𝐿Litalic_L-weakly compact.

  3. (3)

    The inclusion Lp()(μ)Lq()(μ)superscript𝐿𝑝𝜇superscript𝐿𝑞𝜇{L^{p(\cdot)}(\mu)}\hookrightarrow{L^{q(\cdot)}(\mu)}italic_L start_POSTSUPERSCRIPT italic_p ( ⋅ ) end_POSTSUPERSCRIPT ( italic_μ ) ↪ italic_L start_POSTSUPERSCRIPT italic_q ( ⋅ ) end_POSTSUPERSCRIPT ( italic_μ )  is M𝑀Mitalic_M-weakly compact.

  4. (4)

    The inclusion Lp()(μ)Lq()(μ)superscript𝐿𝑝𝜇superscript𝐿𝑞𝜇{L^{p(\cdot)}(\mu)}\hookrightarrow{L^{q(\cdot)}(\mu)}italic_L start_POSTSUPERSCRIPT italic_p ( ⋅ ) end_POSTSUPERSCRIPT ( italic_μ ) ↪ italic_L start_POSTSUPERSCRIPT italic_q ( ⋅ ) end_POSTSUPERSCRIPT ( italic_μ )  is DSS𝐷𝑆𝑆DSSitalic_D italic_S italic_S.

  5. (5)

    The restriction of the inclusion  Lp()(μ)Lq()(μ)superscript𝐿𝑝𝜇superscript𝐿𝑞𝜇{L^{p(\cdot)}(\mu)}\hookrightarrow{L^{q(\cdot)}(\mu)}italic_L start_POSTSUPERSCRIPT italic_p ( ⋅ ) end_POSTSUPERSCRIPT ( italic_μ ) ↪ italic_L start_POSTSUPERSCRIPT italic_q ( ⋅ ) end_POSTSUPERSCRIPT ( italic_μ )  on any subspace spanned by a disjoint sequence  (χEnμ(En)1p(t))subscript𝜒subscript𝐸𝑛𝜇superscriptsubscript𝐸𝑛1𝑝𝑡(\frac{\chi_{E_{n}}}{\mu(E_{n})^{\frac{1}{p(t)}}})( divide start_ARG italic_χ start_POSTSUBSCRIPT italic_E start_POSTSUBSCRIPT italic_n end_POSTSUBSCRIPT end_POSTSUBSCRIPT end_ARG start_ARG italic_μ ( italic_E start_POSTSUBSCRIPT italic_n end_POSTSUBSCRIPT ) start_POSTSUPERSCRIPT divide start_ARG 1 end_ARG start_ARG italic_p ( italic_t ) end_ARG end_POSTSUPERSCRIPT end_ARG )  is not an isomorphism.

  6. (6)

    limxμ(Ω)(μ(Ω)x)(pqpq)(x)=0subscript𝑥𝜇superscriptΩsuperscript𝜇Ω𝑥superscript𝑝𝑞𝑝𝑞𝑥0\lim_{x\rightarrow\mu(\Omega)^{-}}\,\,(\mu(\Omega)-x)^{(\frac{p-q}{p\,q})^{*}(% x)}=0roman_lim start_POSTSUBSCRIPT italic_x → italic_μ ( roman_Ω ) start_POSTSUPERSCRIPT - end_POSTSUPERSCRIPT end_POSTSUBSCRIPT ( italic_μ ( roman_Ω ) - italic_x ) start_POSTSUPERSCRIPT ( divide start_ARG italic_p - italic_q end_ARG start_ARG italic_p italic_q end_ARG ) start_POSTSUPERSCRIPT ∗ end_POSTSUPERSCRIPT ( italic_x ) end_POSTSUPERSCRIPT = 0.

Proof.

(1)(2)12(1)\Rightarrow(2)( 1 ) ⇒ ( 2 ). Let us define the exponent s(t):=p(t)q(t)p(t)q(t)assign𝑠𝑡𝑝𝑡𝑞𝑡𝑝𝑡𝑞𝑡s(t):=\frac{p(t)\,q(t)}{p(t)-q(t)}italic_s ( italic_t ) := divide start_ARG italic_p ( italic_t ) italic_q ( italic_t ) end_ARG start_ARG italic_p ( italic_t ) - italic_q ( italic_t ) end_ARG. Then  1/q(t)=1/p(t)+1/s(t)1𝑞𝑡1𝑝𝑡1𝑠𝑡\,1/q(t)=1/p(t)+1/s(t)1 / italic_q ( italic_t ) = 1 / italic_p ( italic_t ) + 1 / italic_s ( italic_t )  and using Hölder norm inequality we have

fχAq() 2fp()χApqpq()subscriptnorm𝑓subscript𝜒𝐴𝑞2subscriptnorm𝑓𝑝subscriptnormsubscript𝜒𝐴𝑝𝑞𝑝𝑞\|f\,\chi_{A}\|_{q(\cdot)}\,\leq\,2\,\|f\|_{p(\cdot)}\,\|\chi_{A}\|_{\frac{p\,% q}{p-q}(\cdot)}∥ italic_f italic_χ start_POSTSUBSCRIPT italic_A end_POSTSUBSCRIPT ∥ start_POSTSUBSCRIPT italic_q ( ⋅ ) end_POSTSUBSCRIPT ≤ 2 ∥ italic_f ∥ start_POSTSUBSCRIPT italic_p ( ⋅ ) end_POSTSUBSCRIPT ∥ italic_χ start_POSTSUBSCRIPT italic_A end_POSTSUBSCRIPT ∥ start_POSTSUBSCRIPT divide start_ARG italic_p italic_q end_ARG start_ARG italic_p - italic_q end_ARG ( ⋅ ) end_POSTSUBSCRIPT

Hence by the hypothesis and Proposition 3.2 we get

limμ(An)0supfBLp()(μ)fχAnq() 2limμ(An)0χAnpqpq()= 0,subscript𝜇subscript𝐴𝑛0subscriptsupremum𝑓subscript𝐵superscript𝐿𝑝𝜇subscriptnorm𝑓subscript𝜒subscript𝐴𝑛𝑞2subscript𝜇subscript𝐴𝑛0subscriptnormsubscript𝜒subscript𝐴𝑛𝑝𝑞𝑝𝑞 0\lim_{\mu(A_{n})\rightarrow 0}\sup_{f\in B_{{L^{p(\cdot)}(\mu)}}}\|f\chi_{A_{n% }}\|_{q(\cdot)}\,\leq\,2\lim_{\mu(A_{n})\rightarrow 0}\|\chi_{A_{n}}\|_{\frac{% p\,q}{p-q}(\cdot)}\,=\,0,roman_lim start_POSTSUBSCRIPT italic_μ ( italic_A start_POSTSUBSCRIPT italic_n end_POSTSUBSCRIPT ) → 0 end_POSTSUBSCRIPT roman_sup start_POSTSUBSCRIPT italic_f ∈ italic_B start_POSTSUBSCRIPT italic_L start_POSTSUPERSCRIPT italic_p ( ⋅ ) end_POSTSUPERSCRIPT ( italic_μ ) end_POSTSUBSCRIPT end_POSTSUBSCRIPT ∥ italic_f italic_χ start_POSTSUBSCRIPT italic_A start_POSTSUBSCRIPT italic_n end_POSTSUBSCRIPT end_POSTSUBSCRIPT ∥ start_POSTSUBSCRIPT italic_q ( ⋅ ) end_POSTSUBSCRIPT ≤ 2 roman_lim start_POSTSUBSCRIPT italic_μ ( italic_A start_POSTSUBSCRIPT italic_n end_POSTSUBSCRIPT ) → 0 end_POSTSUBSCRIPT ∥ italic_χ start_POSTSUBSCRIPT italic_A start_POSTSUBSCRIPT italic_n end_POSTSUBSCRIPT end_POSTSUBSCRIPT ∥ start_POSTSUBSCRIPT divide start_ARG italic_p italic_q end_ARG start_ARG italic_p - italic_q end_ARG ( ⋅ ) end_POSTSUBSCRIPT = 0 ,

so the inclusion Lp()(μ)Lq()(μ)superscript𝐿𝑝𝜇superscript𝐿𝑞𝜇{L^{p(\cdot)}(\mu)}\hookrightarrow{L^{q(\cdot)}(\mu)}italic_L start_POSTSUPERSCRIPT italic_p ( ⋅ ) end_POSTSUPERSCRIPT ( italic_μ ) ↪ italic_L start_POSTSUPERSCRIPT italic_q ( ⋅ ) end_POSTSUPERSCRIPT ( italic_μ ) is L𝐿Litalic_L-weakly compact.

(2)(3)(4)(5)2345(2)\Rightarrow(3)\Rightarrow(4)\Rightarrow(5)( 2 ) ⇒ ( 3 ) ⇒ ( 4 ) ⇒ ( 5 ) are clear (see Theorem 4.1).

(5)(6)56(5)\Rightarrow(6)( 5 ) ⇒ ( 6 ) is Proposition 3.3.

(6)(1)61(6)\Rightarrow(1)( 6 ) ⇒ ( 1 ). Consider the function  f(t)=pqpq(t)𝑓𝑡𝑝𝑞𝑝𝑞𝑡f(t)=\frac{p-q}{p\,q}(t)italic_f ( italic_t ) = divide start_ARG italic_p - italic_q end_ARG start_ARG italic_p italic_q end_ARG ( italic_t ). From Lemma 3.5 we have that the functions   1pqpq(t)=pqpq(t)1𝑝𝑞𝑝𝑞𝑡𝑝𝑞𝑝𝑞𝑡\frac{1}{\frac{p-q}{pq}(t)}=\frac{pq}{p-q}(t)divide start_ARG 1 end_ARG start_ARG divide start_ARG italic_p - italic_q end_ARG start_ARG italic_p italic_q end_ARG ( italic_t ) end_ARG = divide start_ARG italic_p italic_q end_ARG start_ARG italic_p - italic_q end_ARG ( italic_t )   and   1(pqpq)(x)1superscript𝑝𝑞𝑝𝑞𝑥\frac{1}{(\frac{p-q}{pq})^{*}(x)}divide start_ARG 1 end_ARG start_ARG ( divide start_ARG italic_p - italic_q end_ARG start_ARG italic_p italic_q end_ARG ) start_POSTSUPERSCRIPT ∗ end_POSTSUPERSCRIPT ( italic_x ) end_ARG   are equi-distributed. Thus   (pqpq)=(1/f)superscript𝑝𝑞𝑝𝑞superscript1superscript𝑓(\frac{p\,q}{p-q})^{*}=(1/f^{*})^{*}( divide start_ARG italic_p italic_q end_ARG start_ARG italic_p - italic_q end_ARG ) start_POSTSUPERSCRIPT ∗ end_POSTSUPERSCRIPT = ( 1 / italic_f start_POSTSUPERSCRIPT ∗ end_POSTSUPERSCRIPT ) start_POSTSUPERSCRIPT ∗ end_POSTSUPERSCRIPT  and

0μ(Ω)a(pqpq)(x)𝑑x=0μ(Ω)a(1(pqpq))(x)𝑑xsuperscriptsubscript0𝜇Ωsuperscript𝑎superscript𝑝𝑞𝑝𝑞𝑥differential-d𝑥superscriptsubscript0𝜇Ωsuperscript𝑎superscript1superscript𝑝𝑞𝑝𝑞𝑥differential-d𝑥\int_{0}^{\mu(\Omega)}a^{\left(\frac{pq}{p-q}\right)^{*}(x)}dx=\int_{0}^{\mu(% \Omega)}a^{\left(\frac{1}{(\frac{p-q}{pq})^{*}}\right)^{*}(x)}dx∫ start_POSTSUBSCRIPT 0 end_POSTSUBSCRIPT start_POSTSUPERSCRIPT italic_μ ( roman_Ω ) end_POSTSUPERSCRIPT italic_a start_POSTSUPERSCRIPT ( divide start_ARG italic_p italic_q end_ARG start_ARG italic_p - italic_q end_ARG ) start_POSTSUPERSCRIPT ∗ end_POSTSUPERSCRIPT ( italic_x ) end_POSTSUPERSCRIPT italic_d italic_x = ∫ start_POSTSUBSCRIPT 0 end_POSTSUBSCRIPT start_POSTSUPERSCRIPT italic_μ ( roman_Ω ) end_POSTSUPERSCRIPT italic_a start_POSTSUPERSCRIPT ( divide start_ARG 1 end_ARG start_ARG ( divide start_ARG italic_p - italic_q end_ARG start_ARG italic_p italic_q end_ARG ) start_POSTSUPERSCRIPT ∗ end_POSTSUPERSCRIPT end_ARG ) start_POSTSUPERSCRIPT ∗ end_POSTSUPERSCRIPT ( italic_x ) end_POSTSUPERSCRIPT italic_d italic_x

Now, as 1/f1superscript𝑓1/f^{*}1 / italic_f start_POSTSUPERSCRIPT ∗ end_POSTSUPERSCRIPT is an increasing function it follows, by Lemma 3.6, that  (1/f)(x)=(1/f)(x)=(1/f)(μ(Ω)x)superscript1𝑓𝑥superscript1superscript𝑓𝑥1superscript𝑓𝜇Ω𝑥(1/f)^{*}(x)=(1/f^{*})^{*}(x)=(1/f^{*})(\mu(\Omega)-x)( 1 / italic_f ) start_POSTSUPERSCRIPT ∗ end_POSTSUPERSCRIPT ( italic_x ) = ( 1 / italic_f start_POSTSUPERSCRIPT ∗ end_POSTSUPERSCRIPT ) start_POSTSUPERSCRIPT ∗ end_POSTSUPERSCRIPT ( italic_x ) = ( 1 / italic_f start_POSTSUPERSCRIPT ∗ end_POSTSUPERSCRIPT ) ( italic_μ ( roman_Ω ) - italic_x ). Hence

=0μ(Ω)a1(pqpq)(μ(Ω)x)𝑑x=0μ(Ω)a1(pqpq)(y)𝑑y.absentsuperscriptsubscript0𝜇Ωsuperscript𝑎1superscript𝑝𝑞𝑝𝑞𝜇Ω𝑥differential-d𝑥superscriptsubscript0𝜇Ωsuperscript𝑎1superscript𝑝𝑞𝑝𝑞𝑦differential-d𝑦=\,\int_{0}^{\mu(\Omega)}a^{\frac{1}{(\frac{p-q}{pq})^{*}}(\mu(\Omega)-x)}dx\,% =\,\int_{0}^{\mu(\Omega)}a^{\frac{1}{(\frac{p-q}{pq})^{*}(y)}}dy.= ∫ start_POSTSUBSCRIPT 0 end_POSTSUBSCRIPT start_POSTSUPERSCRIPT italic_μ ( roman_Ω ) end_POSTSUPERSCRIPT italic_a start_POSTSUPERSCRIPT divide start_ARG 1 end_ARG start_ARG ( divide start_ARG italic_p - italic_q end_ARG start_ARG italic_p italic_q end_ARG ) start_POSTSUPERSCRIPT ∗ end_POSTSUPERSCRIPT end_ARG ( italic_μ ( roman_Ω ) - italic_x ) end_POSTSUPERSCRIPT italic_d italic_x = ∫ start_POSTSUBSCRIPT 0 end_POSTSUBSCRIPT start_POSTSUPERSCRIPT italic_μ ( roman_Ω ) end_POSTSUPERSCRIPT italic_a start_POSTSUPERSCRIPT divide start_ARG 1 end_ARG start_ARG ( divide start_ARG italic_p - italic_q end_ARG start_ARG italic_p italic_q end_ARG ) start_POSTSUPERSCRIPT ∗ end_POSTSUPERSCRIPT ( italic_y ) end_ARG end_POSTSUPERSCRIPT italic_d italic_y .

Finally, the boundedness of this integral follows now from the hypothesis and Lemma 3.7 for μ(Ω)=b𝜇Ω𝑏\mu(\Omega)=bitalic_μ ( roman_Ω ) = italic_b, since   (pqpq)()superscript𝑝𝑞𝑝𝑞(\frac{p-q}{pq})^{*}(\cdot)( divide start_ARG italic_p - italic_q end_ARG start_ARG italic_p italic_q end_ARG ) start_POSTSUPERSCRIPT ∗ end_POSTSUPERSCRIPT ( ⋅ ) is decreasing. ∎

Notice that above extends the L𝐿Litalic_L-weak compactness criterion in [9] (Thm 3.4) including now unbounded exponents.

Example 4.9.
  1. (1)

    Let   p(t)=1+ln(1lnt)𝑝𝑡11𝑡p(t)=1+\ln(1-\ln t)italic_p ( italic_t ) = 1 + roman_ln ( 1 - roman_ln italic_t )  on (0,1)01(0,1)( 0 , 1 ). The inclusion   L(0,1)Lp()(0,1)superscript𝐿01superscript𝐿𝑝01L^{\infty}(0,1)\hookrightarrow L^{p(\cdot)}(0,1)italic_L start_POSTSUPERSCRIPT ∞ end_POSTSUPERSCRIPT ( 0 , 1 ) ↪ italic_L start_POSTSUPERSCRIPT italic_p ( ⋅ ) end_POSTSUPERSCRIPT ( 0 , 1 ) is strictly singular, since for every a>1𝑎1a>1italic_a > 1

    01a1+ln(1lnt)𝑑t<.superscriptsubscript01superscript𝑎11𝑡differential-d𝑡\int_{0}^{1}a^{1+\ln(1-\ln t)}dt<\infty.∫ start_POSTSUBSCRIPT 0 end_POSTSUBSCRIPT start_POSTSUPERSCRIPT 1 end_POSTSUPERSCRIPT italic_a start_POSTSUPERSCRIPT 1 + roman_ln ( 1 - roman_ln italic_t ) end_POSTSUPERSCRIPT italic_d italic_t < ∞ .
  2. (2)

    Let   pα(t)=1tαsubscript𝑝𝛼𝑡1superscript𝑡𝛼p_{\alpha}(t)=\frac{1}{t^{\alpha}}italic_p start_POSTSUBSCRIPT italic_α end_POSTSUBSCRIPT ( italic_t ) = divide start_ARG 1 end_ARG start_ARG italic_t start_POSTSUPERSCRIPT italic_α end_POSTSUPERSCRIPT end_ARG  for α>0𝛼0\alpha>0italic_α > 0 on (0,1)01(0,1)( 0 , 1 ). The inclusions  L(0,1)Lpα()(0,1)superscript𝐿01superscript𝐿subscript𝑝𝛼01L^{\infty}(0,1)\hookrightarrow L^{p_{\alpha}(\cdot)}(0,1)italic_L start_POSTSUPERSCRIPT ∞ end_POSTSUPERSCRIPT ( 0 , 1 ) ↪ italic_L start_POSTSUPERSCRIPT italic_p start_POSTSUBSCRIPT italic_α end_POSTSUBSCRIPT ( ⋅ ) end_POSTSUPERSCRIPT ( 0 , 1 )   and Lpα()(0,1)L1(0,1)superscript𝐿subscript𝑝𝛼01superscript𝐿101L^{p_{\alpha}(\cdot)}(0,1)\hookrightarrow L^{1}(0,1)italic_L start_POSTSUPERSCRIPT italic_p start_POSTSUBSCRIPT italic_α end_POSTSUBSCRIPT ( ⋅ ) end_POSTSUPERSCRIPT ( 0 , 1 ) ↪ italic_L start_POSTSUPERSCRIPT 1 end_POSTSUPERSCRIPT ( 0 , 1 )  are not DSS.

    The inclusions   Lpα()(0,1)Lpβ()(0,1)superscript𝐿subscript𝑝𝛼01superscript𝐿subscript𝑝𝛽01L^{p_{\alpha}(\cdot)}(0,1)\hookrightarrow L^{p_{\beta}(\cdot)}(0,1)italic_L start_POSTSUPERSCRIPT italic_p start_POSTSUBSCRIPT italic_α end_POSTSUBSCRIPT ( ⋅ ) end_POSTSUPERSCRIPT ( 0 , 1 ) ↪ italic_L start_POSTSUPERSCRIPT italic_p start_POSTSUBSCRIPT italic_β end_POSTSUBSCRIPT ( ⋅ ) end_POSTSUPERSCRIPT ( 0 , 1 ), for 0<α<β10𝛼𝛽10<\alpha<\beta\leq 10 < italic_α < italic_β ≤ 1 or 1β<α<1𝛽𝛼1\leq\beta<\alpha<\infty1 ≤ italic_β < italic_α < ∞, are not DSS.

  3. (3)

    Let   pα(t)=1(1tα)subscript𝑝𝛼𝑡11superscript𝑡𝛼p_{\alpha}(t)=\frac{1}{(1-t^{\alpha})}italic_p start_POSTSUBSCRIPT italic_α end_POSTSUBSCRIPT ( italic_t ) = divide start_ARG 1 end_ARG start_ARG ( 1 - italic_t start_POSTSUPERSCRIPT italic_α end_POSTSUPERSCRIPT ) end_ARG  on (0,1)01(0,1)( 0 , 1 )  and  α>0𝛼0\alpha>0italic_α > 0. The inclusions   Lpα()(0,1)L1(0,1)superscript𝐿subscript𝑝𝛼01superscript𝐿101L^{p_{\alpha}(\cdot)}(0,1)\hookrightarrow L^{1}(0,1)italic_L start_POSTSUPERSCRIPT italic_p start_POSTSUBSCRIPT italic_α end_POSTSUBSCRIPT ( ⋅ ) end_POSTSUPERSCRIPT ( 0 , 1 ) ↪ italic_L start_POSTSUPERSCRIPT 1 end_POSTSUPERSCRIPT ( 0 , 1 )  and  L(0,1)Lpα()(0,1)superscript𝐿01superscript𝐿subscript𝑝𝛼01L^{\infty}(0,1)\hookrightarrow L^{p_{\alpha}(\cdot)}(0,1)italic_L start_POSTSUPERSCRIPT ∞ end_POSTSUPERSCRIPT ( 0 , 1 ) ↪ italic_L start_POSTSUPERSCRIPT italic_p start_POSTSUBSCRIPT italic_α end_POSTSUBSCRIPT ( ⋅ ) end_POSTSUPERSCRIPT ( 0 , 1 )  are not weakly compact.

  4. (4)

    Let  pα(t)=lnα(1t)subscript𝑝𝛼𝑡superscript𝛼1𝑡p_{\alpha}(t)=\ln^{\alpha}(\frac{1}{t})italic_p start_POSTSUBSCRIPT italic_α end_POSTSUBSCRIPT ( italic_t ) = roman_ln start_POSTSUPERSCRIPT italic_α end_POSTSUPERSCRIPT ( divide start_ARG 1 end_ARG start_ARG italic_t end_ARG ) for  t(0,1/e)𝑡01𝑒t\in(0,1/e)italic_t ∈ ( 0 , 1 / italic_e ) and 0<α<0𝛼0<\alpha<\infty0 < italic_α < ∞. The inclusion   L(0,1/e)Lpα()(0,1/e)superscript𝐿01𝑒superscript𝐿subscript𝑝𝛼01𝑒L^{\infty}(0,1/e)\hookrightarrow L^{p_{\alpha}(\cdot)}(0,1/e)italic_L start_POSTSUPERSCRIPT ∞ end_POSTSUPERSCRIPT ( 0 , 1 / italic_e ) ↪ italic_L start_POSTSUPERSCRIPT italic_p start_POSTSUBSCRIPT italic_α end_POSTSUBSCRIPT ( ⋅ ) end_POSTSUPERSCRIPT ( 0 , 1 / italic_e )  is strictly singular if and only if   0<α<10𝛼10<\alpha<10 < italic_α < 1. Indeed, this follows from Theorem 4.6, since

    limx1/e(1ex)1lnα((1ex)1)= 0subscript𝑥1𝑒superscript1𝑒𝑥1superscript𝛼superscript1𝑒𝑥1 0\lim_{x\rightarrow 1/e}\,\big{(}\frac{1}{e}-x\big{)}^{\frac{1}{\ln^{\alpha}(\,% (\frac{1}{e}-x)^{-1}\,)}}\,=\,0roman_lim start_POSTSUBSCRIPT italic_x → 1 / italic_e end_POSTSUBSCRIPT ( divide start_ARG 1 end_ARG start_ARG italic_e end_ARG - italic_x ) start_POSTSUPERSCRIPT divide start_ARG 1 end_ARG start_ARG roman_ln start_POSTSUPERSCRIPT italic_α end_POSTSUPERSCRIPT ( ( divide start_ARG 1 end_ARG start_ARG italic_e end_ARG - italic_x ) start_POSTSUPERSCRIPT - 1 end_POSTSUPERSCRIPT ) end_ARG end_POSTSUPERSCRIPT = 0

    if and only if   0<α<10𝛼10<\alpha<10 < italic_α < 1. On the other side the inclusions  Lpα()(0,1/e)L1(0,1/e)superscript𝐿subscript𝑝𝛼01𝑒superscript𝐿101𝑒L^{p_{\alpha}(\cdot)}(0,1/e)\hookrightarrow L^{1}(0,1/e)italic_L start_POSTSUPERSCRIPT italic_p start_POSTSUBSCRIPT italic_α end_POSTSUBSCRIPT ( ⋅ ) end_POSTSUPERSCRIPT ( 0 , 1 / italic_e ) ↪ italic_L start_POSTSUPERSCRIPT 1 end_POSTSUPERSCRIPT ( 0 , 1 / italic_e ), for α>0𝛼0\alpha>0italic_α > 0, are not DSS.

    The inclusions  Lpβ()(0,1/e)Lpα()(0,1/e)superscript𝐿subscript𝑝𝛽01𝑒superscript𝐿subscript𝑝𝛼01𝑒L^{p_{\beta}(\cdot)}(0,1/e)\hookrightarrow L^{p_{\alpha}(\cdot)}(0,1/e)italic_L start_POSTSUPERSCRIPT italic_p start_POSTSUBSCRIPT italic_β end_POSTSUBSCRIPT ( ⋅ ) end_POSTSUPERSCRIPT ( 0 , 1 / italic_e ) ↪ italic_L start_POSTSUPERSCRIPT italic_p start_POSTSUBSCRIPT italic_α end_POSTSUBSCRIPT ( ⋅ ) end_POSTSUPERSCRIPT ( 0 , 1 / italic_e ) for  0<α<β<0𝛼𝛽0<\alpha<\beta<\infty0 < italic_α < italic_β < ∞ are not DSS.

  5. (5)

    Let α>0𝛼0\alpha>0italic_α > 0, t(0,1/e)𝑡01𝑒t\in(0,1/e)italic_t ∈ ( 0 , 1 / italic_e ) and

    pα(t):=lnα(1t)lnα(1t)1.assignsubscript𝑝𝛼𝑡superscript𝛼1𝑡superscript𝛼1𝑡1p_{\alpha}(t):=\frac{\ln^{\alpha}(\frac{1}{t})}{\ln^{\alpha}(\frac{1}{t})-1}.italic_p start_POSTSUBSCRIPT italic_α end_POSTSUBSCRIPT ( italic_t ) := divide start_ARG roman_ln start_POSTSUPERSCRIPT italic_α end_POSTSUPERSCRIPT ( divide start_ARG 1 end_ARG start_ARG italic_t end_ARG ) end_ARG start_ARG roman_ln start_POSTSUPERSCRIPT italic_α end_POSTSUPERSCRIPT ( divide start_ARG 1 end_ARG start_ARG italic_t end_ARG ) - 1 end_ARG .

    The inclusion   Lpα()(0,1/e)L1(0,1/e)superscript𝐿subscript𝑝𝛼01𝑒superscript𝐿101𝑒L^{p_{\alpha}(\cdot)}(0,1/e)\hookrightarrow\,L^{1}(0,1/e)italic_L start_POSTSUPERSCRIPT italic_p start_POSTSUBSCRIPT italic_α end_POSTSUBSCRIPT ( ⋅ ) end_POSTSUPERSCRIPT ( 0 , 1 / italic_e ) ↪ italic_L start_POSTSUPERSCRIPT 1 end_POSTSUPERSCRIPT ( 0 , 1 / italic_e )  is weakly compact if and only if  0<α<10𝛼10<\alpha<10 < italic_α < 1.

  6. (6)

    Let q(t):=ln(1t)assign𝑞𝑡1𝑡q(t):=\sqrt{\ln(\frac{1}{t})}italic_q ( italic_t ) := square-root start_ARG roman_ln ( divide start_ARG 1 end_ARG start_ARG italic_t end_ARG ) end_ARG  for t(0,1e)𝑡01𝑒t\in(0,\frac{1}{e})italic_t ∈ ( 0 , divide start_ARG 1 end_ARG start_ARG italic_e end_ARG ) and   p(t):=(1+ϵ)q(t)assign𝑝𝑡1italic-ϵ𝑞𝑡p(t):=(1+\epsilon)q(t)italic_p ( italic_t ) := ( 1 + italic_ϵ ) italic_q ( italic_t ), for some ϵ>0italic-ϵ0\epsilon>0italic_ϵ > 0. The inclusion   Lp()(0,1e)Lq()(0,1e)superscript𝐿𝑝01𝑒superscript𝐿𝑞01𝑒L^{p(\cdot)}(0,\frac{1}{e})\hookrightarrow L^{q(\cdot)}(0,\frac{1}{e})italic_L start_POSTSUPERSCRIPT italic_p ( ⋅ ) end_POSTSUPERSCRIPT ( 0 , divide start_ARG 1 end_ARG start_ARG italic_e end_ARG ) ↪ italic_L start_POSTSUPERSCRIPT italic_q ( ⋅ ) end_POSTSUPERSCRIPT ( 0 , divide start_ARG 1 end_ARG start_ARG italic_e end_ARG )  is DSS. Indeed,

    limx1e(1ex)(pqpq)(x)=limx1e[(1ex)(1q)(x)]ϵ1+ϵ=0.subscript𝑥1𝑒superscript1𝑒𝑥superscript𝑝𝑞𝑝𝑞𝑥subscript𝑥1𝑒superscriptdelimited-[]superscript1𝑒𝑥superscript1𝑞𝑥italic-ϵ1italic-ϵ0\lim_{x\rightarrow\frac{1}{e}}(\frac{1}{e}-x)^{(\frac{p-q}{pq})^{*}(x)}=\lim_{% x\rightarrow\frac{1}{e}}\left[(\frac{1}{e}-x)^{(\frac{1}{q})^{*}(x)}\right]^{% \frac{\epsilon}{1+\epsilon}}=0.roman_lim start_POSTSUBSCRIPT italic_x → divide start_ARG 1 end_ARG start_ARG italic_e end_ARG end_POSTSUBSCRIPT ( divide start_ARG 1 end_ARG start_ARG italic_e end_ARG - italic_x ) start_POSTSUPERSCRIPT ( divide start_ARG italic_p - italic_q end_ARG start_ARG italic_p italic_q end_ARG ) start_POSTSUPERSCRIPT ∗ end_POSTSUPERSCRIPT ( italic_x ) end_POSTSUPERSCRIPT = roman_lim start_POSTSUBSCRIPT italic_x → divide start_ARG 1 end_ARG start_ARG italic_e end_ARG end_POSTSUBSCRIPT [ ( divide start_ARG 1 end_ARG start_ARG italic_e end_ARG - italic_x ) start_POSTSUPERSCRIPT ( divide start_ARG 1 end_ARG start_ARG italic_q end_ARG ) start_POSTSUPERSCRIPT ∗ end_POSTSUPERSCRIPT ( italic_x ) end_POSTSUPERSCRIPT ] start_POSTSUPERSCRIPT divide start_ARG italic_ϵ end_ARG start_ARG 1 + italic_ϵ end_ARG end_POSTSUPERSCRIPT = 0 .

In case of  p+<superscript𝑝p^{+}<\inftyitalic_p start_POSTSUPERSCRIPT + end_POSTSUPERSCRIPT < ∞, the condition  essinf(pq)()>0𝑒𝑠𝑠infimum𝑝𝑞0ess\inf(p-q)(\cdot)>0italic_e italic_s italic_s roman_inf ( italic_p - italic_q ) ( ⋅ ) > 0  is a sufficient condition for the inclusion Lp()(μ)Lq()(μ)superscript𝐿𝑝𝜇superscript𝐿𝑞𝜇{L^{p(\cdot)}(\mu)}\hookrightarrow{L^{q(\cdot)}(\mu)}italic_L start_POSTSUPERSCRIPT italic_p ( ⋅ ) end_POSTSUPERSCRIPT ( italic_μ ) ↪ italic_L start_POSTSUPERSCRIPT italic_q ( ⋅ ) end_POSTSUPERSCRIPT ( italic_μ ) be DSS ([13] Prop.3.3) or L𝐿Litalic_L-weakly compact ([9] Thm. 3.4). However this fails when  p+=superscript𝑝p^{+}=\inftyitalic_p start_POSTSUPERSCRIPT + end_POSTSUPERSCRIPT = ∞, see Example 4.7 (and compare it with Example 4.9 (6)).

5. Regular exponents

In this section we assume some regularity for the exponents, getting then a simpler DSS criterion. Recall that an scalar function f𝑓fitalic_f over a metric measure space (Ω,μ,d)Ω𝜇𝑑(\Omega,\mu,d)( roman_Ω , italic_μ , italic_d ) is said to be (locally) log-Hölder continuous if there exists a constant C>0𝐶0C>0italic_C > 0 such that, for all xyΩ𝑥𝑦Ωx\not=y\in\Omegaitalic_x ≠ italic_y ∈ roman_Ω,

|f(x)f(y)|Cln(e+1d(x,y)).𝑓𝑥𝑓𝑦𝐶𝑒1𝑑𝑥𝑦|f(x)-f(y)|\leq\frac{C}{\ln\left(e+\frac{1}{d(x,y)}\right)}.| italic_f ( italic_x ) - italic_f ( italic_y ) | ≤ divide start_ARG italic_C end_ARG start_ARG roman_ln ( italic_e + divide start_ARG 1 end_ARG start_ARG italic_d ( italic_x , italic_y ) end_ARG ) end_ARG .

The class of log-Hölder continuous exponents is very useful in applications of variable exponent spaces (cf. [8, 7]).

Proposition 5.1.

Let  f:[0,1][0,):𝑓010f:[0,1]\rightarrow[0,\infty)italic_f : [ 0 , 1 ] → [ 0 , ∞ ) be a log-Hölder continuous function. Then its decreasing rearrangement fsuperscript𝑓f^{*}italic_f start_POSTSUPERSCRIPT ∗ end_POSTSUPERSCRIPT is also log-Hölder continuous.

Proof.

Assume that  fsuperscript𝑓f^{*}italic_f start_POSTSUPERSCRIPT ∗ end_POSTSUPERSCRIPT is not log-Hölder continuous, so there exist two sequences (xn)subscript𝑥𝑛(x_{n})( italic_x start_POSTSUBSCRIPT italic_n end_POSTSUBSCRIPT ) and (yn)subscript𝑦𝑛(y_{n})( italic_y start_POSTSUBSCRIPT italic_n end_POSTSUBSCRIPT ) in [0,1]01[0,1][ 0 , 1 ] such that, for every natural n𝑛nitalic_n,

|f(xn)f(yn)|>nln(e+1|xnyn|).superscript𝑓subscript𝑥𝑛superscript𝑓subscript𝑦𝑛𝑛𝑒1subscript𝑥𝑛subscript𝑦𝑛|f^{*}(x_{n})-f^{*}(y_{n})|>\frac{n}{\ln\left(e+\frac{1}{|x_{n}-y_{n}|}\right)}.| italic_f start_POSTSUPERSCRIPT ∗ end_POSTSUPERSCRIPT ( italic_x start_POSTSUBSCRIPT italic_n end_POSTSUBSCRIPT ) - italic_f start_POSTSUPERSCRIPT ∗ end_POSTSUPERSCRIPT ( italic_y start_POSTSUBSCRIPT italic_n end_POSTSUBSCRIPT ) | > divide start_ARG italic_n end_ARG start_ARG roman_ln ( italic_e + divide start_ARG 1 end_ARG start_ARG | italic_x start_POSTSUBSCRIPT italic_n end_POSTSUBSCRIPT - italic_y start_POSTSUBSCRIPT italic_n end_POSTSUBSCRIPT | end_ARG ) end_ARG .

We can suppose   f(xn)>f(yn)superscript𝑓subscript𝑥𝑛superscript𝑓subscript𝑦𝑛f^{*}(x_{n})>f^{*}(y_{n})italic_f start_POSTSUPERSCRIPT ∗ end_POSTSUPERSCRIPT ( italic_x start_POSTSUBSCRIPT italic_n end_POSTSUBSCRIPT ) > italic_f start_POSTSUPERSCRIPT ∗ end_POSTSUPERSCRIPT ( italic_y start_POSTSUBSCRIPT italic_n end_POSTSUBSCRIPT )  for every n𝑛nitalic_n. We will look for sequences (an)subscript𝑎𝑛(a_{n})( italic_a start_POSTSUBSCRIPT italic_n end_POSTSUBSCRIPT ) and (bn)subscript𝑏𝑛(b_{n})( italic_b start_POSTSUBSCRIPT italic_n end_POSTSUBSCRIPT ) such that  |anbn||xnyn|subscript𝑎𝑛subscript𝑏𝑛subscript𝑥𝑛subscript𝑦𝑛|a_{n}-b_{n}|\leq|x_{n}-y_{n}|| italic_a start_POSTSUBSCRIPT italic_n end_POSTSUBSCRIPT - italic_b start_POSTSUBSCRIPT italic_n end_POSTSUBSCRIPT | ≤ | italic_x start_POSTSUBSCRIPT italic_n end_POSTSUBSCRIPT - italic_y start_POSTSUBSCRIPT italic_n end_POSTSUBSCRIPT |  and  |f(an)f(bn)||f(xn)f(yn)|𝑓subscript𝑎𝑛𝑓subscript𝑏𝑛superscript𝑓subscript𝑥𝑛superscript𝑓subscript𝑦𝑛|f(a_{n})-f(b_{n})|\geq|f^{*}(x_{n})-f^{*}(y_{n})|| italic_f ( italic_a start_POSTSUBSCRIPT italic_n end_POSTSUBSCRIPT ) - italic_f ( italic_b start_POSTSUBSCRIPT italic_n end_POSTSUBSCRIPT ) | ≥ | italic_f start_POSTSUPERSCRIPT ∗ end_POSTSUPERSCRIPT ( italic_x start_POSTSUBSCRIPT italic_n end_POSTSUBSCRIPT ) - italic_f start_POSTSUPERSCRIPT ∗ end_POSTSUPERSCRIPT ( italic_y start_POSTSUBSCRIPT italic_n end_POSTSUBSCRIPT ) |, since then

|f(an)f(bn)||f(xn)f(yn)|>nln(e+1|xnyn|)nln(e+1|anbn)).|f(a_{n})-f(b_{n})|\geq|f^{*}(x_{n})-f^{*}(y_{n})|>\frac{n}{\ln\left(e+\frac{1% }{|x_{n}-y_{n}|}\right)}\geq\frac{n}{\ln\left(e+\frac{1}{|a_{n}-b_{n})}\right)}.| italic_f ( italic_a start_POSTSUBSCRIPT italic_n end_POSTSUBSCRIPT ) - italic_f ( italic_b start_POSTSUBSCRIPT italic_n end_POSTSUBSCRIPT ) | ≥ | italic_f start_POSTSUPERSCRIPT ∗ end_POSTSUPERSCRIPT ( italic_x start_POSTSUBSCRIPT italic_n end_POSTSUBSCRIPT ) - italic_f start_POSTSUPERSCRIPT ∗ end_POSTSUPERSCRIPT ( italic_y start_POSTSUBSCRIPT italic_n end_POSTSUBSCRIPT ) | > divide start_ARG italic_n end_ARG start_ARG roman_ln ( italic_e + divide start_ARG 1 end_ARG start_ARG | italic_x start_POSTSUBSCRIPT italic_n end_POSTSUBSCRIPT - italic_y start_POSTSUBSCRIPT italic_n end_POSTSUBSCRIPT | end_ARG ) end_ARG ≥ divide start_ARG italic_n end_ARG start_ARG roman_ln ( italic_e + divide start_ARG 1 end_ARG start_ARG | italic_a start_POSTSUBSCRIPT italic_n end_POSTSUBSCRIPT - italic_b start_POSTSUBSCRIPT italic_n end_POSTSUBSCRIPT ) end_ARG ) end_ARG .

We know by the properties of fsuperscript𝑓f^{*}italic_f start_POSTSUPERSCRIPT ∗ end_POSTSUPERSCRIPT that

|{t[0,1]:f(yn)<f(t)<f(xn)}||xnyn|,conditional-set𝑡01superscript𝑓subscript𝑦𝑛superscript𝑓𝑡superscript𝑓subscript𝑥𝑛subscript𝑥𝑛subscript𝑦𝑛\left|\Big{\{}t\in[0,1]:f^{*}(y_{n})<f^{*}(t)<f^{*}(x_{n})\,\Big{\}}\right|% \leq|x_{n}-y_{n}|,| { italic_t ∈ [ 0 , 1 ] : italic_f start_POSTSUPERSCRIPT ∗ end_POSTSUPERSCRIPT ( italic_y start_POSTSUBSCRIPT italic_n end_POSTSUBSCRIPT ) < italic_f start_POSTSUPERSCRIPT ∗ end_POSTSUPERSCRIPT ( italic_t ) < italic_f start_POSTSUPERSCRIPT ∗ end_POSTSUPERSCRIPT ( italic_x start_POSTSUBSCRIPT italic_n end_POSTSUBSCRIPT ) } | ≤ | italic_x start_POSTSUBSCRIPT italic_n end_POSTSUBSCRIPT - italic_y start_POSTSUBSCRIPT italic_n end_POSTSUBSCRIPT | ,

and

(5.1) |{t[0,1]:f(yn)}<f(t)<f(xn)}||xnyn|.\left|\Big{\{}t\in[0,1]:\,f^{*}(y_{n})\}<f(t)<f^{*}(x_{n})\Big{\}}\,\right|% \leq\,|x_{n}-y_{n}|.| { italic_t ∈ [ 0 , 1 ] : italic_f start_POSTSUPERSCRIPT ∗ end_POSTSUPERSCRIPT ( italic_y start_POSTSUBSCRIPT italic_n end_POSTSUBSCRIPT ) } < italic_f ( italic_t ) < italic_f start_POSTSUPERSCRIPT ∗ end_POSTSUPERSCRIPT ( italic_x start_POSTSUBSCRIPT italic_n end_POSTSUBSCRIPT ) } | ≤ | italic_x start_POSTSUBSCRIPT italic_n end_POSTSUBSCRIPT - italic_y start_POSTSUBSCRIPT italic_n end_POSTSUBSCRIPT | .

For every natural n𝑛nitalic_n we define the disjoint compact sets  An:=f1([0,f(yn)])assignsubscript𝐴𝑛superscript𝑓10superscript𝑓subscript𝑦𝑛A_{n}:=f^{-1}([0,f^{*}(y_{n})])italic_A start_POSTSUBSCRIPT italic_n end_POSTSUBSCRIPT := italic_f start_POSTSUPERSCRIPT - 1 end_POSTSUPERSCRIPT ( [ 0 , italic_f start_POSTSUPERSCRIPT ∗ end_POSTSUPERSCRIPT ( italic_y start_POSTSUBSCRIPT italic_n end_POSTSUBSCRIPT ) ] ) and  Bn:=f1([f(xn),))assignsubscript𝐵𝑛superscript𝑓1superscript𝑓subscript𝑥𝑛B_{n}:=f^{-1}([f^{*}(x_{n}),\infty))italic_B start_POSTSUBSCRIPT italic_n end_POSTSUBSCRIPT := italic_f start_POSTSUPERSCRIPT - 1 end_POSTSUPERSCRIPT ( [ italic_f start_POSTSUPERSCRIPT ∗ end_POSTSUPERSCRIPT ( italic_x start_POSTSUBSCRIPT italic_n end_POSTSUBSCRIPT ) , ∞ ) ). Thus there exist anAnsubscript𝑎𝑛subscript𝐴𝑛a_{n}\in A_{n}italic_a start_POSTSUBSCRIPT italic_n end_POSTSUBSCRIPT ∈ italic_A start_POSTSUBSCRIPT italic_n end_POSTSUBSCRIPT and bnBnsubscript𝑏𝑛subscript𝐵𝑛b_{n}\in B_{n}italic_b start_POSTSUBSCRIPT italic_n end_POSTSUBSCRIPT ∈ italic_B start_POSTSUBSCRIPT italic_n end_POSTSUBSCRIPT such that

|anbn|=min{|rs|:rAn and sBn}.subscript𝑎𝑛subscript𝑏𝑛:𝑟𝑠𝑟subscript𝐴𝑛 and 𝑠subscript𝐵𝑛|a_{n}-b_{n}|\,=\,\min\{|r-s|\,:\,r\in A_{n}\text{ and }s\in B_{n}\}.| italic_a start_POSTSUBSCRIPT italic_n end_POSTSUBSCRIPT - italic_b start_POSTSUBSCRIPT italic_n end_POSTSUBSCRIPT | = roman_min { | italic_r - italic_s | : italic_r ∈ italic_A start_POSTSUBSCRIPT italic_n end_POSTSUBSCRIPT and italic_s ∈ italic_B start_POSTSUBSCRIPT italic_n end_POSTSUBSCRIPT } .

Since f𝑓fitalic_f is continuous we have

f(an)f(yn)andf(bn)f(xn).formulae-sequence𝑓subscript𝑎𝑛superscript𝑓subscript𝑦𝑛and𝑓subscript𝑏𝑛superscript𝑓subscript𝑥𝑛f(a_{n})\leq f^{*}(y_{n})\quad\text{and}\quad f(b_{n})\geq f^{*}(x_{n}).italic_f ( italic_a start_POSTSUBSCRIPT italic_n end_POSTSUBSCRIPT ) ≤ italic_f start_POSTSUPERSCRIPT ∗ end_POSTSUPERSCRIPT ( italic_y start_POSTSUBSCRIPT italic_n end_POSTSUBSCRIPT ) and italic_f ( italic_b start_POSTSUBSCRIPT italic_n end_POSTSUBSCRIPT ) ≥ italic_f start_POSTSUPERSCRIPT ∗ end_POSTSUPERSCRIPT ( italic_x start_POSTSUBSCRIPT italic_n end_POSTSUBSCRIPT ) .

Now, for t=λan+(1λ)bn𝑡𝜆subscript𝑎𝑛1𝜆subscript𝑏𝑛t=\lambda a_{n}+(1-\lambda)b_{n}italic_t = italic_λ italic_a start_POSTSUBSCRIPT italic_n end_POSTSUBSCRIPT + ( 1 - italic_λ ) italic_b start_POSTSUBSCRIPT italic_n end_POSTSUBSCRIPT with 0<λ<10𝜆10<\lambda<10 < italic_λ < 1, it follows from the definition of ansubscript𝑎𝑛a_{n}italic_a start_POSTSUBSCRIPT italic_n end_POSTSUBSCRIPT and bnsubscript𝑏𝑛b_{n}italic_b start_POSTSUBSCRIPT italic_n end_POSTSUBSCRIPT that

f(yn)<f(t)<f(xn).superscript𝑓subscript𝑦𝑛𝑓𝑡superscript𝑓subscript𝑥𝑛f^{*}(y_{n})<f(t)<f^{*}(x_{n}).italic_f start_POSTSUPERSCRIPT ∗ end_POSTSUPERSCRIPT ( italic_y start_POSTSUBSCRIPT italic_n end_POSTSUBSCRIPT ) < italic_f ( italic_t ) < italic_f start_POSTSUPERSCRIPT ∗ end_POSTSUPERSCRIPT ( italic_x start_POSTSUBSCRIPT italic_n end_POSTSUBSCRIPT ) .

Using (5.1), we conclude that  |anbn||xnyn|.subscript𝑎𝑛subscript𝑏𝑛subscript𝑥𝑛subscript𝑦𝑛|a_{n}-b_{n}|\leq|x_{n}-y_{n}|.| italic_a start_POSTSUBSCRIPT italic_n end_POSTSUBSCRIPT - italic_b start_POSTSUBSCRIPT italic_n end_POSTSUBSCRIPT | ≤ | italic_x start_POSTSUBSCRIPT italic_n end_POSTSUBSCRIPT - italic_y start_POSTSUBSCRIPT italic_n end_POSTSUBSCRIPT | .

Proposition 5.2.

Let p()q()𝑝𝑞p(\cdot)\geq q(\cdot)italic_p ( ⋅ ) ≥ italic_q ( ⋅ ) log-Hölder continuous exponents on [0,1]01[0,1][ 0 , 1 ]. If

limx1(1x)(pq)(x)=0,subscript𝑥superscript1superscript1𝑥superscript𝑝𝑞𝑥0\lim_{x\rightarrow 1^{-}}\,(1-x)^{(p-q)^{*}(x)}=0,roman_lim start_POSTSUBSCRIPT italic_x → 1 start_POSTSUPERSCRIPT - end_POSTSUPERSCRIPT end_POSTSUBSCRIPT ( 1 - italic_x ) start_POSTSUPERSCRIPT ( italic_p - italic_q ) start_POSTSUPERSCRIPT ∗ end_POSTSUPERSCRIPT ( italic_x ) end_POSTSUPERSCRIPT = 0 ,

then    essinf(pq)>0.𝑒𝑠𝑠infimum𝑝𝑞0ess\inf(p-q)>0.italic_e italic_s italic_s roman_inf ( italic_p - italic_q ) > 0 .

Proof.

Suppose that essinf(pq)=0𝑒𝑠𝑠infimum𝑝𝑞0ess\inf(p-q)=0italic_e italic_s italic_s roman_inf ( italic_p - italic_q ) = 0 (hence limx1(pq)(x)=0subscript𝑥1superscript𝑝𝑞𝑥0\lim_{x\rightarrow 1}(p-q)^{*}(x)=0roman_lim start_POSTSUBSCRIPT italic_x → 1 end_POSTSUBSCRIPT ( italic_p - italic_q ) start_POSTSUPERSCRIPT ∗ end_POSTSUPERSCRIPT ( italic_x ) = 0). Then, given xn[0,1)subscript𝑥𝑛01x_{n}\in[0,1)italic_x start_POSTSUBSCRIPT italic_n end_POSTSUBSCRIPT ∈ [ 0 , 1 ), we can take xn+1subscript𝑥𝑛1x_{n+1}italic_x start_POSTSUBSCRIPT italic_n + 1 end_POSTSUBSCRIPT close enough to 1111 to get

(xn+1xn)(pq)(xn+1)(1xn)(pq)(1)=(1xn)0=1.similar-to-or-equalssuperscriptsubscript𝑥𝑛1subscript𝑥𝑛superscript𝑝𝑞subscript𝑥𝑛1superscript1subscript𝑥𝑛superscript𝑝𝑞1superscript1subscript𝑥𝑛01(x_{n+1}-x_{n})^{(p-q)^{*}(x_{n+1})}\simeq(1-x_{n})^{(p-q)^{*}(1)}=(1-x_{n})^{% 0}=1.( italic_x start_POSTSUBSCRIPT italic_n + 1 end_POSTSUBSCRIPT - italic_x start_POSTSUBSCRIPT italic_n end_POSTSUBSCRIPT ) start_POSTSUPERSCRIPT ( italic_p - italic_q ) start_POSTSUPERSCRIPT ∗ end_POSTSUPERSCRIPT ( italic_x start_POSTSUBSCRIPT italic_n + 1 end_POSTSUBSCRIPT ) end_POSTSUPERSCRIPT ≃ ( 1 - italic_x start_POSTSUBSCRIPT italic_n end_POSTSUBSCRIPT ) start_POSTSUPERSCRIPT ( italic_p - italic_q ) start_POSTSUPERSCRIPT ∗ end_POSTSUPERSCRIPT ( 1 ) end_POSTSUPERSCRIPT = ( 1 - italic_x start_POSTSUBSCRIPT italic_n end_POSTSUBSCRIPT ) start_POSTSUPERSCRIPT 0 end_POSTSUPERSCRIPT = 1 .

Concretely, we take xn+1subscript𝑥𝑛1x_{n+1}italic_x start_POSTSUBSCRIPT italic_n + 1 end_POSTSUBSCRIPT so that (xn+1xn)(pq)(xn+1)12superscriptsubscript𝑥𝑛1subscript𝑥𝑛superscript𝑝𝑞subscript𝑥𝑛112(x_{n+1}-x_{n})^{(p-q)^{*}(x_{n+1})}\geq\frac{1}{2}( italic_x start_POSTSUBSCRIPT italic_n + 1 end_POSTSUBSCRIPT - italic_x start_POSTSUBSCRIPT italic_n end_POSTSUBSCRIPT ) start_POSTSUPERSCRIPT ( italic_p - italic_q ) start_POSTSUPERSCRIPT ∗ end_POSTSUPERSCRIPT ( italic_x start_POSTSUBSCRIPT italic_n + 1 end_POSTSUBSCRIPT ) end_POSTSUPERSCRIPT ≥ divide start_ARG 1 end_ARG start_ARG 2 end_ARG. So, by induction, for every x0[0,1)subscript𝑥001x_{0}\in[0,1)italic_x start_POSTSUBSCRIPT 0 end_POSTSUBSCRIPT ∈ [ 0 , 1 ), we can construct a sequence (xn)1subscript𝑥𝑛1(x_{n})\nearrow 1( italic_x start_POSTSUBSCRIPT italic_n end_POSTSUBSCRIPT ) ↗ 1 satisfying

(xn+1xn)(pq)(xn+1)12.superscriptsubscript𝑥𝑛1subscript𝑥𝑛superscript𝑝𝑞subscript𝑥𝑛112(x_{n+1}-x_{n})^{(p-q)^{*}(x_{n+1})}\geq\frac{1}{2}.( italic_x start_POSTSUBSCRIPT italic_n + 1 end_POSTSUBSCRIPT - italic_x start_POSTSUBSCRIPT italic_n end_POSTSUBSCRIPT ) start_POSTSUPERSCRIPT ( italic_p - italic_q ) start_POSTSUPERSCRIPT ∗ end_POSTSUPERSCRIPT ( italic_x start_POSTSUBSCRIPT italic_n + 1 end_POSTSUBSCRIPT ) end_POSTSUPERSCRIPT ≥ divide start_ARG 1 end_ARG start_ARG 2 end_ARG .

But, on the other side, p()𝑝p(\cdot)italic_p ( ⋅ ) and q()𝑞q(\cdot)italic_q ( ⋅ ) are log-Hölder continuous, so (pq)()𝑝𝑞(p-q)(\cdot)( italic_p - italic_q ) ( ⋅ ) and (pq)()superscript𝑝𝑞(p-q)^{*}(\cdot)( italic_p - italic_q ) start_POSTSUPERSCRIPT ∗ end_POSTSUPERSCRIPT ( ⋅ ) are log-Hölder too by above proposition. If we also suppose that limx1(1x)(pq)(x)=0subscript𝑥1superscript1𝑥superscript𝑝𝑞𝑥0\lim_{x\rightarrow 1}(1-x)^{(p-q)^{*}(x)}=0roman_lim start_POSTSUBSCRIPT italic_x → 1 end_POSTSUBSCRIPT ( 1 - italic_x ) start_POSTSUPERSCRIPT ( italic_p - italic_q ) start_POSTSUPERSCRIPT ∗ end_POSTSUPERSCRIPT ( italic_x ) end_POSTSUPERSCRIPT = 0 we reach a contradiction, as

(xn+1xn)(pq)(xn+1)=superscriptsubscript𝑥𝑛1subscript𝑥𝑛superscript𝑝𝑞subscript𝑥𝑛1absent\displaystyle(x_{n+1}-x_{n})^{(p-q)^{*}(x_{n+1})}={}( italic_x start_POSTSUBSCRIPT italic_n + 1 end_POSTSUBSCRIPT - italic_x start_POSTSUBSCRIPT italic_n end_POSTSUBSCRIPT ) start_POSTSUPERSCRIPT ( italic_p - italic_q ) start_POSTSUPERSCRIPT ∗ end_POSTSUPERSCRIPT ( italic_x start_POSTSUBSCRIPT italic_n + 1 end_POSTSUBSCRIPT ) end_POSTSUPERSCRIPT = (xn+1xn)(pq)(xn+1)(pq)(xn)+(pq)(xn)superscriptsubscript𝑥𝑛1subscript𝑥𝑛superscript𝑝𝑞subscript𝑥𝑛1superscript𝑝𝑞subscript𝑥𝑛superscript𝑝𝑞subscript𝑥𝑛\displaystyle(x_{n+1}-x_{n})^{(p-q)^{*}(x_{n+1})-(p-q)^{*}(x_{n})+(p-q)^{*}(x_% {n})}( italic_x start_POSTSUBSCRIPT italic_n + 1 end_POSTSUBSCRIPT - italic_x start_POSTSUBSCRIPT italic_n end_POSTSUBSCRIPT ) start_POSTSUPERSCRIPT ( italic_p - italic_q ) start_POSTSUPERSCRIPT ∗ end_POSTSUPERSCRIPT ( italic_x start_POSTSUBSCRIPT italic_n + 1 end_POSTSUBSCRIPT ) - ( italic_p - italic_q ) start_POSTSUPERSCRIPT ∗ end_POSTSUPERSCRIPT ( italic_x start_POSTSUBSCRIPT italic_n end_POSTSUBSCRIPT ) + ( italic_p - italic_q ) start_POSTSUPERSCRIPT ∗ end_POSTSUPERSCRIPT ( italic_x start_POSTSUBSCRIPT italic_n end_POSTSUBSCRIPT ) end_POSTSUPERSCRIPT
=\displaystyle={}= (1xn+1xn)(pq)(xn)(pq)(xn+1)(xn+1xn)(pq)(xn)superscript1subscript𝑥𝑛1subscript𝑥𝑛superscript𝑝𝑞subscript𝑥𝑛superscript𝑝𝑞subscript𝑥𝑛1superscriptsubscript𝑥𝑛1subscript𝑥𝑛superscript𝑝𝑞subscript𝑥𝑛\displaystyle\left(\frac{1}{x_{n+1}-x_{n}}\right)^{(p-q)^{*}(x_{n})-(p-q)^{*}(% x_{n+1})}\cdot(x_{n+1}-x_{n})^{(p-q)^{*}(x_{n})}( divide start_ARG 1 end_ARG start_ARG italic_x start_POSTSUBSCRIPT italic_n + 1 end_POSTSUBSCRIPT - italic_x start_POSTSUBSCRIPT italic_n end_POSTSUBSCRIPT end_ARG ) start_POSTSUPERSCRIPT ( italic_p - italic_q ) start_POSTSUPERSCRIPT ∗ end_POSTSUPERSCRIPT ( italic_x start_POSTSUBSCRIPT italic_n end_POSTSUBSCRIPT ) - ( italic_p - italic_q ) start_POSTSUPERSCRIPT ∗ end_POSTSUPERSCRIPT ( italic_x start_POSTSUBSCRIPT italic_n + 1 end_POSTSUBSCRIPT ) end_POSTSUPERSCRIPT ⋅ ( italic_x start_POSTSUBSCRIPT italic_n + 1 end_POSTSUBSCRIPT - italic_x start_POSTSUBSCRIPT italic_n end_POSTSUBSCRIPT ) start_POSTSUPERSCRIPT ( italic_p - italic_q ) start_POSTSUPERSCRIPT ∗ end_POSTSUPERSCRIPT ( italic_x start_POSTSUBSCRIPT italic_n end_POSTSUBSCRIPT ) end_POSTSUPERSCRIPT
\displaystyle\leq{} (1xn+1xn)Mln(e+1xn+1xn)(1xn)(pq)(xn)n0,𝑛superscript1subscript𝑥𝑛1subscript𝑥𝑛𝑀𝑒1subscript𝑥𝑛1subscript𝑥𝑛superscript1subscript𝑥𝑛superscript𝑝𝑞subscript𝑥𝑛0\displaystyle\left(\frac{1}{x_{n+1}-x_{n}}\right)^{\frac{M}{\ln\left(e+\frac{1% }{x_{n+1}-x_{n}}\right)}}\cdot(1-x_{n})^{(p-q)^{*}(x_{n})}\xrightarrow{n% \rightarrow\infty}0,( divide start_ARG 1 end_ARG start_ARG italic_x start_POSTSUBSCRIPT italic_n + 1 end_POSTSUBSCRIPT - italic_x start_POSTSUBSCRIPT italic_n end_POSTSUBSCRIPT end_ARG ) start_POSTSUPERSCRIPT divide start_ARG italic_M end_ARG start_ARG roman_ln ( italic_e + divide start_ARG 1 end_ARG start_ARG italic_x start_POSTSUBSCRIPT italic_n + 1 end_POSTSUBSCRIPT - italic_x start_POSTSUBSCRIPT italic_n end_POSTSUBSCRIPT end_ARG ) end_ARG end_POSTSUPERSCRIPT ⋅ ( 1 - italic_x start_POSTSUBSCRIPT italic_n end_POSTSUBSCRIPT ) start_POSTSUPERSCRIPT ( italic_p - italic_q ) start_POSTSUPERSCRIPT ∗ end_POSTSUPERSCRIPT ( italic_x start_POSTSUBSCRIPT italic_n end_POSTSUBSCRIPT ) end_POSTSUPERSCRIPT start_ARROW start_OVERACCENT italic_n → ∞ end_OVERACCENT → end_ARROW 0 ,

because (1xn)(pq)(xn)n0𝑛superscript1subscript𝑥𝑛superscript𝑝𝑞subscript𝑥𝑛0(1-x_{n})^{(p-q)^{*}(x_{n})}\xrightarrow{n\rightarrow\infty}0( 1 - italic_x start_POSTSUBSCRIPT italic_n end_POSTSUBSCRIPT ) start_POSTSUPERSCRIPT ( italic_p - italic_q ) start_POSTSUPERSCRIPT ∗ end_POSTSUPERSCRIPT ( italic_x start_POSTSUBSCRIPT italic_n end_POSTSUBSCRIPT ) end_POSTSUPERSCRIPT start_ARROW start_OVERACCENT italic_n → ∞ end_OVERACCENT → end_ARROW 0  and  (1xn+1xn)Mln(e+1xn+1xn)eM.superscript1subscript𝑥𝑛1subscript𝑥𝑛𝑀𝑒1subscript𝑥𝑛1subscript𝑥𝑛superscript𝑒𝑀\left(\frac{1}{x_{n+1}-x_{n}}\right)^{\frac{M}{\ln\left(e+\frac{1}{x_{n+1}-x_{% n}}\right)}}\rightarrow e^{M}.( divide start_ARG 1 end_ARG start_ARG italic_x start_POSTSUBSCRIPT italic_n + 1 end_POSTSUBSCRIPT - italic_x start_POSTSUBSCRIPT italic_n end_POSTSUBSCRIPT end_ARG ) start_POSTSUPERSCRIPT divide start_ARG italic_M end_ARG start_ARG roman_ln ( italic_e + divide start_ARG 1 end_ARG start_ARG italic_x start_POSTSUBSCRIPT italic_n + 1 end_POSTSUBSCRIPT - italic_x start_POSTSUBSCRIPT italic_n end_POSTSUBSCRIPT end_ARG ) end_ARG end_POSTSUPERSCRIPT → italic_e start_POSTSUPERSCRIPT italic_M end_POSTSUPERSCRIPT .

Corollary 5.3.

Let  p()q()𝑝𝑞p(\cdot)\geq q(\cdot)italic_p ( ⋅ ) ≥ italic_q ( ⋅ )   be log-Hölder continuous exponents on [0,1]01[0,1][ 0 , 1 ] with p+<superscript𝑝p^{+}<\inftyitalic_p start_POSTSUPERSCRIPT + end_POSTSUPERSCRIPT < ∞. TFAE:

  1. (1)

    essinf(p()q())>0𝑒𝑠𝑠infimum𝑝𝑞0ess\inf(p(\cdot)-q(\cdot))>0italic_e italic_s italic_s roman_inf ( italic_p ( ⋅ ) - italic_q ( ⋅ ) ) > 0.

  2. (2)

    The inclusion  Lp()[0,1]Lq()[0,1]superscript𝐿𝑝01superscript𝐿𝑞01{L^{p(\cdot)}[0,1]}\hookrightarrow{L^{q(\cdot)}[0,1]}italic_L start_POSTSUPERSCRIPT italic_p ( ⋅ ) end_POSTSUPERSCRIPT [ 0 , 1 ] ↪ italic_L start_POSTSUPERSCRIPT italic_q ( ⋅ ) end_POSTSUPERSCRIPT [ 0 , 1 ]  is DSS.

  3. (3)

    limx1(1x)(pq)(x)=0subscript𝑥superscript1superscript1𝑥superscript𝑝𝑞𝑥0\lim_{x\rightarrow 1^{-}}\,\,(1-x)^{(p-q)^{*}(x)}=0roman_lim start_POSTSUBSCRIPT italic_x → 1 start_POSTSUPERSCRIPT - end_POSTSUPERSCRIPT end_POSTSUBSCRIPT ( 1 - italic_x ) start_POSTSUPERSCRIPT ( italic_p - italic_q ) start_POSTSUPERSCRIPT ∗ end_POSTSUPERSCRIPT ( italic_x ) end_POSTSUPERSCRIPT = 0.

Proof.

The above proposition shows that (1)1(1)( 1 ) and (3)3(3)( 3 ) are equivalent, and the others follows from Theorem 4.1. ∎

Remark 5.4.

Notice that every other statement at Theorem 4.1 is also equivalent.

Example 5.5.

(i) Inclusions  Lpα()[0,1]Lp[0,1]superscript𝐿subscript𝑝𝛼01superscript𝐿𝑝01L^{p_{\alpha}(\cdot)}[0,1]\hookrightarrow L^{p}[0,1]italic_L start_POSTSUPERSCRIPT italic_p start_POSTSUBSCRIPT italic_α end_POSTSUBSCRIPT ( ⋅ ) end_POSTSUPERSCRIPT [ 0 , 1 ] ↪ italic_L start_POSTSUPERSCRIPT italic_p end_POSTSUPERSCRIPT [ 0 , 1 ]  are not DSS , for   pα(x)=p+xαsubscript𝑝𝛼𝑥𝑝superscript𝑥𝛼p_{\alpha}(x)=p+x^{\alpha}italic_p start_POSTSUBSCRIPT italic_α end_POSTSUBSCRIPT ( italic_x ) = italic_p + italic_x start_POSTSUPERSCRIPT italic_α end_POSTSUPERSCRIPT,   α>0𝛼0\alpha>0italic_α > 0  and   1p<1𝑝1\leq p<\infty1 ≤ italic_p < ∞.

In general  essinf(pq)()>0𝑒𝑠𝑠infimum𝑝𝑞0ess\inf(p-q)(\cdot)>0italic_e italic_s italic_s roman_inf ( italic_p - italic_q ) ( ⋅ ) > 0  is not an equivalence for an inclusion be DSS but just a sufficient condition (even in the case of be p()𝑝p(\cdot)italic_p ( ⋅ ) continuous and q()𝑞q(\cdot)italic_q ( ⋅ ) log-Hölder continuous):

(ii) Take a log-Hölder continuous exponent q()𝑞q(\cdot)italic_q ( ⋅ ), the continuous function

r(x)=ln([log2(1x)]2j)log2(1x),𝑟𝑥superscriptdelimited-[]subscript21𝑥2𝑗subscript21𝑥r(x)=\frac{\ln\left(\left[\log_{2}(1-x)\right]^{2j}\right)}{-\log_{2}(1-x)},italic_r ( italic_x ) = divide start_ARG roman_ln ( [ roman_log start_POSTSUBSCRIPT 2 end_POSTSUBSCRIPT ( 1 - italic_x ) ] start_POSTSUPERSCRIPT 2 italic_j end_POSTSUPERSCRIPT ) end_ARG start_ARG - roman_log start_POSTSUBSCRIPT 2 end_POSTSUBSCRIPT ( 1 - italic_x ) end_ARG ,

(for j𝑗jitalic_j natural) and the exponent  p()=q()+r(12e)χ[0,12e)+r()χ[12e,1]𝑝𝑞𝑟1superscript2𝑒subscript𝜒01superscript2𝑒𝑟subscript𝜒1superscript2𝑒1p(\cdot)=q(\cdot)+r(1-2^{-e})\chi_{[0,1-2^{-e})}+r(\cdot)\chi_{[1-2^{-e},1]}italic_p ( ⋅ ) = italic_q ( ⋅ ) + italic_r ( 1 - 2 start_POSTSUPERSCRIPT - italic_e end_POSTSUPERSCRIPT ) italic_χ start_POSTSUBSCRIPT [ 0 , 1 - 2 start_POSTSUPERSCRIPT - italic_e end_POSTSUPERSCRIPT ) end_POSTSUBSCRIPT + italic_r ( ⋅ ) italic_χ start_POSTSUBSCRIPT [ 1 - 2 start_POSTSUPERSCRIPT - italic_e end_POSTSUPERSCRIPT , 1 ] end_POSTSUBSCRIPT. Then  essinf(pq)()=0𝑒𝑠𝑠infimum𝑝𝑞0ess\inf(p-q)(\cdot)=0italic_e italic_s italic_s roman_inf ( italic_p - italic_q ) ( ⋅ ) = 0  but the inclusion  Lp()[0,1]Lq()[0,1]superscript𝐿𝑝01superscript𝐿𝑞01L^{p(\cdot)}[0,1]\hookrightarrow L^{q(\cdot)}[0,1]italic_L start_POSTSUPERSCRIPT italic_p ( ⋅ ) end_POSTSUPERSCRIPT [ 0 , 1 ] ↪ italic_L start_POSTSUPERSCRIPT italic_q ( ⋅ ) end_POSTSUPERSCRIPT [ 0 , 1 ]  is M𝑀Mitalic_M-weakly compact for big enough jp+𝑗superscript𝑝j\geq p^{+}italic_j ≥ italic_p start_POSTSUPERSCRIPT + end_POSTSUPERSCRIPT and hence DSS (see [22] p.9).

We do not know whether above criteria can be extended to bounded open subsets in  Rnsuperscript𝑅𝑛R^{n}italic_R start_POSTSUPERSCRIPT italic_n end_POSTSUPERSCRIPT  (n2𝑛2n\geq 2italic_n ≥ 2).

6. The infinite measure case

In order to study DSS inclusions on an infinite measure, we can always assume that  μ(Ωd)=𝜇subscriptΩ𝑑\mu(\Omega_{d})=\inftyitalic_μ ( roman_Ω start_POSTSUBSCRIPT italic_d end_POSTSUBSCRIPT ) = ∞  (in Proposition 2.1), avoiding trivial cases of non-DSS inclusions.

If the inclusion  Lp()(μ)Lq()(μ)superscript𝐿𝑝𝜇superscript𝐿𝑞𝜇{L^{p(\cdot)}(\mu)}\hookrightarrow{L^{q(\cdot)}(\mu)}italic_L start_POSTSUPERSCRIPT italic_p ( ⋅ ) end_POSTSUPERSCRIPT ( italic_μ ) ↪ italic_L start_POSTSUPERSCRIPT italic_q ( ⋅ ) end_POSTSUPERSCRIPT ( italic_μ )  holds and p+<superscript𝑝p^{+}<\inftyitalic_p start_POSTSUPERSCRIPT + end_POSTSUPERSCRIPT < ∞, then the sets  Dεsubscript𝐷𝜀D_{\varepsilon}italic_D start_POSTSUBSCRIPT italic_ε end_POSTSUBSCRIPT  (for every ε>0𝜀0\varepsilon>0italic_ε > 0) has infinite measure, where

Dε:={tΩd:p(t)<q(t)+ε}.assignsubscript𝐷𝜀conditional-set𝑡subscriptΩ𝑑𝑝𝑡𝑞𝑡𝜀D_{\varepsilon}:=\{t\in\Omega_{d}:p(t)<q(t)+\varepsilon\}.italic_D start_POSTSUBSCRIPT italic_ε end_POSTSUBSCRIPT := { italic_t ∈ roman_Ω start_POSTSUBSCRIPT italic_d end_POSTSUBSCRIPT : italic_p ( italic_t ) < italic_q ( italic_t ) + italic_ε } .

Indeed, assume that there exists ε>0𝜀0\varepsilon>0italic_ε > 0 such that μ(Dε)<𝜇subscript𝐷𝜀\mu(D_{\varepsilon})<\inftyitalic_μ ( italic_D start_POSTSUBSCRIPT italic_ε end_POSTSUBSCRIPT ) < ∞. Then,   μ(ΩdDε)=𝜇subscriptΩ𝑑subscript𝐷𝜀\mu(\Omega_{d}\setminus D_{\varepsilon})=\inftyitalic_μ ( roman_Ω start_POSTSUBSCRIPT italic_d end_POSTSUBSCRIPT ∖ italic_D start_POSTSUBSCRIPT italic_ε end_POSTSUBSCRIPT ) = ∞  and if   r(t):=pqpq(t)assign𝑟𝑡𝑝𝑞𝑝𝑞𝑡r(t):=\frac{p\,q}{p-q}(t)italic_r ( italic_t ) := divide start_ARG italic_p italic_q end_ARG start_ARG italic_p - italic_q end_ARG ( italic_t ), we have  r|ΩdDε+p+q+ε<r^{+}_{|\Omega_{d}\setminus D_{\varepsilon}}\leq\frac{p^{+}q^{+}}{\varepsilon}<\inftyitalic_r start_POSTSUPERSCRIPT + end_POSTSUPERSCRIPT start_POSTSUBSCRIPT | roman_Ω start_POSTSUBSCRIPT italic_d end_POSTSUBSCRIPT ∖ italic_D start_POSTSUBSCRIPT italic_ε end_POSTSUBSCRIPT end_POSTSUBSCRIPT ≤ divide start_ARG italic_p start_POSTSUPERSCRIPT + end_POSTSUPERSCRIPT italic_q start_POSTSUPERSCRIPT + end_POSTSUPERSCRIPT end_ARG start_ARG italic_ε end_ARG < ∞.   Hence, for every λ>1𝜆1\lambda>1italic_λ > 1,

Ωdλr(t)𝑑μΩdDελr(t)𝑑μλp+q+εμ(ΩdDε)=subscriptsubscriptΩ𝑑superscript𝜆𝑟𝑡differential-d𝜇subscriptsubscriptΩ𝑑subscript𝐷𝜀superscript𝜆𝑟𝑡differential-d𝜇superscript𝜆superscript𝑝superscript𝑞𝜀𝜇subscriptΩ𝑑subscript𝐷𝜀\int_{\Omega_{d}}\lambda^{-r(t)}d\mu\geq\int_{\Omega_{d}\setminus D_{% \varepsilon}}\lambda^{-r(t)}d\mu\geq\lambda^{-\frac{p^{+}q^{+}}{\varepsilon}}% \mu(\Omega_{d}\setminus D_{\varepsilon})=\infty∫ start_POSTSUBSCRIPT roman_Ω start_POSTSUBSCRIPT italic_d end_POSTSUBSCRIPT end_POSTSUBSCRIPT italic_λ start_POSTSUPERSCRIPT - italic_r ( italic_t ) end_POSTSUPERSCRIPT italic_d italic_μ ≥ ∫ start_POSTSUBSCRIPT roman_Ω start_POSTSUBSCRIPT italic_d end_POSTSUBSCRIPT ∖ italic_D start_POSTSUBSCRIPT italic_ε end_POSTSUBSCRIPT end_POSTSUBSCRIPT italic_λ start_POSTSUPERSCRIPT - italic_r ( italic_t ) end_POSTSUPERSCRIPT italic_d italic_μ ≥ italic_λ start_POSTSUPERSCRIPT - divide start_ARG italic_p start_POSTSUPERSCRIPT + end_POSTSUPERSCRIPT italic_q start_POSTSUPERSCRIPT + end_POSTSUPERSCRIPT end_ARG start_ARG italic_ε end_ARG end_POSTSUPERSCRIPT italic_μ ( roman_Ω start_POSTSUBSCRIPT italic_d end_POSTSUBSCRIPT ∖ italic_D start_POSTSUBSCRIPT italic_ε end_POSTSUBSCRIPT ) = ∞

which, using Proposition 2.1, gives a contradiction.

Proposition 6.1.

Let (Ω,μ)Ω𝜇(\Omega,\mu)( roman_Ω , italic_μ ) be an atomless infinite measure space and exponents  p()𝑝p(\cdot)italic_p ( ⋅ ) and q()𝑞q(\cdot)italic_q ( ⋅ ). If the inclusion  Lp()(μ)Lq()(μ)superscript𝐿𝑝𝜇superscript𝐿𝑞𝜇{L^{p(\cdot)}(\mu)}\hookrightarrow{L^{q(\cdot)}(\mu)}italic_L start_POSTSUPERSCRIPT italic_p ( ⋅ ) end_POSTSUPERSCRIPT ( italic_μ ) ↪ italic_L start_POSTSUPERSCRIPT italic_q ( ⋅ ) end_POSTSUPERSCRIPT ( italic_μ )  holds and  p+<superscript𝑝p^{+}<\inftyitalic_p start_POSTSUPERSCRIPT + end_POSTSUPERSCRIPT < ∞, then the inclusion is non-DSS.

Proof.

We shall proceed in a similar way as in [23] Thm 3.4. It is enough to find a disjoint sequence (fn)subscript𝑓𝑛(f_{n})( italic_f start_POSTSUBSCRIPT italic_n end_POSTSUBSCRIPT ) generating the same infinite dimensional (closed) subspace in Lp()(μ)superscript𝐿𝑝𝜇{L^{p(\cdot)}(\mu)}italic_L start_POSTSUPERSCRIPT italic_p ( ⋅ ) end_POSTSUPERSCRIPT ( italic_μ ) as well as in Lq()(μ)superscript𝐿𝑞𝜇{L^{q(\cdot)}(\mu)}italic_L start_POSTSUPERSCRIPT italic_q ( ⋅ ) end_POSTSUPERSCRIPT ( italic_μ ). To do so, if we could take functions   fn:=χAnassignsubscript𝑓𝑛subscript𝜒subscript𝐴𝑛f_{n}:=\chi_{A_{n}}italic_f start_POSTSUBSCRIPT italic_n end_POSTSUBSCRIPT := italic_χ start_POSTSUBSCRIPT italic_A start_POSTSUBSCRIPT italic_n end_POSTSUBSCRIPT end_POSTSUBSCRIPT, where (An)subscript𝐴𝑛(A_{n})( italic_A start_POSTSUBSCRIPT italic_n end_POSTSUBSCRIPT ) is a disjoint sequence with μ(An)=1𝜇subscript𝐴𝑛1\mu(A_{n})=1italic_μ ( italic_A start_POSTSUBSCRIPT italic_n end_POSTSUBSCRIPT ) = 1 verifying that    p|An+q|An<1np^{+}_{|_{A_{n}}}-q^{-}_{|_{A_{n}}}<\frac{1}{n}italic_p start_POSTSUPERSCRIPT + end_POSTSUPERSCRIPT start_POSTSUBSCRIPT | start_POSTSUBSCRIPT italic_A start_POSTSUBSCRIPT italic_n end_POSTSUBSCRIPT end_POSTSUBSCRIPT end_POSTSUBSCRIPT - italic_q start_POSTSUPERSCRIPT - end_POSTSUPERSCRIPT start_POSTSUBSCRIPT | start_POSTSUBSCRIPT italic_A start_POSTSUBSCRIPT italic_n end_POSTSUBSCRIPT end_POSTSUBSCRIPT end_POSTSUBSCRIPT < divide start_ARG 1 end_ARG start_ARG italic_n end_ARG, the proof would be finished.

Indeed, under these hypotheses we have the inclusions

p|An[fn]p()p|An+\ell_{p^{-}_{|_{A_{n}}}}\hookrightarrow[f_{n}]_{p(\cdot)}\hookrightarrow\ell_{% p^{+}_{|_{A_{n}}}}roman_ℓ start_POSTSUBSCRIPT italic_p start_POSTSUPERSCRIPT - end_POSTSUPERSCRIPT start_POSTSUBSCRIPT | start_POSTSUBSCRIPT italic_A start_POSTSUBSCRIPT italic_n end_POSTSUBSCRIPT end_POSTSUBSCRIPT end_POSTSUBSCRIPT end_POSTSUBSCRIPT ↪ [ italic_f start_POSTSUBSCRIPT italic_n end_POSTSUBSCRIPT ] start_POSTSUBSCRIPT italic_p ( ⋅ ) end_POSTSUBSCRIPT ↪ roman_ℓ start_POSTSUBSCRIPT italic_p start_POSTSUPERSCRIPT + end_POSTSUPERSCRIPT start_POSTSUBSCRIPT | start_POSTSUBSCRIPT italic_A start_POSTSUBSCRIPT italic_n end_POSTSUBSCRIPT end_POSTSUBSCRIPT end_POSTSUBSCRIPT end_POSTSUBSCRIPT

and

q|An[fn]q()q|An+.\ell_{q^{-}_{|_{A_{n}}}}\hookrightarrow[f_{n}]_{q(\cdot)}\hookrightarrow\ell_{% q^{+}_{|_{A_{n}}}}.roman_ℓ start_POSTSUBSCRIPT italic_q start_POSTSUPERSCRIPT - end_POSTSUPERSCRIPT start_POSTSUBSCRIPT | start_POSTSUBSCRIPT italic_A start_POSTSUBSCRIPT italic_n end_POSTSUBSCRIPT end_POSTSUBSCRIPT end_POSTSUBSCRIPT end_POSTSUBSCRIPT ↪ [ italic_f start_POSTSUBSCRIPT italic_n end_POSTSUBSCRIPT ] start_POSTSUBSCRIPT italic_q ( ⋅ ) end_POSTSUBSCRIPT ↪ roman_ℓ start_POSTSUBSCRIPT italic_q start_POSTSUPERSCRIPT + end_POSTSUPERSCRIPT start_POSTSUBSCRIPT | start_POSTSUBSCRIPT italic_A start_POSTSUBSCRIPT italic_n end_POSTSUBSCRIPT end_POSTSUBSCRIPT end_POSTSUBSCRIPT end_POSTSUBSCRIPT .

Using now Proposition 2.2 we have that the Nakano sequence spaces   p|Anp|An+q|Anq|An+\ell_{p^{-}_{|_{A_{n}}}}\cong\ell_{p^{+}_{|_{A_{n}}}}\cong\ell_{q^{-}_{|_{A_{n% }}}}\cong\ell_{q^{+}_{|_{A_{n}}}}roman_ℓ start_POSTSUBSCRIPT italic_p start_POSTSUPERSCRIPT - end_POSTSUPERSCRIPT start_POSTSUBSCRIPT | start_POSTSUBSCRIPT italic_A start_POSTSUBSCRIPT italic_n end_POSTSUBSCRIPT end_POSTSUBSCRIPT end_POSTSUBSCRIPT end_POSTSUBSCRIPT ≅ roman_ℓ start_POSTSUBSCRIPT italic_p start_POSTSUPERSCRIPT + end_POSTSUPERSCRIPT start_POSTSUBSCRIPT | start_POSTSUBSCRIPT italic_A start_POSTSUBSCRIPT italic_n end_POSTSUBSCRIPT end_POSTSUBSCRIPT end_POSTSUBSCRIPT end_POSTSUBSCRIPT ≅ roman_ℓ start_POSTSUBSCRIPT italic_q start_POSTSUPERSCRIPT - end_POSTSUPERSCRIPT start_POSTSUBSCRIPT | start_POSTSUBSCRIPT italic_A start_POSTSUBSCRIPT italic_n end_POSTSUBSCRIPT end_POSTSUBSCRIPT end_POSTSUBSCRIPT end_POSTSUBSCRIPT ≅ roman_ℓ start_POSTSUBSCRIPT italic_q start_POSTSUPERSCRIPT + end_POSTSUPERSCRIPT start_POSTSUBSCRIPT | start_POSTSUBSCRIPT italic_A start_POSTSUBSCRIPT italic_n end_POSTSUBSCRIPT end_POSTSUBSCRIPT end_POSTSUBSCRIPT end_POSTSUBSCRIPT. Hence, we deduce that   [fn]p()[fn]q()subscriptdelimited-[]subscript𝑓𝑛𝑝subscriptdelimited-[]subscript𝑓𝑛𝑞[f_{n}]_{p(\cdot)}\cong\,[f_{n}]_{q(\cdot)}[ italic_f start_POSTSUBSCRIPT italic_n end_POSTSUBSCRIPT ] start_POSTSUBSCRIPT italic_p ( ⋅ ) end_POSTSUBSCRIPT ≅ [ italic_f start_POSTSUBSCRIPT italic_n end_POSTSUBSCRIPT ] start_POSTSUBSCRIPT italic_q ( ⋅ ) end_POSTSUBSCRIPT.

Let us now construct such disjoint sequence (An)subscript𝐴𝑛(A_{n})( italic_A start_POSTSUBSCRIPT italic_n end_POSTSUBSCRIPT ) with the above properties. We remarked above that for every natural n𝑛nitalic_n  we have μ(D12n)=𝜇subscript𝐷12𝑛\mu(D_{\frac{1}{2n}})=\inftyitalic_μ ( italic_D start_POSTSUBSCRIPT divide start_ARG 1 end_ARG start_ARG 2 italic_n end_ARG end_POSTSUBSCRIPT ) = ∞   (or either p()|Aq()|Aevaluated-at𝑝𝐴evaluated-at𝑞𝐴p(\cdot)|_{A}\equiv q(\cdot)|_{A}italic_p ( ⋅ ) | start_POSTSUBSCRIPT italic_A end_POSTSUBSCRIPT ≡ italic_q ( ⋅ ) | start_POSTSUBSCRIPT italic_A end_POSTSUBSCRIPT over a positive measure subset AΩ𝐴ΩA\subset\Omegaitalic_A ⊂ roman_Ω and then the proof is trivial). Thus, if we make a finite partition  {[xi,xi+1)}subscript𝑥𝑖subscript𝑥𝑖1\{[x_{i},x_{i+1})\}{ [ italic_x start_POSTSUBSCRIPT italic_i end_POSTSUBSCRIPT , italic_x start_POSTSUBSCRIPT italic_i + 1 end_POSTSUBSCRIPT ) } of the interval  [1,p+)1superscript𝑝[1,p^{+})[ 1 , italic_p start_POSTSUPERSCRIPT + end_POSTSUPERSCRIPT )  where  1=x1<x2<<xk=p+1subscript𝑥1subscript𝑥2subscript𝑥𝑘superscript𝑝1=x_{1}<x_{2}<...<x_{k}=p^{+}1 = italic_x start_POSTSUBSCRIPT 1 end_POSTSUBSCRIPT < italic_x start_POSTSUBSCRIPT 2 end_POSTSUBSCRIPT < … < italic_x start_POSTSUBSCRIPT italic_k end_POSTSUBSCRIPT = italic_p start_POSTSUPERSCRIPT + end_POSTSUPERSCRIPT  and  xi+1xi<12nsubscript𝑥𝑖1subscript𝑥𝑖12𝑛x_{i+1}-x_{i}<\frac{1}{2n}italic_x start_POSTSUBSCRIPT italic_i + 1 end_POSTSUBSCRIPT - italic_x start_POSTSUBSCRIPT italic_i end_POSTSUBSCRIPT < divide start_ARG 1 end_ARG start_ARG 2 italic_n end_ARG  for every natural i𝑖iitalic_i, we can assure that, for some j𝑗jitalic_j,

μ(D12np1([xj,xj+1)))=.𝜇subscript𝐷12𝑛superscript𝑝1subscript𝑥𝑗subscript𝑥𝑗1\mu\left(D_{\frac{1}{2n}}\cap p^{-1}\left([x_{j},x_{j+1})\right)\right)=\infty.italic_μ ( italic_D start_POSTSUBSCRIPT divide start_ARG 1 end_ARG start_ARG 2 italic_n end_ARG end_POSTSUBSCRIPT ∩ italic_p start_POSTSUPERSCRIPT - 1 end_POSTSUPERSCRIPT ( [ italic_x start_POSTSUBSCRIPT italic_j end_POSTSUBSCRIPT , italic_x start_POSTSUBSCRIPT italic_j + 1 end_POSTSUBSCRIPT ) ) ) = ∞ .

Even more, by the definition of  D12nsubscript𝐷12𝑛D_{\frac{1}{2n}}italic_D start_POSTSUBSCRIPT divide start_ARG 1 end_ARG start_ARG 2 italic_n end_ARG end_POSTSUBSCRIPT, it is also true that

μ(D12np1([xj,xj+1))q1([xj12n,xj+1)))=.𝜇subscript𝐷12𝑛superscript𝑝1subscript𝑥𝑗subscript𝑥𝑗1superscript𝑞1subscript𝑥𝑗12𝑛subscript𝑥𝑗1\mu\left(D_{\frac{1}{2n}}\cap p^{-1}\left([x_{j},x_{j+1})\right)\cap q^{-1}([x% _{j}-\frac{1}{2n},x_{j+1}))\right)=\infty.italic_μ ( italic_D start_POSTSUBSCRIPT divide start_ARG 1 end_ARG start_ARG 2 italic_n end_ARG end_POSTSUBSCRIPT ∩ italic_p start_POSTSUPERSCRIPT - 1 end_POSTSUPERSCRIPT ( [ italic_x start_POSTSUBSCRIPT italic_j end_POSTSUBSCRIPT , italic_x start_POSTSUBSCRIPT italic_j + 1 end_POSTSUBSCRIPT ) ) ∩ italic_q start_POSTSUPERSCRIPT - 1 end_POSTSUPERSCRIPT ( [ italic_x start_POSTSUBSCRIPT italic_j end_POSTSUBSCRIPT - divide start_ARG 1 end_ARG start_ARG 2 italic_n end_ARG , italic_x start_POSTSUBSCRIPT italic_j + 1 end_POSTSUBSCRIPT ) ) ) = ∞ .

Thus, we conclude that there exists a set Ensubscript𝐸𝑛E_{n}italic_E start_POSTSUBSCRIPT italic_n end_POSTSUBSCRIPT (the above set) with infinite measure such that

pEn+qEnxj+1(xj12n)1n.subscriptsuperscript𝑝subscript𝐸𝑛subscriptsuperscript𝑞subscript𝐸𝑛subscript𝑥𝑗1subscript𝑥𝑗12𝑛1𝑛p^{+}_{E_{n}}-q^{-}_{E_{n}}\leq x_{j+1}-(x_{j}-\frac{1}{2n})\,\leq\,\frac{1}{n}.italic_p start_POSTSUPERSCRIPT + end_POSTSUPERSCRIPT start_POSTSUBSCRIPT italic_E start_POSTSUBSCRIPT italic_n end_POSTSUBSCRIPT end_POSTSUBSCRIPT - italic_q start_POSTSUPERSCRIPT - end_POSTSUPERSCRIPT start_POSTSUBSCRIPT italic_E start_POSTSUBSCRIPT italic_n end_POSTSUBSCRIPT end_POSTSUBSCRIPT ≤ italic_x start_POSTSUBSCRIPT italic_j + 1 end_POSTSUBSCRIPT - ( italic_x start_POSTSUBSCRIPT italic_j end_POSTSUBSCRIPT - divide start_ARG 1 end_ARG start_ARG 2 italic_n end_ARG ) ≤ divide start_ARG 1 end_ARG start_ARG italic_n end_ARG .

Now, since μ(En)=𝜇subscript𝐸𝑛\mu(E_{n})=\inftyitalic_μ ( italic_E start_POSTSUBSCRIPT italic_n end_POSTSUBSCRIPT ) = ∞  for each natural n𝑛nitalic_n, we can take  An(En(i=1n1Ai)c)subscript𝐴𝑛subscript𝐸𝑛superscriptsuperscriptsubscript𝑖1𝑛1subscript𝐴𝑖𝑐A_{n}\subset\left(E_{n}\cap(\bigcup_{i=1}^{n-1}A_{i})^{c}\right)italic_A start_POSTSUBSCRIPT italic_n end_POSTSUBSCRIPT ⊂ ( italic_E start_POSTSUBSCRIPT italic_n end_POSTSUBSCRIPT ∩ ( ⋃ start_POSTSUBSCRIPT italic_i = 1 end_POSTSUBSCRIPT start_POSTSUPERSCRIPT italic_n - 1 end_POSTSUPERSCRIPT italic_A start_POSTSUBSCRIPT italic_i end_POSTSUBSCRIPT ) start_POSTSUPERSCRIPT italic_c end_POSTSUPERSCRIPT )  with μ(An)=1𝜇subscript𝐴𝑛1\mu(A_{n})=1italic_μ ( italic_A start_POSTSUBSCRIPT italic_n end_POSTSUBSCRIPT ) = 1 getting so the needed disjoint sequence (An)subscript𝐴𝑛(A_{n})( italic_A start_POSTSUBSCRIPT italic_n end_POSTSUBSCRIPT ). ∎

Proposition 6.2.

Let (Ω,μ)Ω𝜇(\Omega,\mu)( roman_Ω , italic_μ ) be an atomless infinite measure space and exponents  p()𝑝p(\cdot)italic_p ( ⋅ ) and  q()𝑞q(\cdot)italic_q ( ⋅ ). If the inclusion  Lp()(μ)Lq()(μ)superscript𝐿𝑝𝜇superscript𝐿𝑞𝜇{L^{p(\cdot)}(\mu)}\hookrightarrow{L^{q(\cdot)}(\mu)}italic_L start_POSTSUPERSCRIPT italic_p ( ⋅ ) end_POSTSUPERSCRIPT ( italic_μ ) ↪ italic_L start_POSTSUPERSCRIPT italic_q ( ⋅ ) end_POSTSUPERSCRIPT ( italic_μ )  holds and the distribution function of  q()𝑞q(\cdot)italic_q ( ⋅ )  verifies  μq()(n)=subscript𝜇𝑞𝑛\mu_{q(\cdot)}(n)=\inftyitalic_μ start_POSTSUBSCRIPT italic_q ( ⋅ ) end_POSTSUBSCRIPT ( italic_n ) = ∞  for every natural n𝑛nitalic_n, then the inclusion is not DSS.

Proof.

Consider the sets

Dn={tΩ:q(t)>n}En={tΩ:p(t)>n}.subscript𝐷𝑛conditional-set𝑡Ω𝑞𝑡𝑛subscript𝐸𝑛conditional-set𝑡Ω𝑝𝑡𝑛D_{n}=\{\,t\in\Omega\,\,:\,\,q(t)>n\,\}\,\subset\,E_{n}=\{\,t\in\Omega\,\,:\,% \,p(t)>n\,\}.italic_D start_POSTSUBSCRIPT italic_n end_POSTSUBSCRIPT = { italic_t ∈ roman_Ω : italic_q ( italic_t ) > italic_n } ⊂ italic_E start_POSTSUBSCRIPT italic_n end_POSTSUBSCRIPT = { italic_t ∈ roman_Ω : italic_p ( italic_t ) > italic_n } .

Since  μ(Dn)=𝜇subscript𝐷𝑛\mu(D_{n})=\inftyitalic_μ ( italic_D start_POSTSUBSCRIPT italic_n end_POSTSUBSCRIPT ) = ∞, the increasing sequence  (Dn\Em)msubscript\subscript𝐷𝑛subscript𝐸𝑚𝑚(D_{n}\backslash E_{m})_{m}( italic_D start_POSTSUBSCRIPT italic_n end_POSTSUBSCRIPT \ italic_E start_POSTSUBSCRIPT italic_m end_POSTSUBSCRIPT ) start_POSTSUBSCRIPT italic_m end_POSTSUBSCRIPT  verifies  μ(Dn\Em)𝜇\subscript𝐷𝑛subscript𝐸𝑚\mu(D_{n}\backslash E_{m})\nearrow\inftyitalic_μ ( italic_D start_POSTSUBSCRIPT italic_n end_POSTSUBSCRIPT \ italic_E start_POSTSUBSCRIPT italic_m end_POSTSUBSCRIPT ) ↗ ∞ as m𝑚m\rightarrow\inftyitalic_m → ∞.

Hence, by the hypotheses, it is possible to find a strictly increasing sequence  (nk)ksubscriptsubscript𝑛𝑘𝑘(n_{k})_{k}( italic_n start_POSTSUBSCRIPT italic_k end_POSTSUBSCRIPT ) start_POSTSUBSCRIPT italic_k end_POSTSUBSCRIPT   of natural numbers and a disjoint sequence of measurable sets  (Ak)ksubscriptsubscript𝐴𝑘𝑘(A_{k})_{k}( italic_A start_POSTSUBSCRIPT italic_k end_POSTSUBSCRIPT ) start_POSTSUBSCRIPT italic_k end_POSTSUBSCRIPT  with  μ(Ak)=1𝜇subscript𝐴𝑘1\mu(A_{k})=1italic_μ ( italic_A start_POSTSUBSCRIPT italic_k end_POSTSUBSCRIPT ) = 1   such that

nkq|Akp|Akandq|Ak+p|Ak+nk+1.n_{k}\leq q^{-}_{|A_{k}}\leq p^{-}_{|A_{k}}\qquad\text{and}\qquad q^{+}_{|A_{k% }}\leq p^{+}_{|A_{k}}\leq n_{k+1}.italic_n start_POSTSUBSCRIPT italic_k end_POSTSUBSCRIPT ≤ italic_q start_POSTSUPERSCRIPT - end_POSTSUPERSCRIPT start_POSTSUBSCRIPT | italic_A start_POSTSUBSCRIPT italic_k end_POSTSUBSCRIPT end_POSTSUBSCRIPT ≤ italic_p start_POSTSUPERSCRIPT - end_POSTSUPERSCRIPT start_POSTSUBSCRIPT | italic_A start_POSTSUBSCRIPT italic_k end_POSTSUBSCRIPT end_POSTSUBSCRIPT and italic_q start_POSTSUPERSCRIPT + end_POSTSUPERSCRIPT start_POSTSUBSCRIPT | italic_A start_POSTSUBSCRIPT italic_k end_POSTSUBSCRIPT end_POSTSUBSCRIPT ≤ italic_p start_POSTSUPERSCRIPT + end_POSTSUPERSCRIPT start_POSTSUBSCRIPT | italic_A start_POSTSUBSCRIPT italic_k end_POSTSUBSCRIPT end_POSTSUBSCRIPT ≤ italic_n start_POSTSUBSCRIPT italic_k + 1 end_POSTSUBSCRIPT .

Consider now the (closed) subspace  [χAk]p()subscriptdelimited-[]subscript𝜒subscript𝐴𝑘𝑝[\chi_{A_{k}}]_{p(\cdot)}[ italic_χ start_POSTSUBSCRIPT italic_A start_POSTSUBSCRIPT italic_k end_POSTSUBSCRIPT end_POSTSUBSCRIPT ] start_POSTSUBSCRIPT italic_p ( ⋅ ) end_POSTSUBSCRIPT, formed by all functions of the form  kλkχAkLp()(μ)subscript𝑘subscript𝜆𝑘subscript𝜒subscript𝐴𝑘superscript𝐿𝑝𝜇\sum_{k}\lambda_{k}\chi_{A_{k}}\in{L^{p(\cdot)}(\mu)}∑ start_POSTSUBSCRIPT italic_k end_POSTSUBSCRIPT italic_λ start_POSTSUBSCRIPT italic_k end_POSTSUBSCRIPT italic_χ start_POSTSUBSCRIPT italic_A start_POSTSUBSCRIPT italic_k end_POSTSUBSCRIPT end_POSTSUBSCRIPT ∈ italic_L start_POSTSUPERSCRIPT italic_p ( ⋅ ) end_POSTSUPERSCRIPT ( italic_μ )  such that ρp()(λk>NλkχAk)N0𝑁subscript𝜌𝑝𝜆subscript𝑘𝑁subscript𝜆𝑘subscript𝜒subscript𝐴𝑘0\rho_{p(\cdot)}(\lambda\sum_{k>N}\lambda_{k}\chi_{A_{k}})\xrightarrow{{N% \rightarrow\infty}}0italic_ρ start_POSTSUBSCRIPT italic_p ( ⋅ ) end_POSTSUBSCRIPT ( italic_λ ∑ start_POSTSUBSCRIPT italic_k > italic_N end_POSTSUBSCRIPT italic_λ start_POSTSUBSCRIPT italic_k end_POSTSUBSCRIPT italic_χ start_POSTSUBSCRIPT italic_A start_POSTSUBSCRIPT italic_k end_POSTSUBSCRIPT end_POSTSUBSCRIPT ) start_ARROW start_OVERACCENT italic_N → ∞ end_OVERACCENT → end_ARROW 0 for every λ>0𝜆0\lambda>0italic_λ > 0.

If  kλkχAkLp()(μ)subscript𝑘subscript𝜆𝑘subscript𝜒subscript𝐴𝑘superscript𝐿𝑝𝜇\sum_{k}\lambda_{k}\chi_{A_{k}}\in{L^{p(\cdot)}(\mu)}∑ start_POSTSUBSCRIPT italic_k end_POSTSUBSCRIPT italic_λ start_POSTSUBSCRIPT italic_k end_POSTSUBSCRIPT italic_χ start_POSTSUBSCRIPT italic_A start_POSTSUBSCRIPT italic_k end_POSTSUBSCRIPT end_POSTSUBSCRIPT ∈ italic_L start_POSTSUPERSCRIPT italic_p ( ⋅ ) end_POSTSUPERSCRIPT ( italic_μ ), then, for some r>0𝑟0r>0italic_r > 0,

k|λkr|nk+1k|λkr|p|Ak+ρp()(kλkχAkr)k|λkr|p|Akk|λkr|nk.\sum_{k}|\frac{\lambda_{k}}{r}|^{n_{k+1}}\leq\sum_{k}|\frac{\lambda_{k}}{r}|^{% p^{+}_{|A_{k}}}\leq\rho_{p(\cdot)}\big{(}\frac{\sum_{k}\lambda_{k}\chi_{A_{k}}% }{r}\big{)}\leq\sum_{k}|\frac{\lambda_{k}}{r}|^{p^{-}_{|A_{k}}}\leq\sum_{k}|% \frac{\lambda_{k}}{r}|^{n_{k}}.∑ start_POSTSUBSCRIPT italic_k end_POSTSUBSCRIPT | divide start_ARG italic_λ start_POSTSUBSCRIPT italic_k end_POSTSUBSCRIPT end_ARG start_ARG italic_r end_ARG | start_POSTSUPERSCRIPT italic_n start_POSTSUBSCRIPT italic_k + 1 end_POSTSUBSCRIPT end_POSTSUPERSCRIPT ≤ ∑ start_POSTSUBSCRIPT italic_k end_POSTSUBSCRIPT | divide start_ARG italic_λ start_POSTSUBSCRIPT italic_k end_POSTSUBSCRIPT end_ARG start_ARG italic_r end_ARG | start_POSTSUPERSCRIPT italic_p start_POSTSUPERSCRIPT + end_POSTSUPERSCRIPT start_POSTSUBSCRIPT | italic_A start_POSTSUBSCRIPT italic_k end_POSTSUBSCRIPT end_POSTSUBSCRIPT end_POSTSUPERSCRIPT ≤ italic_ρ start_POSTSUBSCRIPT italic_p ( ⋅ ) end_POSTSUBSCRIPT ( divide start_ARG ∑ start_POSTSUBSCRIPT italic_k end_POSTSUBSCRIPT italic_λ start_POSTSUBSCRIPT italic_k end_POSTSUBSCRIPT italic_χ start_POSTSUBSCRIPT italic_A start_POSTSUBSCRIPT italic_k end_POSTSUBSCRIPT end_POSTSUBSCRIPT end_ARG start_ARG italic_r end_ARG ) ≤ ∑ start_POSTSUBSCRIPT italic_k end_POSTSUBSCRIPT | divide start_ARG italic_λ start_POSTSUBSCRIPT italic_k end_POSTSUBSCRIPT end_ARG start_ARG italic_r end_ARG | start_POSTSUPERSCRIPT italic_p start_POSTSUPERSCRIPT - end_POSTSUPERSCRIPT start_POSTSUBSCRIPT | italic_A start_POSTSUBSCRIPT italic_k end_POSTSUBSCRIPT end_POSTSUBSCRIPT end_POSTSUPERSCRIPT ≤ ∑ start_POSTSUBSCRIPT italic_k end_POSTSUBSCRIPT | divide start_ARG italic_λ start_POSTSUBSCRIPT italic_k end_POSTSUBSCRIPT end_ARG start_ARG italic_r end_ARG | start_POSTSUPERSCRIPT italic_n start_POSTSUBSCRIPT italic_k end_POSTSUBSCRIPT end_POSTSUPERSCRIPT .

Hence, since the Nakano sequence spaces   (nk)(nk+1)subscriptsubscript𝑛𝑘subscriptsubscriptsubscript𝑛𝑘1\ell_{(n_{k})}\cong\ell_{\infty}\cong\ell_{(n_{k+1})}roman_ℓ start_POSTSUBSCRIPT ( italic_n start_POSTSUBSCRIPT italic_k end_POSTSUBSCRIPT ) end_POSTSUBSCRIPT ≅ roman_ℓ start_POSTSUBSCRIPT ∞ end_POSTSUBSCRIPT ≅ roman_ℓ start_POSTSUBSCRIPT ( italic_n start_POSTSUBSCRIPT italic_k + 1 end_POSTSUBSCRIPT ) end_POSTSUBSCRIPT, we deduce that the subspace  [χAk]p()c0subscriptdelimited-[]subscript𝜒subscript𝐴𝑘𝑝subscript𝑐0[\chi_{A_{k}}]_{p(\cdot)}\cong c_{0}[ italic_χ start_POSTSUBSCRIPT italic_A start_POSTSUBSCRIPT italic_k end_POSTSUBSCRIPT end_POSTSUBSCRIPT ] start_POSTSUBSCRIPT italic_p ( ⋅ ) end_POSTSUBSCRIPT ≅ italic_c start_POSTSUBSCRIPT 0 end_POSTSUBSCRIPT.

Analogously the corresponding subspace defined similarly  [χAk]q()subscriptdelimited-[]subscript𝜒subscript𝐴𝑘𝑞[\chi_{A_{k}}]_{q(\cdot)}[ italic_χ start_POSTSUBSCRIPT italic_A start_POSTSUBSCRIPT italic_k end_POSTSUBSCRIPT end_POSTSUBSCRIPT ] start_POSTSUBSCRIPT italic_q ( ⋅ ) end_POSTSUBSCRIPT  in Lq()(μ)superscript𝐿𝑞𝜇{L^{q(\cdot)}(\mu)}italic_L start_POSTSUPERSCRIPT italic_q ( ⋅ ) end_POSTSUPERSCRIPT ( italic_μ ) satisfies that   [χAk]q()c0subscriptdelimited-[]subscript𝜒subscript𝐴𝑘𝑞subscript𝑐0[\chi_{A_{k}}]_{q(\cdot)}\cong c_{0}[ italic_χ start_POSTSUBSCRIPT italic_A start_POSTSUBSCRIPT italic_k end_POSTSUBSCRIPT end_POSTSUBSCRIPT ] start_POSTSUBSCRIPT italic_q ( ⋅ ) end_POSTSUBSCRIPT ≅ italic_c start_POSTSUBSCRIPT 0 end_POSTSUBSCRIPT. Therefore we conclude that   [χAk]p()[χAk]q()c0subscriptdelimited-[]subscript𝜒subscript𝐴𝑘𝑝subscriptdelimited-[]subscript𝜒subscript𝐴𝑘𝑞subscript𝑐0[\chi_{A_{k}}]_{p(\cdot)}\cong[\chi_{A_{k}}]_{q(\cdot)}\cong c_{0}[ italic_χ start_POSTSUBSCRIPT italic_A start_POSTSUBSCRIPT italic_k end_POSTSUBSCRIPT end_POSTSUBSCRIPT ] start_POSTSUBSCRIPT italic_p ( ⋅ ) end_POSTSUBSCRIPT ≅ [ italic_χ start_POSTSUBSCRIPT italic_A start_POSTSUBSCRIPT italic_k end_POSTSUBSCRIPT end_POSTSUBSCRIPT ] start_POSTSUBSCRIPT italic_q ( ⋅ ) end_POSTSUBSCRIPT ≅ italic_c start_POSTSUBSCRIPT 0 end_POSTSUBSCRIPT. ∎

Combining now the two above propositions we cover the different cases for the the following statement (notice that the case   q+<p+=superscript𝑞superscript𝑝q^{+}<p^{+}=\inftyitalic_q start_POSTSUPERSCRIPT + end_POSTSUPERSCRIPT < italic_p start_POSTSUPERSCRIPT + end_POSTSUPERSCRIPT = ∞  and μp()(n)=subscript𝜇𝑝𝑛\mu_{p(\cdot)}(n)=\inftyitalic_μ start_POSTSUBSCRIPT italic_p ( ⋅ ) end_POSTSUBSCRIPT ( italic_n ) = ∞ for every natural n𝑛nitalic_n is not possible according with Proposition 2.1).

Theorem 6.3.

Let (Ω,μ)Ω𝜇(\Omega,\mu)( roman_Ω , italic_μ ) be a non-atomic infinite measure space and exponents  p()𝑝p(\cdot)italic_p ( ⋅ ) and q()𝑞q(\cdot)italic_q ( ⋅ ). If the inclusion  Lp()(μ)Lq()(μ)superscript𝐿𝑝𝜇superscript𝐿𝑞𝜇{L^{p(\cdot)}(\mu)}\hookrightarrow{L^{q(\cdot)}(\mu)}italic_L start_POSTSUPERSCRIPT italic_p ( ⋅ ) end_POSTSUPERSCRIPT ( italic_μ ) ↪ italic_L start_POSTSUPERSCRIPT italic_q ( ⋅ ) end_POSTSUPERSCRIPT ( italic_μ )  holds, then it is not DSS.

Proof.

In the case of the existence of a subset  Ω0ΩsubscriptΩ0Ω\Omega_{0}\subset\Omegaroman_Ω start_POSTSUBSCRIPT 0 end_POSTSUBSCRIPT ⊂ roman_Ω   with   μ(Ω0)=𝜇subscriptΩ0\mu(\Omega_{0})=\inftyitalic_μ ( roman_Ω start_POSTSUBSCRIPT 0 end_POSTSUBSCRIPT ) = ∞  and   p|Ω0+<p^{+}_{|\Omega_{0}}<\inftyitalic_p start_POSTSUPERSCRIPT + end_POSTSUPERSCRIPT start_POSTSUBSCRIPT | roman_Ω start_POSTSUBSCRIPT 0 end_POSTSUBSCRIPT end_POSTSUBSCRIPT < ∞, the above Proposition 6.1 gives the result. For the other possible cases we can use now Proposition 6.2. ∎

Remark 6.4.

The non-DSS property of inclusions between variable Lebesgue spaces on infinite measures is not true in the class of Orlicz spaces. For example the inclusions   Lφ(0,)Lψ(0,)superscript𝐿𝜑0superscript𝐿𝜓0L^{\varphi}(0,\infty)\hookrightarrow L^{\psi}(0,\infty)italic_L start_POSTSUPERSCRIPT italic_φ end_POSTSUPERSCRIPT ( 0 , ∞ ) ↪ italic_L start_POSTSUPERSCRIPT italic_ψ end_POSTSUPERSCRIPT ( 0 , ∞ )  are DSS for the Orlicz functions  φ(x)=xrxs𝜑𝑥superscript𝑥𝑟superscript𝑥𝑠\varphi(x)=x^{r}\vee x^{s}italic_φ ( italic_x ) = italic_x start_POSTSUPERSCRIPT italic_r end_POSTSUPERSCRIPT ∨ italic_x start_POSTSUPERSCRIPT italic_s end_POSTSUPERSCRIPT  and  ψ(x)=xpxq𝜓𝑥superscript𝑥𝑝superscript𝑥𝑞\psi(x)=x^{p}\vee x^{q}italic_ψ ( italic_x ) = italic_x start_POSTSUPERSCRIPT italic_p end_POSTSUPERSCRIPT ∨ italic_x start_POSTSUPERSCRIPT italic_q end_POSTSUPERSCRIPT, (r<pq<s𝑟𝑝𝑞𝑠r<p\leq q<sitalic_r < italic_p ≤ italic_q < italic_s) (cf. [14] Ex. 4.9).

Remark 6.5.

Note that also for infinite measures if the inclusion   L(μ)Lp()(μ)superscript𝐿𝜇superscript𝐿𝑝𝜇L^{\infty}(\mu)\hookrightarrow{L^{p(\cdot)}(\mu)}italic_L start_POSTSUPERSCRIPT ∞ end_POSTSUPERSCRIPT ( italic_μ ) ↪ italic_L start_POSTSUPERSCRIPT italic_p ( ⋅ ) end_POSTSUPERSCRIPT ( italic_μ )   holds then it is non-DSS.

Indeed, assume that for an exponent function p()𝑝p(\cdot)italic_p ( ⋅ ) the inclusion   L(μ)Lp()(μ)superscript𝐿𝜇superscript𝐿𝑝𝜇L^{\infty}(\mu)\hookrightarrow{L^{p(\cdot)}(\mu)}italic_L start_POSTSUPERSCRIPT ∞ end_POSTSUPERSCRIPT ( italic_μ ) ↪ italic_L start_POSTSUPERSCRIPT italic_p ( ⋅ ) end_POSTSUPERSCRIPT ( italic_μ )   holds. Then,   χΩLp()(μ)subscript𝜒Ωsuperscript𝐿𝑝𝜇\chi_{\Omega}\in{L^{p(\cdot)}(\mu)}italic_χ start_POSTSUBSCRIPT roman_Ω end_POSTSUBSCRIPT ∈ italic_L start_POSTSUPERSCRIPT italic_p ( ⋅ ) end_POSTSUPERSCRIPT ( italic_μ ), so there exists   λ>1𝜆1\lambda>1italic_λ > 1  such that  Ωdμλp(t)<subscriptΩ𝑑𝜇superscript𝜆𝑝𝑡\int_{\Omega}\frac{d\mu}{\lambda^{p(t)}}\,<\infty∫ start_POSTSUBSCRIPT roman_Ω end_POSTSUBSCRIPT divide start_ARG italic_d italic_μ end_ARG start_ARG italic_λ start_POSTSUPERSCRIPT italic_p ( italic_t ) end_POSTSUPERSCRIPT end_ARG < ∞. It follows that the sets  Dn={tΩ:p(t)>n}subscript𝐷𝑛conditional-set𝑡Ω𝑝𝑡𝑛D_{n}=\{t\in\Omega:p(t)>n\}italic_D start_POSTSUBSCRIPT italic_n end_POSTSUBSCRIPT = { italic_t ∈ roman_Ω : italic_p ( italic_t ) > italic_n }  has infinite measure for every natural n𝑛nitalic_n (since   μ(Dnc)<)\mu(D_{n}^{c})<\infty)italic_μ ( italic_D start_POSTSUBSCRIPT italic_n end_POSTSUBSCRIPT start_POSTSUPERSCRIPT italic_c end_POSTSUPERSCRIPT ) < ∞ ). Now reasoning as in above Proposition 6.2, we can easily deduce that the inclusion  L(μ)Lp()(μ)superscript𝐿𝜇superscript𝐿𝑝𝜇L^{\infty}(\mu)\hookrightarrow{L^{p(\cdot)}(\mu)}italic_L start_POSTSUPERSCRIPT ∞ end_POSTSUPERSCRIPT ( italic_μ ) ↪ italic_L start_POSTSUPERSCRIPT italic_p ( ⋅ ) end_POSTSUPERSCRIPT ( italic_μ )  is non-DSS.

References

  • [1] F. Albiac and N.J. Kalton; Topics in Banach space theory (2nd ed.). Springer, 2016.
  • [2] Ch. Aliprantis and O. Burkinshaw; Positive operators. Academic Press, 1985.
  • [3] S.V. Astashkin; The Rademacher system in function spaces. Birkhäuser, 2020.
  • [4] S.V. Astashkin, F.L. Hernández and E.M. Semenov; Strictly singular inclusions of rearrangement invariant spaces and Rademacher spaces. Studia Math. 193 (2009), 269-283.
  • [5] S.V. Astashkin and E.M. Semenov; Some properties of embeddings of rearrangement invariant spaces. Sbornik Math. 210:10 (2019), 1361-1379.
  • [6] C. Bennett and R.C. Sharpley; Interpolation of operators. Pure and Applied Mathematics 129. Academic Press, 1988.
  • [7] D.V. Cruz-Uribe and A. Fiorenza; Variable Lebesgue spaces: foundations and harmonic analysis. Birkhäuser/Springer, 2013.
  • [8] L. Diening, P. Harjulehto, P. Hästö and M. Ružička; Lebesgue and Sobolev spaces with variable exponents. Lecture Notes in Mathematics, vol. 2017. Springer, 2011.
  • [9] D.E. Edmunds, A. Gogatishvili and A. Nekvinda; Almost compact and compact embeddings of variable exponent spaces. Studia Math. 268 (2023), no. 2, 187-211.
  • [10] D.E. Edmunds, J. Lang and A. Nekvinda; On Lp(x)superscript𝐿𝑝𝑥L^{p(x)}italic_L start_POSTSUPERSCRIPT italic_p ( italic_x ) end_POSTSUPERSCRIPT norms. Proc. R. Soc. London A 255 (1999), 219-225.
  • [11] A. Fiorenza, A. Gogatishvili, A. Nekvinda and J. Rokotoson; Remarks on compactness results for variable exponent spaces Lp()superscript𝐿𝑝L^{p(\cdot)}italic_L start_POSTSUPERSCRIPT italic_p ( ⋅ ) end_POSTSUPERSCRIPT. J. Math. Pures Appl. 157 (2022), 136-144.
  • [12] J. Flores, F.L. Hernández, N.J. Kalton and P. Tradacete; Characterizations of strictly singular operators on Banach lattices. J. London Math. Soc. (2) 79 (2009), no. 3, 612-630.
  • [13] J. Flores, F.L. Hernández, C. Ruiz and M. Sanchiz; On the structure of variable exponent spaces. Indag. Math. (N.S.) 31 (2020), no. 5, 831-841.
  • [14] A. García del Amo, F.L. Hernández and C. Ruiz; Disjointly strictly singular operators and interpolation. Proc. Roy. Soc. Edinburgh Sect. A 126 (1996), no. 5, 1011-1026.
  • [15] A. García del Amo, F.L. Hernández, E. Semenov and V. Sánchez; Disjointly strictly singular inclusions between rearrangement invariant spaces. J. London Math. Soc. 62 (2000), 239-252.
  • [16] S. Goldberg; Unbounded linear operators: theory and applications. Dover, 1966.
  • [17] P. Górka and A. Macios; Almost everything you need to know about relatively compact sets in variable Lebesgue spaces. J. Funct. Anal. 269 (2015), 1925-1949.
  • [18] F.L. Hernández, S.Ya. Novikov and E.M. Semenov; Strictly singular embeddings between rearrangement invariant spaces. Positivity 7:1-2 (2003), 119-124.
  • [19] F.L. Hernández, Y. Raynaud and E.M. Semenov; Bernstein widths and superstrictly singular inclusions. Oper. Theory Adv. Appl. vol. 218 (2012), 359-376.
  • [20] F.L. Hernández and B. Rodríguez-Salinas; On psubscript𝑝\ell_{p}roman_ℓ start_POSTSUBSCRIPT italic_p end_POSTSUBSCRIPT-complemented copies in Orlicz spaces II. Israel J. Math. 68 (1989), 27-55.
  • [21]                                     ; Lattice embedding Lpsuperscript𝐿𝑝L^{p}italic_L start_POSTSUPERSCRIPT italic_p end_POSTSUPERSCRIPT into Orlicz spaces. Israel J. Math. 90 (1995), 167-188.
  • [22] F.L. Hernández, C. Ruiz and M. Sanchiz; Weak compactness in variable exponent spaces. J. Funct. Anal. 281 (2021), 109087.
  • [23]                                     ; Weak compactness and representation in variable exponent Lebesgue spaces on infinite measure spaces. Rev. Real Acad. Cienc. Exactas Fis. Nat. Ser. A-Mat, 116,152 (2022).
  • [24] M. Krasnoselskii and Ya. Rutickii; Convex functions and Orlicz spaces. Noordhoff, 1961.
  • [25] S. Krein, J. Petunin and E. Semenov; Interpolation of linear operators. Transl. Amer. Math. Soc., 1982.
  • [26] J. Lindenstrauss and L. Tzafriri; Classical Banach spaces I. Springer-Verlag, 1977.
  • [27]                                     ; Classical Banach spaces II. Springer-Verlag, 1979.
  • [28] L. Maligranda and W. Wnuk; Landau-type theorems for variable Lebesgue spaces. Commentationes Math. 55 (2015), 119-126.
  • [29] J. Musielak; Orlicz spaces and modular spaces. Lecture Notes in Mathematics, vol. 1034. Springer, 1983.
  • [30] H. Nakano; Modulared sequence spaces. Proc. Japan Acad. 27 (1951), 411-415.
  • [31] M. Sanchiz; Structure and operators on variable Lebesgue spaces. Ph.D. Thesis, Madrid Complutense University, 2023.